Jump to content

Wikipedia:Reference desk/Science: Difference between revisions

From Wikipedia, the free encyclopedia
Content deleted Content added
Line 614: Line 614:


:What I think they are saying is that while the '''average''' distance of these planets from the Sun may not change by much - the orbits may become more oval...less circular. If that's enough to make the orbit of one planet cross the orbit of another - then KAPOWW!!! - a very big mess! However, the timescale for these events is in the billions of years - more than you, I or humanity in general need worry about. Many of the events described are not due to happen for 5 billion years - and that's about as long as the Sun will last. So it's really kinda unimportant in the grand scheme of things. [[User:SteveBaker|SteveBaker]] ([[User talk:SteveBaker|talk]]) 02:54, 10 June 2009 (UTC)
:What I think they are saying is that while the '''average''' distance of these planets from the Sun may not change by much - the orbits may become more oval...less circular. If that's enough to make the orbit of one planet cross the orbit of another - then KAPOWW!!! - a very big mess! However, the timescale for these events is in the billions of years - more than you, I or humanity in general need worry about. Many of the events described are not due to happen for 5 billion years - and that's about as long as the Sun will last. So it's really kinda unimportant in the grand scheme of things. [[User:SteveBaker|SteveBaker]] ([[User talk:SteveBaker|talk]]) 02:54, 10 June 2009 (UTC)

== Quantum physics paradox ==

Where's the mistake in this formula?
:<math> i \hbar = i \hbar \frac {dp}{dp} = i \hbar \frac {d}{dp} p = xp = x (-i \hbar \frac {d}{dx}) = -i \hbar (x \frac {d}{dx}) = -i \hbar (\frac {d}{dx} x - 1) = -i \hbar (1-1) = 0 </math>
[[Special:Contributions/70.26.154.226|70.26.154.226]] ([[User talk:70.26.154.226|talk]]) 04:48, 10 June 2009 (UTC)

Revision as of 04:48, 10 June 2009

Welcome to the science section
of the Wikipedia reference desk.
Select a section:
Want a faster answer?

Main page: Help searching Wikipedia

   

How can I get my question answered?

  • Select the section of the desk that best fits the general topic of your question (see the navigation column to the right).
  • Post your question to only one section, providing a short header that gives the topic of your question.
  • Type '~~~~' (that is, four tilde characters) at the end – this signs and dates your contribution so we know who wrote what and when.
  • Don't post personal contact information – it will be removed. Any answers will be provided here.
  • Please be as specific as possible, and include all relevant context – the usefulness of answers may depend on the context.
  • Note:
    • We don't answer (and may remove) questions that require medical diagnosis or legal advice.
    • We don't answer requests for opinions, predictions or debate.
    • We don't do your homework for you, though we'll help you past the stuck point.
    • We don't conduct original research or provide a free source of ideas, but we'll help you find information you need.



How do I answer a question?

Main page: Wikipedia:Reference desk/Guidelines

  • The best answers address the question directly, and back up facts with wikilinks and links to sources. Do not edit others' comments and do not give any medical or legal advice.
See also:



June 4

NDE recovery, fiction vs real life

I've seen it a hundred times on the tube: someone is found completely inert, often in water, and there's a moment of suspense – is the character pining for the fiords? – while CPR is attempted; then the rescuee noisily resumes breathing, and immediately is fully awake (though disoriented).

Does that really happen? —Tamfang (talk) 00:20, 4 June 2009 (UTC)[reply]

While it's possible for a victim to regain consciousness, any good CPR will break ribs. Going from unresponsive to verbal (making sounds without any meaning) is probably the best you can hope for. Certainly most CPR will not result in a save, and you can ask anyone in the field. M@$+[[@]] Ju ~ 00:33, 4 June 2009 (UTC)[reply]
In that situation, would the heart necessarily have stopped? If all that is required is mouth-to-mouth then I think full conciousness can return pretty quickly. CPR is normally just to keep the person alive until someone with a defibrillator gets there (and even then, your chances aren't anywhere near as good as TV hospital dramas would have you believe). --Tango (talk) 01:39, 4 June 2009 (UTC)[reply]
Isn't it the case that in Real Life, the vast majority of 'flatlines' still result in death, despite the best efforts of everyone? EDIT: Also, that a defibrillator is useless in this situation, despite what TV tells us? --Kurt Shaped Box (talk) 02:45, 4 June 2009 (UTC)[reply]
Properly done CPR need not break ribs. Edison (talk) 02:58, 4 June 2009 (UTC)[reply]
I have no idea if this is actually true but I remember reading one of those 'true medical confessions!' books ages ago in which an anonymous MD stated that it was not unknown for doctors to deliberately break the patients ribs by performing rough CPR on a patient that they already knew was toast - if the relatives were watching. The thinking being that they'd see that and be assured that absolutely everything that could've be done had been done in an attempt to save the patient... --Kurt Shaped Box (talk) 03:06, 4 June 2009 (UTC)[reply]
That is correct; the ever-popular 'flatline' (beeeeeeeeeeeeeeeeeep) in televised medical dramas is not a shockable rhythm. (See asystole). Cardiac arrests can be divided into two broad groups: those which still include some mechanical action by the heart (ventricular tachycardia, atrial fibrillation), and those which don't (asystole, pulseless electrical activity). The former are susceptible to defibrillation and have a much higher survival rate. The latter aren't shockable, and have a very poor prognosis.
Our article on cardiac arrest notes an overall survival rate of about 15% for in-hospital arrests. (Out of hospital rates are lower.) Patients with shockable rhythms fare about ten times better than those with asystole. TenOfAllTrades(talk) 13:48, 4 June 2009 (UTC)[reply]
There are many studies of near drowning events, unfortunately i don't have access to any that would answer your question. 100% of victims will survive in the short term (otherwise it's a drowning and not a near drowning). Approximately 80% will survive beyond 24 hours, perhaps with some degree of neurological deficit. Many children with cyanosis or hypoxia following recovery resume breathing after clearing the airway and one or two rescue breaths, and are conscious and alert immediately thereafter.
If the victim is in arrest the prognosis is much poorer, hypoxia has been prolonged and the brain is now ischemic. CPR alone will probably not result in a return of spontaneous circulation, let alone regaining consciousness, defibrillation and/or drugs are required. One thing to note tho is that even professional healthcare providers have a poor success rate at finding a carotid pulse during a suspected arrest event. The following scenario most likely could happen: an apneic victim is removed from the water, rescuers begin CPR but fail to note the presence of a pulse. The victim resumes breathing and shortly regains consciousness. Chest compressions were performed but were not required.—eric 15:37, 4 June 2009 (UTC)[reply]

To clarify, my question is not about the odds of survival after (near)drowning or heart attack or whatever, but about the TV cliché of sudden recovery of full consciousness. —Tamfang (talk) 19:02, 4 June 2009 (UTC)[reply]

Penile's Erectiom Angel

plz answeer, how can i measure my Penile's Erection angle ? —Preceding unsigned comment added by Greatfencer (talkcontribs) 01:11, 4 June 2009 (UTC)[reply]

I suppose a mirror might help. —Tamfang (talk) 02:33, 4 June 2009 (UTC)[reply]
Protractor? --Kurt Shaped Box (talk) 02:43, 4 June 2009 (UTC)[reply]
Of course, the angle of the dangle is inversely proportional to the heat of the beat... --Jayron32.talk.contribs 02:50, 4 June 2009 (UTC)[reply]
(EC)Use your goniometer. If you do not have one handy, the angle of the dangle has been said to be inversely proportionate to the heat of the meat, so a Meat thermometer might allow an accurate indirect measurement. Other anatomical surrogate measurements are mentioned in the work cited. Edison (talk) 02:54, 4 June 2009 (UTC)[reply]
The word "goniometer" does not mean "a thing to measure gonads," just as "episcotister" is not a device to test episcopals. Edison (talk) 04:54, 5 June 2009 (UTC)[reply]
(EC)How's about making an appointment for a visit to your local hospital's Penile tumescence lab? For some reason, the hospital seen on House M.D. would appear to have more than one. --Kurt Shaped Box (talk) 03:00, 4 June 2009 (UTC)[reply]
Employ the service of a fluffer who charges by the degree and read the invoice. Cuddlyable3 (talk) 09:53, 4 June 2009 (UTC)[reply]
<applause> —Tamfang (talk) 19:14, 4 June 2009 (UTC)[reply]
Drop a barometer from the top to determine the height, and use trigonometry. I am assuming that you know, or can measure, the length. -- Coneslayer (talk) 13:08, 4 June 2009 (UTC)[reply]
<applause> —Tamfang (talk) 19:14, 4 June 2009 (UTC)[reply]
Take the barometer to a fluffer and say "I'll give you this really nice barometer if you'll measure the angle". SteveBaker (talk) 23:45, 4 June 2009 (UTC)[reply]
I think that when dealing with an erection angel I wouldn't worry about her measurements. APL (talk) 03:49, 5 June 2009 (UTC)[reply]
Does an erection angel work for a sex goddess? --Jayron32.talk.contribs 04:21, 5 June 2009 (UTC)[reply]

<sigh> Okay. What you need is a protractor. That's a half-circle (usually plastic) with marks or scratches indicating the various angles. Stand upright (yeah, erect) and place the center of the flat edge of the protractor against the side of your erect penis, so that the protractor is straight up and down. It will probably be easier to take a measurement from your belly downwards rather than from your balls upwards. HTH. Matt Deres (talk) 23:58, 5 June 2009 (UTC)[reply]

Hmmm - perhaps an inclinometer - it measures your inclination. SteveBaker (talk) 03:19, 6 June 2009 (UTC)[reply]

I'm not sure why anyone would care about the angle . . . as long as the wee fella is stiff the angle really does not matter . . .

It's one half of a Parabola, anyway, thus a conic section. Edison (talk) 05:26, 10 June 2009 (UTC)[reply]

global warming and human water retention

06:34, 4 June 2009 (UTC)Paul fitts (talk)What year did the true science of global warming start? what was the Earth's population at that time? What is the percentage of the human body that is made of water? If you had a 3ft cube of ice, and you melted it...how much water would that be in gallons??

the main reason for my questions.....it doesn't really apprear that sea levels are rising, so if ice caps and glaciers are "melting", and the water levels aren't really rising.....wouldn't it stand to reason that, that the "melted" water has to go somewhere, why not human water retention to make up 3+ billion more we've created over ther last 30 years???


Paul Fitts

The Tuvaluans beg to differ. According to this Reuters article, their whole country could disappear under the waves in 30-50 years. Another factor (which I was reminded of by An Inconvenient Truth) is that if the glaciers are in the water, their melting won't raise the water level. It's when the land-based ice melts that we have to worry. Clarityfiend (talk) 07:29, 4 June 2009 (UTC)[reply]
We have, of course, and article at Current sea level rise. Sea level is rising several mm per year. I'm too lazy to work in feet and gallons (which gallons, anyways?), but one cubic meter of ice has 1000 l and will melt into very roughly 900 l of water. --Stephan Schulz (talk) 07:36, 4 June 2009 (UTC)[reply]
Google can convert between units, put in something like "3 cubic feet in gallons". After that go outside and watch some grass carefully for a few hours. Did you see it grow? Dmcq (talk) 08:00, 4 June 2009 (UTC)[reply]
To put this in context - sea levels are rising a few millimeters each year - but each millimeter of rise represents 360,000,000,000 cubic meters of water. There are about 7 billion people - even if each of us was retaining a cubic meter of water (not even close!) we'd represent only about 0.02 millimeters of ocean depth. No - the reason the rate seems low is that 360,000,000,000 cubic meters means that it takes an awful lot of water to raise all of the oceans in the world by one millimeter. But while a few millimeters may not sound much - over 100 years, that's enough to drown quite a few coastal cities. Sadly, the evidence is that the rate of increase is going up year on year - so we could easily have a dozen or more meters of ocean level rise during the lifetimes of our children - of the younger Ref.Desk denizens. SteveBaker (talk) 15:00, 4 June 2009 (UTC)[reply]

Why are washers so called?

It was one of those idle, late night conversations in the tour van on the way home from a gig ... which led precisely nowhere. My theory is that the bigger examples are called penny washers because they are the size of pre-decimal pennies, but what about smaller varieties, and where does 'washer' come from? Any ideas please? Turbotechie (talk) 07:58, 4 June 2009 (UTC)[reply]

According to [1] (80% down the page) the origin of the term "washer" for that piece of metal is unknown, though it has had that meaning for at least 400 years and perhaps as many as 650 years. Dragons flight (talk) 08:23, 4 June 2009 (UTC)[reply]
Outside a hardware store hung the alarming sign "Nut screws washer and bolts". Cuddlyable3 (talk) 09:42, 4 June 2009 (UTC)[reply]
Not forgetting there was a launderette next door Mikenorton (talk) 10:01, 4 June 2009 (UTC)[reply]
You are right that penny washers are named for the old pennies, and often they are a similar size, but the term can actually be applied to any size washer. It is a washer with a disproportionately small centre-hole. SpinningSpark 16:34, 4 June 2009 (UTC)[reply]
Just for my own education, do you have a source I can read up on SSpark? I would frame it the other way, since washers are designed for the inserted item, so rather than "disproportionately small centre-hole", I would say "disproportionately large outside-diameter". In my very limited engineering experience in a very specialized field, we termed these "standard" and "frictional" washers, where "frictional" were the big ones, designed to offer a larger weight-bearing surface, as opposed to the I/D / O/D sizes needed to transfer load from a bolt head to a typical clearance hole. Franamax (talk) 23:42, 4 June 2009 (UTC)[reply]

Do facial products work?

Is there any scientific evidence that any face creams, anti wrinkle creams, eye treatments etc are any better than just splashing water on your face or is it really just hype? Kirk Uk —Preceding unsigned comment added by 87.82.79.175 (talk) 09:26, 4 June 2009 (UTC)[reply]

All 3 are better than water at generating profit for someone. Medical eye treatments have to be certified as safe. Cuddlyable3 (talk) 09:46, 4 June 2009 (UTC)[reply]

Many of the anti-wrinkle creams are proven to give a temporary lift and do so by using proven science, similarly darkness-removal creams can be proven to remove the appearance of darkness by masking/covering it. You may notice that in adverts of these types the claims are always quite vague (to paraphrase Charlie Brooker)...terms such as "98% of respondents agree", "help reduce appearance of", "helps fight" are all very 'vague' and undetailed - throw in a few random science-sounding (merged with natural-sounding) words and you've got something that says nothing when reviewed by a legal team but can suggest 'proof' to the average consumer. 194.221.133.226 (talk) 10:22, 4 June 2009 (UTC)[reply]

In the UK, they have to say "improves the appearance of wrinkles" rather than "removes wrinkles" on the ads now. There was one company a couple of years back that got absolutely castigated for making completely false claims about the abilities of their product (Google for 'Boxwellox'). Not quite as bad as the toothpaste that claimed to be able to split water molecules, producing free oxygen for a deeper clean - but still... --Kurt Shaped Box (talk) 10:47, 4 June 2009 (UTC)[reply]

Sunscreen definitely helps us against wrinkles. It can be scientifically tested that protecting your face against UV rays will make you look younger than you are. For example, faces of truck drivers that have been laterally exposed to sun light have been analyzed, and the half exposed to sun light looked older than the other half.--Mr.K. (talk) 10:57, 4 June 2009 (UTC)[reply]

It helps preventively against wrinkles, by the way. --Mr.K. (talk) 12:01, 4 June 2009 (UTC)[reply]
The big hype of these things in the UK ended quite a few years ago when the manufacturers were faced with either having to downsize their claims - or be treated as medical/pharmacuticals. The fair trade people argued that if they ACTUALLY reduced wrinkles, then these creams must be penetrating the skin and acting on the tissues beneath - which would require them to be classified as drugs. If all they do is fill in the wrinkles - or change their color/reflectivity to make them temporarily less noticable - then that's OK, it's just a cosmetic effect. I don't understand why that's not also the case in the USA. Certainly the claims they make are ridiculously impossible - and if they were possible, these cosmetics would certainly have to be seriously tested because they could have any number of dangerous side-effects. To the extent that they block sunlight, they might work...but their effects are essentially just changes in appearance. SteveBaker (talk) 14:52, 4 June 2009 (UTC)[reply]
Well, they also work as well as any other Moisturizer; that is by providing a barrier against evaporation, they cause the underlying skin to retain more moisture. Higher moisture content equals plumper epithelial cells, and plumper cells equals less wrinkles. Of course, a $5.00 bottle of any decent mositurizing lotion will do that; dropping $40.00 on a small 4 ounce tub of the same stuff mixed with a little make-up to cover over dark patches seems excessive to me... But then again, I'm not an aging woman trying to recapture my lost youth, what do I know. --Jayron32.talk.contribs 17:34, 4 June 2009 (UTC)[reply]
Hey, wait, here's [2] a SCIENTIFIC assessment of a particular facial product sold by that 'well known high street chemist'. Whether you are impressed by the fact that 43% of the meagre sample thought their skin had improved is up to you. 23% of the placebo sample thought their skin had improved. Richard Avery (talk) 17:38, 4 June 2009 (UTC)[reply]
A study involving 50 people; I assume half received the placebo and half the cream? That means of 25 receiving the placebo, 6 liked it, and of 25 receiving the cream, 12 liked it? I wouldn't call such small sample size "statistically significant". The error bars on a sample size of 25 would probably be bigger than the difference between the samples. Heck, I could flip a coin 25 times and get the same results. Additionally, as the sample did not compare the expensive treatment to a cheap one, only to a placebo, it only addresses the possibility that the expensive cream is better than nothing but not neccessarily better than cheaper alternatives. What you have here is a clear example of How to Lie with Statistics. --Jayron32.talk.contribs 17:45, 4 June 2009 (UTC)[reply]
Ah! "Buy our amazing anti-wrinkle cream - it has a one in five chance of being better than smearing lard on your face." SteveBaker (talk) 19:51, 4 June 2009 (UTC)[reply]
But lard has the advantage of making you smell like pie crust and biscuits. Who wouldn't want to smell fresh-baked pie crust all day? --Jayron32.talk.contribs 20:44, 4 June 2009 (UTC)[reply]
If I did it right, that's a 99.9999999635% significance level. If I did it wrong, it's even higher. This should be a two-tailed T-test (try saying that ten times fast), right? There may be problems with that study, but sample size isn't one of them. — DanielLC 16:48, 5 June 2009 (UTC)[reply]
According to the Mayo Clinic, "Research suggests that some wrinkle creams contain ingredients that may improve wrinkles. But many of these ingredients haven't undergone scientific research to prove this benefit. If you're looking for a face-lift in a bottle, you probably won't find it in over-the-counter (nonprescription) wrinkle creams. But they may slightly improve the appearance of your skin, depending on how long you use the product and the amount and type of the active ingredient in the wrinkle cream."[3] A Quest For Knowledge (talk) 17:58, 4 June 2009 (UTC)[reply]
(EC)::There is Absorption (skin). If any of the products achieve that the questions become: Do you want the substance that gets absorbed in your system? and Does your body do anything with the absorbed substance in the layers of skin that is beneficial or does the absorbed substance just accumulate or get transported away? There are some things like Botox which is a neurotoxin and Silicone or animal derived Collagen that can be injected into the skin. Since they work "internally" they get treated as medical/pharmacutical. Nevertheless they have come under criticism because the two questions above didn't come up with consistent answers when asked by different reviewers. The cosmetics industry is trying to stay away form the cost of having to get their products subjected to licensing for pharmaceuticals (see Cosmeceutical). So they have to come up with things that do not "affect the structure or function of the human body". The reputable ones also try to make sure that their products do not contain known toxins or substances that cause harm in the short term (long term use sometimes reveals things that didn't show up in testing.) Probably more driven by their desire to avoid costly product liability suits than anything else. Adding Surfactants like soap to water makes it a more efficient detergent. It also removes the protective layer of lipids covering the skin and knocks ph of the acid mantle out of whack. To counterbalance that you can apply oils to your skin after washing. Since using pure oils would give you an unpleasant and impractical film of oil on your skin what we use are emulsions. (Also see Cold cream). Since those are prone to Rancidification and microbial growth you'll not only find emulsifiers and stabilizers but also preservatives in those. (Sometimes cleverly marketed as new Vitamin ingredient) In a further step the cosmetics industry then created Moisturizers. Splashing water on your face would not have the same effect, rather the opposite actually. It would rinse away some more oils exposing cells to evaporation and upset the hydrostatic balance causing further drying. It works through constricting fine blood vessels in the skin. AFAIK interest in reducing wrinkles has only gained momentum throughout the past 70 years or so. That along with our increasing knowledge of biochemical processes in the human body has led to many theories being put forward and being rebuked in that respect. The focus used to be the moisture contents of the stratum corneum. Now things like electrolyte balance, Free radicals, cell aging and nerve stimulation, among others, are under study. Some of those studies are either financed by the cosmetics industry through grants, or done in their labs. Study results from company owned labs are rarely published. (If one of their labs found the holy grail in anti-wrinkle treatment I bet they'd gladly spin off a subsidiary and go through the pharmaceutical trials.;-) Facial products work in the regard that they don't leave greasy marks on your clothes or things you touch, don't spoil fast, smell pleasant and supply lipids to your skin. A cooling effect while the continuous phase of the emulsion evaporates is also well established. Product differentiation has companies add various ingredients that at best can provide better product performance and at least not cause any harm. You'd have to research each labeled ingredient separately to get an idea. It is very likely though that lots of substances aren't listed or any research studies are under lock and key at the company. 71.236.26.74 (talk) 00:04, 5 June 2009 (UTC)[reply]

To DanielLC, your analysis of the Boots anti-wrinkle cream data is not correct. The data uses discrete variables, not continuous variables. There are two categories of subjects: those with placebo, and those with the new cream. There are two categories of outcome: improvement, or no improvement. We need a chi-square test. The article indicates a total of 60 people in the trial. Let's assume that 30 people were in each group.

Observed values
Cream type Improvement No Improvement Total
New cream 13 17 30
Placebo 7 23 30
Total 20 40 60


Expected values
Cream type Improvement No Improvement Total
New cream 10 20 30
Placebo 10 20 30
Total 20 40 60


Running a chi-square test (without Yates correction) gives a probability of 0.10

This is not statistically significant (by usual criteria). Axl ¤ [Talk] 20:36, 7 June 2009 (UTC)[reply]

Although this uses a discrete distribution, it can be approximated as normal distribution. I suggest we move this to the math desk and ask them what test to use. — DanielLC 22:17, 10 June 2009 (UTC)[reply]
Feel free to ask at the maths desk, although I am not particularly interested in their opinion. Perhaps you would consider showing your working to demonstrate your conclusion? Thanks. Axl ¤ [Talk] 16:19, 11 June 2009 (UTC)[reply]

What happens to toxic sewage sludge?

About 40 - 60% (depending on whether you are in the EU or USA) of treated sewage sludge (biosolids) is reused as agricultural fertilizer. From what I can see on the article on biosolids, the main reason some biosolids cannot be used as fertilizer is that they have a heavy metal and toxic substance content that is too high. Is this true? If so, what happens to this waste? Is it incinerated, landfilled etc? —Preceding unsigned comment added by 157.203.42.175 (talk) 13:23, 4 June 2009 (UTC)[reply]

According to this parliamentary note [4], in the UK 62% is applied to agricultural land, 19% is incinerated, 11% is being used in land reclamation with the remainder going to landfill or composting. It also discusses the use of sludge to generate energy from biogas through anaerobic digestion. The report does not mention toxicity as a problem, apart from the issue of Endocrine-Disrupting Chemicals. Mikenorton (talk) 14:11, 4 June 2009 (UTC)[reply]
Sludge has some information. 71.236.26.74 (talk) 04:10, 5 June 2009 (UTC)[reply]

Frozen peas float when cooked

When you put frozen peas in a saucepan of cold water they sink to the bottom. When the water is heated up the peas float to the surface. But as ice is lighter than water, and frozen peas must contain some ice, I would expect the opposite to happen, i.e. they would start off floating then sink when heated. What is going on here? Lonegroover (talk) 14:45, 4 June 2009 (UTC)[reply]

Peas contain lots of stuff besides just water, so maybe enough to overcome the buoyancy from decrease in density of the water being frozen. Do the peas remain the same size, or do they expand when they thaw/heat? Does this happen only if the water gets hot, or can you reproduce it with room-temp water (to exclude nucleated steam giving the lift)? DMacks (talk) 14:53, 4 June 2009 (UTC)[reply]
Don't forget that water is densest at about 4C. My guess (and it really is just speculation) would be the following:
Water, in a saucepan at about 10 degrees C, combined with peas at about -18C. Peas initially float in cold water before sinking so clearly the water is denser for a short period. Presumably due to the low temperature of the peas? I would assume that the peas rapidly warm due to their high surface area to volume ratio. As such, the water within the peas would be denser than the water in the pan. Peas have other constituent components, but I would suggest the density of the water in the peas outweighs and (lower) density from the solid matter of the pea. The overall density of the pea should approach the point where they are denser than the surrounding water - this shouldn't be too difficult, the water will be getting colder from the peas but also warmer from the heat input.
I would imagine that the peas remain denser (colder) than the surrounding water for a while - since the source of heat would warm the water first, then the peas When the water boils, the peas could reach a temperature equilibrium with the water (since the water cannot get hotter, regardless of heat input, without turning to steam), or at least become warm enough such that the weighted average of the density of the pea (i.e. the solid matter and water contained within the pea) could overcome the density of the hot/boiling water. —Preceding unsigned comment added by 157.203.42.175 (talk) 15:07, 4 June 2009 (UTC)[reply]
The obvious answer is thermal expansion of the peas. Assume the peas have a density near water, but slightly more dense. This is because they are mostly water, but with some solid materials. As the peas heat, their radius expands by some small amount - but their volume increases according to (radius3), so their density will decrease inversely to that. Although water does change its density slightly, it's generally a good assumption that it is an incompressible fluid. Its density should not change significantly between room temperature and near-boiling. (Our article gives about a 5% change, but I don't know if I trust the source data). But, the water inside the peas will also expand - and so the only relevant volume change is the thermal expansion of the solid materials in the pea. Nimur (talk) 14:44, 5 June 2009 (UTC)[reply]

What happens if I drink five-year-old soda?

Not fridged, the 12-pack wasn't even opened. Just curious. I think I'll throw it out anyway. 67.243.7.41 (talk) 15:34, 4 June 2009 (UTC)[reply]

OR having tried it once. It won't be toxic, but it may be unpalatable as the bubbles will be gone and it may be a bit sludgy on the bottom of the can. 65.121.141.34 (talk) 16:23, 4 June 2009 (UTC)[reply]
This is not medical advice BUT: you might gain super powers. It's a possibility. Consider your future life as Soda Man, and whether or not you are willing to take on that responsibility. --98.217.14.211 (talk) 16:55, 4 June 2009 (UTC)[reply]
Throwing it away it probably wise. I doubt there will be anything harmful about it, but it probably won't taste very nice (depending on how it was stored). It is possible the seal has been broken somehow which might have allowed something harmful to get in, so probably not worth the risk. --Tango (talk) 18:38, 4 June 2009 (UTC)[reply]
Sugar soda may retain its flavor longer than artificially sweetened soda. I once had some old pop sweetened with Nutrasweet and all sweetness was gone. It tasted like unsweetened Coke with pineapple juice added. Heat speeds the breakdown of Nutrasweet. There is always a possibility that over time there could be greater leaching of metal or plastic from the can into the drink. If it is sealed, how would any carbon dioxide escape to make it flat? Edison (talk) 18:42, 4 June 2009 (UTC)[reply]
Major OR here, but I had some cans of orange soda that actually started leaking through the bottom of the cans after eight or nine years. They were unopened and keep in a closet. The soda actually caused corrosion and leaked out. cheers, 10draftsdeep (talk) 19:38, 4 June 2009 (UTC)[reply]
If its in a can, the soda may have eroded away enough of the can to impart a metallic taste. Don't know if its toxic though. Livewireo (talk) 21:22, 4 June 2009 (UTC)[reply]
Most modern cans have a coating of plastic on the inside I do believe. If the (presumably) citric acid in the orange soda and/or the carbonic acid in any soda has gotten through to the metal of the outside of the can, it's likely also dissolved the plastic coating on the way. This would presumably include any phthalate and/or bisphenol components of the coating plastic. Luckily, most plastic compositions are considered trade secrets, and there is intense dispute over the bio-effects of various plastic additives (softeners, etc.) which may or may not be in the plastic anyway (trade secret!) - so we can just dismiss the whole thing as "no definite evidence". Franamax (talk) 23:33, 4 June 2009 (UTC)[reply]
Check this out: Benzene in soft drinks, then check the label of your soda can. It won't kill you (immediately) but it's not healthy either. Also "a soft drink such as a cola has a pH of 2.7-3.0 compared to battery acid which is 1.0". That and the other ingredients, plus the can should make for an interesting environment for many chemical reactions and interesting compounds to form over time. 71.236.26.74 (talk) 00:26, 5 June 2009 (UTC)[reply]
Of course, you do realize that a pH of 3.0 is 1% as acidic as a pH of 1.0. Thus, soda has about 1% of the acid concentration of battery acid. Of course, the liquid in your stomach is pH of around 2 or so, which means that the soda is only 10% as acidic as your own gastric juices, or if you prefer, your gastric juices are 10 times as acidic as soda. Welcome to the wonderful world of logarithms. --Jayron32.talk.contribs 04:17, 5 June 2009 (UTC)[reply]
That's one of the reason I put that quote from an ooold paper in quotation marks. Your stomach is however a pretty good example of an environment that has interesting chemical reactions happening. 71.236.26.74 (talk) 06:00, 5 June 2009 (UTC)[reply]
Note that you generally don't want to drink any non-diet soda with a broken seal that's older than a few days - once it gets exposed to the environment I would expect the bacteria to come in and party on the sugar. A diet soda on the other hand might be fine for a longer period of time. I'm not sure. Dcoetzee 06:39, 5 June 2009 (UTC)[reply]
Nope Aspartame is less stable that sugar and gets broken down after a while. The interesting thing is what reaction products you're going to get. 71.236.26.74 (talk) 07:28, 5 June 2009 (UTC)[reply]

Capturing a warm bath's heat

It's winter in the southern hemisphere and a nice way to heat up is by taking a warm bath. I couldn't help thinking though of all the energy that gets lost when the warm water flows out the drain after a bath. Given that:

  • a kilocalorie is the amount of energy it takes to heat a liter of water by 1 degree Celsius
  • lets assume my bathtub holds 150 liters of water
  • it's about 17 degrees in my apartment, the water after I'm done bathing is 40 degrees Celsius, a 23 degree difference

I asked Google: "(150 * 23) * kilocalories in kilowatt hours" and got "(150 * 23) * kilocalories = 4.00966667 kilowatt hours"

That's 4 kilowatt hours of energy down the drain! That's like a 1000 watt heater staying on for 4 hours. so this makes me ask, does it make sense to keep the water in the bathtub until it has cooled down so that my place will heat up a bit? One concern is evaporation that might cause the humidity to rise and thus the energy is kept as latent heat rather than actual. In that case, is there something simple one can do to prevent evaporation?

196.210.200.167 (talk) 16:14, 4 June 2009 (UTC) Eon[reply]

Any added humidity will make you feel warmer, which is just as good, isn't it? --Sean 18:25, 4 June 2009 (UTC)[reply]
I'm not sure. The formulas are given in Fahrenheit, but I suspect at 17 degrees a higher humidity might even make you feel colder. Seems counter intuitive though that just leaving the water in the bathtub can make such a huge difference. 196.210.200.167 (talk) 19:39, 4 June 2009 (UTC) Eon[reply]
A lid on the bath would prevent evaporation. --Tango (talk) 18:36, 4 June 2009 (UTC)[reply]
I don't see why leaving the water in the bath until it's cooled right down wouldn't work. The humidity might be a problem for your bathroom - but you'll certainly have a slightly lower heating bill if you do this. It's not a bad idea actually. SteveBaker (talk) 19:44, 4 June 2009 (UTC)[reply]
According to my electric bill, 4kWh is worth almost 35c, though I would imagine that it would not apply the heat evenly throughout your dwelling (unless you have incredible outer wall insulation)65.121.141.34 (talk) 19:53, 4 June 2009 (UTC)[reply]
Don't have the time to google for it, but there is a product that recovers heat from water and air leaving the house through bathroom vents and plumbing. Can't recall what it was called. (I think "this old house" had one in one of their shows)71.236.26.74 (talk) 00:31, 5 June 2009 (UTC)o.k. found a couple of pages [5] [6], Matt's comment here [7], [8] Ugh, should have known and we do have a page Waste Heat Recovery Unit - 71.236.26.74 (talk) 03:59, 5 June 2009 (UTC)[reply]
My contingency plan for what to do in the event of a prolonged power failure in winter includes leaving the hot (and a bit of cold) water trickling into bathtubs and sinks, to keep the pipes and drains from freezing and to heat the house from the water heater. This assumes the overflow can remove the water with the drain plugged. A diverter to direct the hot water to the far end of the tub might be useful. One would get tired of a batch mode of filling the tub and then draining it after an hour, repeated 24x per day. Fireplace and kitchen range (hob) or oven could also be used to advantage, but with suitable care for carbon monoxide. Edison (talk) 04:49, 5 June 2009 (UTC)[reply]

gravitational equations

Is/are the equation/equations for gravity at the center of a Black Hole the same as the equation/equations for gravity in unoccupied (empty) space? ---- Taxa (talk) 17:20, 4 June 2009 (UTC)[reply]

At the very centre of a black hole, the singularity, the laws of physics break down (ie. we don't really know what happens), so there are no equations. In an appropriate coordinate system (eg. Kruskal–Szekeres coordinates) you can use the same equations to describe everywhere except the singularity, though. --Tango (talk) 18:47, 4 June 2009 (UTC)[reply]
The singularity at the very center of a black hole does things to the equations that physics depends on that are essentially the same as dividing by zero on your pocket calculator. There is no meaningful answer. But a billionth of a trillionth of a gnat's eyebrow from the center, the laws of physics should be pretty well-behaved. The equations are exactly the same - but because these are rather extreme circumstances, you have to use the full relativistic forms of these equations, not the 'low speed/low gravity' approximations that we all learned in high school. SteveBaker (talk) 19:41, 4 June 2009 (UTC)[reply]
Eyebrows?
Eyebrows?
What is the length of a gnat's eyebrow in Planck lengths? Does anyone know? --Tango (talk) 02:08, 5 June 2009 (UTC)[reply]
Well, the photo at right claims to be 100x magnification. But (...pet hate...) I'm viewing this simultaneously on a 14" monitor and an 81" plasma screen...so it's a bit hard to measure exactly! SteveBaker (talk) 04:04, 5 June 2009 (UTC)[reply]
It's a featured picture as well, I'm surprised nobody caught that... --Tango (talk) 12:29, 5 June 2009 (UTC)[reply]

DO GRAVITATIONAL ENERGY OF EARTH REDUCE BY TIME?

Earth keeps the moon in it's gravity spending it's energy .From where do earth gain this energy? If earth do not gain energy,then is it's gravitational energy reducing by time?Surabhi12 (talk) 18:06, 4 June 2009 (UTC)[reply]

The earth does not spend energy in order to keep its moon. Dauto (talk) 18:19, 4 June 2009 (UTC)[reply]
Well, not in classical physics. But in relativistic physics, the circling moon will cause gravitational waves, which will carry away some of the potential energy of the Earth-Moon system. The effect is very small, and, at the moment, entirely overshadowed by the tidal transfer of rotational momentum from the Earth to the Moon. See Gravitational wave#Power_radiated_by_Orbiting_Bodies. --Stephan Schulz (talk) 18:26, 4 June 2009 (UTC)[reply]
The moon is falling towards the Earth, but luckily it keeps missing, just like cannonball "C" shown in the picture. It doesn't take any energy for this to happen. See orbit. --Sean 18:31, 4 June 2009 (UTC)[reply]
And the Earth is falling towards the moon, but luckily the moon keeps moving out of the way ;) Gandalf61 (talk) 12:30, 5 June 2009 (UTC)[reply]
Somebody owes me a dollar. (I've decided that I'm charging the universe a dollar every time someone confuses a 'force' (gravity in this case) with 'energy' - I plan to earn enough money from this confusion to replenish my poor 401K). The force and energy are very different things. When you stick a fridge magnet onto your fridge, the magnet exerts a force on the metal. But so long as the magnet doesn't move - it doesn't take any energy whatever for it to just hang there...magnets don't "run down". It's the same with the moon. The gravity pull between earth and moon is just a force. So long as the two bodies don't get closer or further apart - there is no energy transaction involved. Forces don't "run down" - they just are. If you hang something from the ceiling with a rope - the rope exerts a force...the rope doesn't "run down". Now, if something were to pull the moon further from the earth - that 'something' would have to expend some energy to do it...if the moon (for some reason) were to fall closer to the earth - then it would actually gain energy (kinetic energy initially - then one hell of a lot of heat energy when it kersplatted into the middle of the pacific ocean!). SteveBaker (talk) 19:35, 4 June 2009 (UTC)[reply]
Would another explanation be that the Earth is neither exerting a force nor expending energy, it is simply warping space-time such that the shortest path for the Moon to travel through the geodesic happens to be a circle? Just asking... Franamax (talk) 20:07, 4 June 2009 (UTC)[reply]
Now now, don't confuse a good discussion with a simple, elegent and correct answer. No one needs that now! --Jayron32.talk.contribs 20:34, 4 June 2009 (UTC)[reply]
Yes, but relativity gives people headaches! That, and the concepts Steve is talking about apply to all forces, even ones that can't be easily dismissed as mere geometry. --Tango (talk)
I'll give you a dollar. Remind me if see you - bring change! --Tango (talk) 02:07, 5 June 2009 (UTC)[reply]
Oooh! Thank-you! Actually - with the number of people making this error - I'm pretty sure I can still turn a profit at one Zimbabwean dollar per confused OP.  :-) SteveBaker (talk) 03:58, 5 June 2009 (UTC)[reply]
Single Zimbabwean dollars are now so rare that, pervesely, they are probably now valuable to collectors. SpinningSpark 13:48, 5 June 2009 (UTC)[reply]
and another piece of trivia (I can't leave this one alone) is according to this the $US is now worth more than a mole of the original $ZW. That must be a first for a currency. SpinningSpark 14:17, 5 June 2009 (UTC)[reply]

problem of race

Imagine:

A car travels 10km in an hour 
A bus travels 20km in an hour  
both have a race
Car is ahead of bus at point A.
By the time bus moves to the point A,car must have move little ahead say to point b.
By the time bus moves to the point b,car must have move little ahead again say to point c.
By the time bus moves to the point c,car must have move little ahead say to point d.
This continues .........
Thus car has to become the winner.
IS THIS POSSIBLE?  —Preceding unsigned comment added by Surabhi12 (talkcontribs) 18:30, 4 June 2009 (UTC)[reply] 
See Zeno's paradoxes. Short answer - it takes an infinite number of steps for the bus to overtake the car but because each of those steps takes sufficiently less time than the one before the total amount of time required to overtake is finite. See convergent series for the maths behind that. --Tango (talk) 18:35, 4 June 2009 (UTC)[reply]
Another way to look at this is that suppose the car starts off with a lead of (say) 10km. We know that using 'sensible' math, that the bus will travel at a distance D in time T=D/20 hours and the car at time T=(D-10)/10 hours. When the bus overtakes the car, they are at the same distance at the same time - so we can solve the simultaneous equations and calculate T as 1 hour. The bus catches the car after 1 hour - then zooms right past it. No problem, no controversy, no paradox.
But the crazy Zeno's paradox way says: The time it takes the bus to reach A is 1/2 hour. By that time, the car has travelled 5km to point B. 15 minutes later, the bus reaches B and the car has gone on another 2.5km to C. 7.5 minutes later, the bus reaches C...so Zeno tells us where the bus and the car are after 30+15+7.5+3.75+1.875+... minutes. Mathematically, that infinite series adds up to 59.999... minutes. And the distance that the car and bus have travelled in that time is 19.999...km from the start. Well, that's all very interesting and exciting - but by refusing to every allow the 'victim' of the paradox to just go ahead and ask where the vehicles are after a longer amount of time, he arbitarily forces us to look only at times before the two vehicles actually meet. If you limit your calculations to only times before the vehicles meet - you're obviously never going to find the time when they do actually meet. It's not a paradox - it's a dumb way of stopping the person from answering a ridiculously simple question!
But why doe Xeno insist on calculating all of these intermediate positions and stubbornly refuse to calculate where they are after exactly one hour? Because he's some stupid philosopher trying to make a name for himself by inventing a "paradox". This is why philosophers are a waste of quarks. We have a perfectly simple, completely understood problem with a VERY simple, non-paradoxical conclusion...but NO...the dumb-as-a-bag-of-hammers philosopher has to insist on never calculating the answer but instead answering an infinite series of questions that we don't need to know the answer to.
It's really no different to me saying "What is 2+2?" - but instead of just going ahead and counting it out on your fingers, I arbitarily insist that you first calculate a totally unrelated sum: 1+0.5+0.25+0.125+... which (if you try to do it the hard way) will take you an infinite amount of time - and thereby cunningly prevent you from calculating 2+2. Why the heck would you ever want to do it like that? It's obviously a stupid and unnecessary way to answer my question. So - please treat anything any philosopher says much as you would a comedian. Amusing, possibly mildly entertaining - but in no way relating to reality. SteveBaker (talk) 19:22, 4 June 2009 (UTC)[reply]
Well that and your number series will only equal 2 when you are done. 65.121.141.34 (talk) 19:49, 4 June 2009 (UTC)[reply]
I disagree - I think it is instructive to consider the "paradox" and realize that it is solved by convergence. The obvious logical solution allows the student to intuitively grasp the otherwise abstract idea that adding an infinite number of elements can still lead to a finite solution. This does have practical consequences in the study of advanced science and physics - it's an effective way to consider the parity between discrete and continuous number representations, or quantized vs. continuous physical models. Nimur (talk) 23:19, 4 June 2009 (UTC)[reply]
Perhaps the solution is to tell the philosopher you think he's wrong and ask him to write out the whole series just to be sure... Franamax (talk) 20:10, 4 June 2009 (UTC)[reply]
It is a little unfair to say philosophers are a waste of quarks in reference to ancient Greek philosophers. I agree modern ones are a waste of quarks, but in ancient times "philosophy" was a far broader subject since other things didn't exist yet. It included mathematics and what would now fall under science (but I'm not willing to call what they did science, since it didn't follow the scientific method). It easy for us now to laugh at them not understanding these simple concepts, but they are only simple to us because someone else has explained them to us. The ancient Greeks (and philosophers for quite a while after them) had some very odd ideas about infinity (as do most people today who don't have formal training in mathematics). The reason Zeno's problem seemed paradoxical to them was because they didn't think it was possible to do an infinite number of things (the idea that they got easier and easier so the total amount of resources required remained finite wasn't known to them). You may find Supertask interesting. --Tango (talk) 02:03, 5 June 2009 (UTC)[reply]
I am reminded of a quote attributed to Johannes Keppler, though it may be apocryphal. I will paraphrase. Basically, Keppler was lecturing on the organization of the solar system, and one of his students asked something like: "Weren't people 100 years ago rather stupid, I mean, they thought the earth was the center of the Universe, and that everything revolved around us. Doesn't that that make those people rather unintelligent." To which Keppler replied something like "If they were right, and the sun DID revolve around the Earth, would the sky look any different?" which makes the point that its easy given the sum of all human knowledge of today to riducule the past thinkers as somehow stupider than us; but we have the benefit of the incremental progress we have made towards understanding the universe, a process they themselves were a part of. One could just as easily have commented that Newton was an idiot for not taking into account relativistice effects of near-light-speed travel, or that proponents of phlogiston theory were stupid when they thought heat was a substance. 100 years from now people will think we are stupid because some "scientific fact" we are certain is right turns out to be inaccurate in some way. Xeno's ideas look stupid because he didn't have the 2000 years of mathematical thought already spelled out when he devised his paradoxes. However, the basic concept that infinity is a special idea that DOES need its own set of rules to deal with was a unheardof idea at Xeno's time. The fact that anyone was thinking about the infinite several hundreds of years B.C. is pretty prescient if you ask me. --Jayron32.talk.contribs 02:52, 5 June 2009 (UTC)[reply]
I'd just like to point out that 59.999...=60. You could have just said the infinite series just adds up to an hour. Also, this paradox was formulated before infinite series (it isn't just what you get when you add all the values together), so you could only get something below 60, but arbitrarily close to it. — DanielLC 16:16, 5 June 2009 (UTC)[reply]
That's true - but it doesn't get Zeno off the hook. If we ask when does the hare/Achilles/bus pass the tortoise/car? Or better still we can be REALLY clear and ask: "When is Achilles ten feet ahead of the tortoise?" Then Zeno is arguing that they just meet at T=59.999...=60 - but he's not letting you find out whether the hare ever PASSES the tortoise. So this rather wonderful mathematical result doesn't help us out much! SteveBaker (talk) 02:59, 6 June 2009 (UTC)[reply]
Of course, Zeno wasn't actually trying to find out whether or when Achilles passed the Tortoise; everyday observation made it obvious that moving things routinely passed other things. Rather, as Nimur alluded to, he was examining philosophical postulates made by others and using this paradox (alongside several others) to demonstrate their invalidity (much as Ernst Schrödinger didn't really believe a cat could be simultaneously alive and dead, his gedankenexperiment was meant to demonstrate that the concept of quantum superposition was absurd).
In Zeno's time, some philosophers had postulated that space was (S), others that it was not (s), infinitely divisible; ditto for time (T, t), giving 4 possible combinations (ST, St, sT, st). Zeno constructed a paradox for each of the four scenarios, apparently showing that each was demonstrably untrue (although the st case apparently is true). Perhaps if he'd had Leibnitz's and Newton's benefit of a more manipulable number system, he'd have managed to come up with calculus a couple of millennia earlier. 87.81.230.195 (talk) 16:38, 6 June 2009 (UTC)[reply]

Labour and multiple births

Given that Twins, and higher number births, are individuals, do mothers ever experience a sort of delayed labor, in which the second child is born at a vastly different time or even date then their sibling? For example, Paul and John are twins, Paul is born first, and John is born 2 days later, after the mother reentered labor.--HoneymaneHeghlu meH QaQ jajvam 19:31, 4 June 2009 (UTC)[reply]

When someone finds a good source on the range, with some statistics, please add the data to the Multiple birth and Twin articles, which should discuss this, but currently lack any data (or even a mention). Tempshill (talk) 19:53, 4 June 2009 (UTC)[reply]
If anyone wants to ask at the WP:Library for the fulltexts, here are two sources: "Management of Delayed-Interval Delivery in Multiple Gestations", S. Cristinelli et al., Fetal Diagnosis & Therapy, Jul/Aug 2005, Vol. 20, Issue 4, pp.285-290. (AN 18247478) [9] and "Delayed delivery of multiple gestations: Maternal and neonatal outcomes", M. Kalchbrenner et al., American Journal of Obstetrics & Gynecology, 179(5), pp1145-1149, Nov. 1998. [10]. According to the abstracts: in 6 cases studied, the delivery interval is 2:93 (median 7) days; the literature (148 cases from 1979-2001) supports 2:153 (median 31) days; and for 7 cases studied, a difference of 32.6 days. I'm rather startled at some of these numbers for delay in delivery, obviously some of the first-borns were very premature. However the absolute numbers indicate that this amount of delay is very rare. As for minor delays (<2 days), I didn't find much. I've fired off a query to my sister. Franamax (talk) 20:39, 4 June 2009 (UTC)[reply]
Oohh, jackpot! "Twin-to-twin delivery time interval:...", W. Stein et al., Acta Obstetricia et Gynecologica, 2008; 87: 346�353. [11] 4,110 "normal" deliveries, mean interval = 13.5 min (SD 17.1); 75.8% within 15 min, 16.4% within 16-30 min, 4.3% within 31-45 min, 1.7% within 46-60 min, 1.8% > 60 min. Now those are statistics! :) If anyone wants to try writing this up, I have some of the fulltexts and we can put up one of those "Refdesk significantly improved an article" thingies! Franamax (talk) 21:05, 4 June 2009 (UTC)[reply]
Those statistics with a median of several days are meaningless - the sample has clearly been chosen to be made up of cases with a large delay (since that is what they were studying). The rest of their results may well be interesting, but the length of the delays isn't since they were specially chosen. (The medians are probably only given in order to describe the sample chosen, not to imply anything about delayed births.) --Tango (talk) 01:54, 5 June 2009 (UTC)[reply]
Franamax, this is outstanding! I added a new "Delivery interval" section to Twin, let me know what you think. The section needs the discussion of the extremely long intervals and could use discussion (beyond the stats) of the very unusual situation where labor begins and then ends, and then a month later begins again. Thanks! Tempshill (talk) 05:47, 5 June 2009 (UTC)[reply]
Looks good, I'll work a little more on those anomalous cases where labour ceases and the delay runs to several days. I also had found this in Google Books (Multiple Pregnancy, Blickstein, Keith & Keith), which gets right into the details. Another successful RefDesk collaboration! :) Franamax (talk) 07:10, 5 June 2009 (UTC)[reply]
Thanks guys!--HoneymaneHeghlu meH QaQ jajvam 00:58, 6 June 2009 (UTC)[reply]
This time gap had puzzled me in the article on the Chukwu octuplets -- one of the babies was born days before the others. I don't get it. Once the amniotic sac has been breached, the rest of the fetuses are left high and dry. (So to speak.) I'm guessing, but the sequence might be: woman goes into premature labour, rushed to hospital, one baby is already in the birth canal, it comes out and is handed over to neonatologist, contraction-inhibiting drugs are immediately administered to the woman, bed rest in hospital, lots of monitoring, try to give the other bun(s) as much more time in the oven as possible. Something along those lines? BrainyBabe (talk) 20:27, 7 June 2009 (UTC)[reply]

helium balloon

If one were to take a balloon that was made of a material that could stretch an infinite amount while maintaining its structure, and filled it with enough helium for it to rise quickly, would it make it into space (assuming temperature also does not effect the structure of the helium compartment)? It is going far slower then the speed needed for orbit, but helium does escape from our atmosphere.65.121.141.34 (talk) 19:58, 4 June 2009 (UTC)[reply]

If we're going to invent materials which cannot possibly exist, why not just cast a magic spell on the balloon and teleport it to space? But seriously, the ballon would rise to a point where its density matched that of the atmosphere. For your theoretical infinitely strecthy balloon, since the volume could be infinitely large, the density could be infinitely small, which means the balloon would keep drifting up. I just have a hard time picturing a helium balloon being infinitely stretchy. --Jayron32.talk.contribs 20:41, 4 June 2009 (UTC)[reply]
As I understand the way you've stated the material, the answer is maybe space, and maybe escape. It's important to remember that escape velocity is relevant only to a specific impulse of thrust. Anything that continues thrusting as it rises need not reach escape velocity as defined at the surface in order to reach escape velocity. Anyway, the balloon matches the pressure of the atmosphere and will continue to rise (and expand) until it reaches an altitude where its density is the also same as the surrounding atmosphere. This may well be higher than 100 km, which is the usual qualification for reaching "space". This further may be high enough for the solar wind to thrust it away from Earth, thus maybe escaping. In the absence of solar wind, though (and possibly in spite of it), the balloon reaches stable equilibrium -- were it to rise, it would be more dense than the surrounding atmosphere and thus sink. Were it to sink, it would be less dense and thus rise. Since it's still in the atmosphere and its movements dictated by the atmosphere, I wouldn't characterize this as an orbit and I wouldn't expect the balloon to be in free fall. — Lomn 20:50, 4 June 2009 (UTC)[reply]
I was going to mention how the balloon material would have a density greater than that of helium, thus we could calculate the weight of the balloon at Earth surface and add in the weight of helium it encloses and make a definite calculation of the equilibrium point where the overall density of the balloon-helium system was equal to that of the atmosphere. But of course, since it's infinitely stretchy, we can fill the balloon with all the helium on Earth before we release it, so yes, the average density approaches that of helium alone. Without some constraints, this really can't be calculated... Franamax (talk) 22:23, 4 June 2009 (UTC)[reply]
And below a critical density, the atmosphere will behave sufficiently like a vacuum or a sparse plasma. Once the balloon is in the ionosphere or thermosphere, buoyancy will be an insignificant force, and thermal interactions will become insignificant (although, a balloon of near-infinite volume might have sufficient number of thermal collisions to break down our cold plasma assumptions...) Nimur (talk) 23:24, 4 June 2009 (UTC)[reply]
You didn't give us enough information to answer your question. Tell us what's the mass of the ballon, what's the mass of the helium inside and what is the tension on the baloon's rubber, and than we may be able to give you a reasonable answer. Dauto (talk) 00:59, 5 June 2009 (UTC)[reply]
The atmosphere has no end, so I guess the simplest answer is that it will continue to rise and expand until interactions with the rest of the solar system become more significant than interactions with the Earth (which basically means it has escaped the Earth). At some point, either bouyancy would cease to be significant or you would reach a point where the atmosphere gives way to the Interplanetary medium, which I believe is mostly (ionised) hydrogen, so is less dense than helium at equal pressure (to the extent that pressure is well defined in such circumstances). However, your assumptions are clearly impossible, so the only entirely accurate answer is "anything could happen". (See Vacuous truth.) --Tango (talk) 01:46, 5 June 2009 (UTC)[reply]
Actually, the atmosphere has an ill-defined end; it gets infinitessimally thinner as you keep going out from the earth. This does NOT mean that it has no end; to assume it never ends is to make the same error that is made in Xeno's paradoxes. (See discussion elsewhere at the ref desk on this). Eventually, the matter density of the atmosphere fades to match the average matter density of the rest of the so-called "empty space" of the rest of the solar system. At that point, we can say that the atmosphere has ended, since it has become indistinguishable from space. That line can be clearly drawn around the earth, so there is no point in saying it "never" ends. The atmosphere is a physical example of a convergent infinite series, and the operative part here is "convergent". The limit where the diminishing atmosphere converges with the rest of space we can say the atmosphere ends. --Jayron32.talk.contribs 03:06, 5 June 2009 (UTC)[reply]
By "has no end" I meant it never becomes a true vacuum, which is what would be required for this idealised balloon to stop rising and float on top of the atmosphere. I talked about the atmosphere giving way to the interplanetary medium, so I think it is quite clear that I understand what is going on... --Tango (talk) 03:10, 5 June 2009 (UTC)[reply]
Well, lets call it a misunderstanding over the use of imprecise language. It would be easy to assume that, when you say it "has no end" that the it continues on forever. It doesn't really, its just that you have to define what you mean by "end". --Jayron32.talk.contribs 03:32, 5 June 2009 (UTC)[reply]

Falling for infinitly long.

Since I was learning about terminal velocity I was wondering that would it be physically possible for an object to have a terminal velocity greater than c. If it were a point object such that A was very small, falling through an low density fluid with a low drag coefficient, and a great enough mass. If this were possible (albeit physically it would take a long time to attain it; imagine it falling through an infintly long tube), would that mean that as v--> c, the mass would go to zero. Such that its K(e) would never exceed E=mc2. Or would the mass remain the same given that E2=(mc2)2+(pc)2. Using the second equation E could increase without bound? I would imagine a vacuum would have a drag coefficient of zero, this would make terminal velocity infinite, although a perfect vaccum is impossible. —Preceding unsigned comment added by 24.171.145.63 (talk) 21:16, 4 June 2009 (UTC)[reply]

The short answer is "no". I think we have an article, relativistic addition of velocities or some such? If not, it should redirect somewhere.
Things are a bit more complicated if you consider the large-scale geometry of the universe -- see observable universe. One way of expressing the edge of the observable universe is that it's where galaxies are moving away from us faster than c. --Trovatore (talk) 21:20, 4 June 2009 (UTC)[reply]
Velocity_addition_formula#Special_Theory_of_Relativity. --Tango (talk) 01:38, 5 June 2009 (UTC)[reply]
Off topic, but the volume beyond which the recession speed exceeds c is called the Hubble volume and is smaller than the visible universe. The recession speed at the edge of the visible universe is around 3.3c. -- BenRG (talk) 17:12, 6 June 2009 (UTC)[reply]
Why would the mass go to zero? Rest mass is always the same, irrespective of velocity. Relativistic mass increases as velocity increases. The total energy of a moving object is the sum of the rest energy, mc2 (where m is the rest mass), and the kinetic energy (for small velocities that's mv2/2, for velocities near the speed of light you need to use relativity). The rest energy is constant, the kinetic energy increases with velocity (and increases without bound, even though the velocity is bounded above by the speed of light). --Tango (talk) 01:38, 5 June 2009 (UTC)[reply]

A structural designation

When listing or describing archictectural structures what does HEW (insert number here) mean e.g. HEW 365, which is for the Gasworks Railway Tunnel around Kings Cross in London? Simply south (talk) 22:20, 4 June 2009 (UTC)[reply]

I think it is the Panel for Historical Engineering Works designation; and think said panel is part of or connected with the Institution of Civil Engineers. See, for instance, the introduction to London and the Thames Valley, by Denis Smith, Institution of Civil Engineers (Great Britain), and then note that the book lists HEW numbers against each structure described. --Tagishsimon (talk) 22:33, 4 June 2009 (UTC)[reply]
See also ICE overview of PHEW and a PHEW database with a crappy user interface --Tagishsimon (talk) 22:36, 4 June 2009 (UTC)[reply]
Thank you for that. Simply south (talk) 23:17, 4 June 2009 (UTC)[reply]

Understand weak interaction and beta decay

I tried long to understand how a force could result in β decay.

As for my knowledge I usually think of a force as a vectors (or a vector field if we speak of fields): electric force between 2 charges as 2 vectors with the same direction, the same for gravitational force, and I think I could do the same for strong force. But when I read of weak force and its role in β decay, first I see only feynman diagrams, which I understand, but I cannot fit them with my idea of vectors. Second I see only one particle interacts with this force, when for all of the other forces there are at least 2 particles, or a field and a particle.

I conclude saying that I think I understand the basic mechanism of β- decay. So, said in really raw words: a neutron emits (on its own or there is a reason?) a W- boson, which "extracts" one unit of negative electric charge from the neutron, and the neutron is transformed in a proton. Then the W- boson decays in an electron and an electron antineutrino.

Could someone help me and clean my doubts?

ColOfAbRiX (talk) 23:56, 4 June 2009 (UTC)[reply]

You said you see only one particle interacting in the beta decay, but when you gave us an example of beta decay you mentioned a neutron, a proton, a W-, an electron and an anti-neutrino. I count five different particles. Even if we exclude the W which plays the role of the "force" here, you still have four interacting particles. Forget about vectors, they won't help you here. You should think of "forces" as interactions between different fields ()The vertices of the Feynman diagrams. Dauto (talk) 01:19, 5 June 2009 (UTC)[reply]


June 5

Distribution of stellar classes

According to the table in Harvard spectral classification, stars of spectral class M comprise over ~76% of all main sequence stars while Sun-like stars in spectral class G comprise ~7.6% of all main sequence stars. This suggests to me that class M stars outnumber class G stars by a ratio of 10:1. My own OR suggests that is roughly true within 20 light years of the Sun, but is that ratio maintained throughout the Milky Way galaxy? Presumably similar ratios can be calculated for other spectral classes as well - I am particularly interested if some parts of the Galaxy are lacking in stars a particular spectral class. Astronaut (talk) 00:10, 5 June 2009 (UTC)[reply]

I think the ratio is maintained on a large scale (presumably universally, although I'm not sure how much evidence there is to support that). I'm not sure about the distribution of different spectral classes throughout the galaxy, but there are certainly differences in the metallicity of stars in different areas (that article explains it fully). --Tango (talk) 01:32, 5 June 2009 (UTC)[reply]
I saw the metallicity article, but I'm not convinced there is a strong link between metallicity and spectral class, and anyway I'm really only interested in stars in the Milky Way disc. The kind of thing I'm trying to find out is if, for example, we observed that the population density of O and B class stars in the spiral arms was typically double that in the regions between the spiral arms, could we assume the same was true of the population density of G, K and M class stars, even though we cannot observe them directly? In other words, can I use the distribution of bright stars that can be observed at great distances, to reliably infer the existance of a much larger population of dim stars at these great distances? Astronaut (talk) 04:39, 5 June 2009 (UTC)[reply]
I don't think you can do that because the O class stars are short lived (less than 10 million years) and won't have enough time to move very far from the star nursery where they are born, while smaller stars can live much longer and will have a more uniforme spacial distribution. Dauto (talk) 06:43, 5 June 2009 (UTC)[reply]
One factor in variation of star population is related to the size of the galactic body they are in. Globular clusters are known to lose stars through a process analogous to (and actually called by some) "evaporation". The lighter stars in the cluster are the most "volatile" since they will gain the highest velocities in energy exchange interactions with other stars. Globular clusters thus tend to have a relative poverty of K and M-type stars since these will be the first to achieve escape velocities. SpinningSpark 13:08, 5 June 2009 (UTC)[reply]
Two important keywords for this type of question are the initial mass function (IMF) and the star formation history, and it's the latter that chiefly determines the current distribution of stellar types in a given stellar population. Starting from the IMF, an assumption about the star formation history and using stellar evolution models, one can try to model the stellar populations in various environments of the Milky Way, but also in other galaxies. Obviously, while in the Milky Way one can make a tally of individual stars, in other galaxies one only has a limited amount of information, like the colours of galaxies or integrated spectra. What a population of stars in a given area looks like at a given point in time thus depends on both the IMF and the star formation history. From modelling this sort of information one can then try to reconstruct these two factors. I'm far from being an expert in this, but my impression is that the IMF is fairly universal. There are indeed different versions that have been tried and which vary in particular at the low mass end, i.e. at late spectral types, but that controversy seems to be due more to our ignorance rather than variation in different environments. The star formation history is much more variable. Early-type stars, O and B, say, are very short-lived and can only exist in environments where stars are currently being formed. If you switch off star formation (e.g. by removing cold gas) these stars will disappear very quickly and you'll be left with a population that is dominated by late-type stars, main-sequence stars dominating by number, and red giants dominating the light output. The neighbourhood of the sun is in the disk of the Milky Way which is still forming stars, hence we might expect some early-type stars (typically forming OB associations). The bulge forms much less stars and is therefore dominated by later-type stars. The extreme cases would be "red and dead" elliptical galaxies. --Wrongfilter (talk) 16:24, 5 June 2009 (UTC)[reply]

Q about lovebirds

Is it true that it's impossible to keep one lovebird as a pet? Is it also true that if you have two lovebirds and one of them dies, then the other one will become depressed, stop eating and die soon after? Thanks. --84.67.12.110 (talk) 00:13, 5 June 2009 (UTC)[reply]

I've looked around and read a bunch of likely pages - and I can't find anyone that says this is true. However, it's clear that they are extremely active, intelligent, social birds and might get depressed because they don't have anything to keep them amused when their mate dies. I suspect that the myth of one bird dying and it's mate dying "of a broken heart" soon after has it's roots in the fact that if you buy two birds of the same age on the same day - keep them in the same cage and feed them the same food - then the odds are pretty good that whatever kills one of them will soon kill the other...especially if it's a bit depressed because it's bored. This would seem a lot like death from a broken heart - but in all likelyhood it's probably just that whatever killed the first one also kills the second. As for keeping just one of them - according to most of the sites I've read, it'll do OK PROVIDING you give it tons and tons of attention and keep it busy and interested in life. Having two of them relieves you of some of this huge commitment in that (to some extent) they'll keep each other amused when you aren't around. SteveBaker (talk) 03:50, 5 June 2009 (UTC)[reply]
This song recounts a field observation of catabythismus incidental to amorous dysfunction by Petroica macrocephala, the Miromiro or New Zealand Tit, a bird of the Petroicidae (Australasian robin) family colloquially known as a tom-tit. Cuddlyable3 (talk) 08:39, 5 June 2009 (UTC)[reply]

largest possible Black Hole

Is there a radius at which the density of a Black Hole is so low that it is no longer a Black Hole? --71.100.6.71 (talk) 02:50, 5 June 2009 (UTC)[reply]

No. You are correct that the density of a black hole (ie. the average density within the event horizon as viewed from a distance) decreases as the black hole grows, but there is no minimum density required to remain a black hole. --Tango (talk) 03:15, 5 June 2009 (UTC)[reply]
Density of Supermassive black holes can be surprisingly low. You will not even notice you entered it. You will just be unable to get out. --131.188.3.20 (talk) 12:51, 7 June 2009 (UTC)[reply]

zinc cloride

why zinc cloride is not weighed while it is still hot? —Preceding unsigned comment added by 115.134.149.71 (talk) 03:51, 5 June 2009 (UTC)[reply]

Nothing should be weighed while its temperature is different from the ambient temperature. If the substance it hotter than the air it will generate convection currents which can easily disrupt a sensitive balance and give an inaccurate reading. If you are going to weigh something, let it cool to room temperature so that such convection currents do not screw with the balance. Of course, you already knew all of this, because you paid attention when your chemistry teacher explained all of this to you. Right? --Jayron32.talk.contribs 04:02, 5 June 2009 (UTC)[reply]

Lame flight data recorders

The flight data recorders presently in use on jumbo jets send out a beacon signal detectable to a reported one mile, but the planes may crash in ocean depths of up to 4 miles, such as the recent Air France jet crash. Would the acoustic beacon power have to increase as the square or the cube to be detectable at 4 times the present distance, so there would be a great likelihood of recovery? This could mean 16 times the radiated acoustic signal or 64 times, respectively. News articles also say the batteries will fail after 30 days of beeping. Why couldn't the flight data recorder have a transponder which sends out a very powerful signal only when a powerful search ship probe signal triggers it, meaning it could still be found long after the crash? Edison (talk) 05:26, 5 June 2009 (UTC)[reply]

I think the deal with FDRs is that they are built to an old standard, and the standard has never been updated, because having one standard is preferable in most cases than having 100 different types of FDRs. If they are all identical, then one always knows what to look for. Actually, the better standard would seem to instead broadcast the information via a digital data stream over a satelite system. The problem is that the FDR is attached to the plane. If the flight data information were stored outside the plane, it would greatly reduce the need to recover the box. The black box could still exist as a backup. Of course, the technology exists to do exactly that; the problem would be retrofitting the worlds airplane fleet and relevent ground locations with the system, it would be a daunting task. But the technology certainly exists to devise a better system. --Jayron32.talk.contribs 05:33, 5 June 2009 (UTC)[reply]
I don't know how much data they record, but surely it's enough to make every plane in the air constantly streaming it over expensive satellite bandwidth impractical. --Sean 14:11, 5 June 2009 (UTC)[reply]
Also, couldn't bad weather (like the recent Air France crash occurred in) interrupt the signal? TastyCakes (talk) 14:25, 5 June 2009 (UTC)[reply]
In an simple isotropic medium it would generally be distance squared, but there is a correction for scattering so it is really more like , where λ is about 1 km in the ocean. Which means that at 4 km you get about 1/300th the power as at 1 km. A further effect is the diffraction around the sofar channel in the ocean, which makes it nearly impossible for deep accoustic signals to ever reach the surface anyway, so even with a more powerful transmitter you'd still have to dangle a deep hydrophone to hear it. Dragons flight (talk) 16:44, 5 June 2009 (UTC)[reply]
Transmitting 4 miles would require more like 2000 times the power as transmitting 1 mile, given the formula above. Dragons flight (talk) 17:04, 5 June 2009 (UTC)[reply]
The Mary Sears was unable to locate the black boxes from Adam Air Flight 574 using hull mounted sonar. After returning to port and equiping a Towed Pinger Locator she detected both at a depth of 1700m. According to Jane's a TPL can detect the signal from a max depth of 20,000 ft.—eric 19:31, 5 June 2009 (UTC)[reply]
According to the Adam Air accident report they were found at 2000 and 1900 meters, and the Mary Sears was required to pass within 500m of a beacon to locate them.—eric 20:08, 5 June 2009 (UTC)[reply]

Radiocarbon and DNA evidence on Voynich Manuscript

Has the Voynich Manuscript been subjected to radiometric dating to determine the age of its materials, or to DNA testing to determine the region of their origin? NeonMerlin 05:41, 5 June 2009 (UTC)[reply]

According to this Yale journal (scroll down to "Shelf Life"), dated February 2009, "two outside specialists are analyzing the pigments in its ink and carbon dating a tiny sample of its vellum." I haven't come across any reports of their findings, though. Deor (talk) 15:02, 5 June 2009 (UTC)[reply]
Is it a coincidence that this question came today? Jørgen (talk) 17:02, 5 June 2009 (UTC)[reply]
My guess is that Merlin's inquiry was sparked after reading Mr. Munroe's comic this morning, and not by chance. 17:07, 5 June 2009 (UTC)

Electrical power usage of computer

Hi, in this green age I guess it's relevant to ask this question. I've always been told to leave my computer on all the time, as repeated on/off switching could not only damage the switch, it causes electrical spikes which could damage the motherboard(?) over time. However, now my latest concern is that leaving it on consumes at least 600W per hour (that is the size of the power supply unit with the fan). Is this true? When the computer goes into power saving mode (turn off monitor and hard disk) - how much less does it use? Would it use even less if I tell the computer to hibernate during times of no usage? Sandman30s (talk) 09:24, 5 June 2009 (UTC)[reply]

While there is certainly some increased wear and tear from turning it on and off, I don't think it is much to worry about. If you're just stepping away for 10 minutes it probably isn't worth turning it off, but if it is going to be several hours it would be good to. It's difficult to say how much power it uses at different times, it varies from computer to computer, but the 600W figure would be a maximum (plus whatever your monitor uses and any other equipment that has its own mains plug), it won't use that all the time. You can get a power meter that goes inbetween the plug and the mains socket that will tell you exactly what it is using, I think most high street electrical stores will have them. Hibernation is actually the same as turning it off completely, the only difference is that it saves its current state to the hard drive so it can load back to the same point when it turn it back on. It will probably still use some power when switched off unless you unplug it or switch it off at the mains (see Wall wart). Leaving it on standby will use a significant amount of power. --Tango (talk) 12:37, 5 June 2009 (UTC)[reply]
Thanks, I think I'll do some tests with a power meter. Sandman30s (talk) 13:27, 5 June 2009 (UTC)[reply]
Also, note that in the winter, if you heat your house with electrical heating, you can just leave your computer on as all "wasted" energy contributes to heating your house anyway. (and conversely, if you use AC in the summer, you waste "double" the energy by leaving the computer on (I'd think?)) Jørgen (talk) 13:54, 5 June 2009 (UTC)[reply]
Both of your estimates are accurate - in data centers, it is estimated that total electric bills are equal to 2 times the power consumption of the electronics - this is a very common rule of thumb for estimating costs. This means that for each watt of useful electronics, an extra watt is necessary for air conditioners. It's hard to grasp this heating/air-conditioning concept intuitively - you "know" your computer is warm, but it's not that big... If you have the chance to visit a server-room, get permission to walk BEHIND one of the rack cabinets - behind a rack of 40 high-performance systems - you'll be blasted by the hot-air fan output which might reach a hundred kilowatts - it's like a furnace! Nimur (talk) 14:53, 5 June 2009 (UTC)[reply]
Thanks for the info! This should mean that environmentally friendly data centers should be placed in the basements of large offices / residential buildings in cold areas, so that the excess heat can be used to heat the building. I think I've read some articles on large data processing centers being located in places where hydro power is prevalent, or where there is a river available for cooling, both of which would be fine, but I think that placing them in cold areas - pumping cold air in (in some controlled process, of course) to cool the components and using the excess heat for heating - would be an even better approach? Now, I know the logistics of this might be hard, but I'm sort of surprised that no fringe-targeted "environmentally friendly cluster" has marketed this yet. (Oh, and sorry for hijacking the question) Jørgen (talk) 15:08, 5 June 2009 (UTC)[reply]
The use of cold, outside air as a source of cooling for data centers is already an accepted solution in some areas. Care must be taken that outside air does not introduce fine particulates that might damage equipment, and the temperature and humidity of incoming air must be tuned; these challenges and concerns have delayed the adoption of so-called air-side economizers or outside air economizers. Nevertheless, some more recent studies (by Intel and some electrical utilities; linked from our articles) have allayed fears over these risks. Here's an article about a Canadian firm that gets about 65% of their data center cooling from outside air: [12]. TenOfAllTrades(talk) 16:51, 5 June 2009 (UTC)[reply]
Thanks! Good to see that good ideas are put to use. Jørgen (talk) 17:00, 5 June 2009 (UTC)[reply]
In this interview with an IT manager working at the South Pole, he says that at their old data center somebody had just cut a hole in the wall to cool everything inside. Tempshill (talk) 17:33, 5 June 2009 (UTC)[reply]
Actually that's only likely the case if you have central heating and heat your house 24/7 to a constant temperature. In fact even then I'm not convinced it's going to be equal unless every space and passive part in your house is at a constant temperature which I would expect unlikely. Also if you are using a heat pump then even if it is really the case, you're still going to usually spend more energy then you would have otherwise because of the better coefficient of performance for the heat generated (may be a bit confusing in the context because not all of the energy input is going to make heat but even that which is, can basically be considered akin to an electrical heater). At least though computers are usually on the ground, not near the ceiling like those who try to argue about lights. Also it is unlikely all energy input ends up as heat, even when idle. Computers do do processing etc and some of the wasted energy would likely be in sound, perhaps em etc which is not going to end up as heat (at least not in your house). If you are using a modern computer with Cool'n'quiet, Speedstep or a similar power saving technology and an efficient PSU, I would say you would still often use at least 100W when idle and even presuming 80% ends up as heat (which I expect is a bit high), and all that heat is useful for you (which as I've explained I think is unlikely) that's still 20W. Over 90 days that's 43.2kWh. That's easily a few dollars/pounds/Euros/whatevers (depending on energy costs in your area) and equivalent to leaving a 1000W heater on in the middle of nowhere for 43.2 hours for no purpose. Of course in practice your computer is not going to be off 24/7 either but it's entirely resonable to turn off your computer if you're not going to need it for an hour or more. If you need a fast start, you could always enable some standy mode. Ultimately, it's always to let specialised equipment do what they were designed to do. Don't use a computer or a light as your heater (well heat lights are designed to generate heat and light but that's a different issue altogether). Use a heater. Yes it does mean the wasted energy calculation is not as simple as it seems but it doesn't mean you shouldn't consider wasted energy because it's all just heat and you want heat. The heat has to be in the right place and the right time otherwise it's still wasted. Nil Einne (talk) 07:54, 7 June 2009 (UTC)[reply]
As I mentioned above, it makes sense to turn off your computer but it's highly unlikely your computer uses 600W even at full load. Firstly power supplies for computers are always more then the computer ever needs. If it's a professional designed computer, e.g. from Dell etc, then the power supply maximum output would be closer to the maximum expected requirement but still higher. If it's a noname or assembled by a friend then there's a good chance the power supply is way more then the computer ever needs (this isn't a good thing but it's common). Commonly people add up the maximum possible draw of every component in the computer, except that this situation usually never actually occurs in practice. Secondly most cheap, no name power supplies are rated much higher then they can actually reliably supply. In other words, even if you power supply is rated 600W, it will often not be able to reliably draw close to that before having major problems. Thirdly as I have already hinted at, this is as maximum load. If you have a modern CPU and a modern GPU and even to some extent a modern motherboard, these are designed so they will draw a lot less when idle or close to idle (old ones to but not to such an extent). With CPUs, make sure the powersaving technology (probably SpeedStep for Intel and Cool'n'quiet for AMD) is probably enabled and working. With GPUs try and get an updated driver from the manufacturer (Nvidia or AMD/ATI) and look for any powersaving options. Note that if you have an IGP these don't use much but there may sitll be some power saving options. If everything is set up right, I would expect even for quite a good computer, you can expect it to use about 100W or so when idle (a very rough estimate) and 200-300W at maximum load. Even with something like SLI/Crossfire and quad core 500W would be a high maximum. In other words, the power used is likely less then you may think (but it doesn't mean it makes sense to waste it). As has already been mentioned if you're really interested, you can buy a power meter, and plug your computer into this and see how much it draws. (This is the only reliable way to work out how much a computer uses, anyone who tries to estimate it by adding up components is coming up with a highly speculatory number, and if looking at reviews, I would ignore such figures and may be even the whole review.) The only other thing I can say is you may want to consider a new PSU. As I've stated, it's likely more then you need. If it's some no name POS (as I've suggested if it's a decent branded computer, e.g. Dell, HP-Compaq etc then the PSU is probably an okay one), this is highly problematic because many crappy PSUs are extremely inefficient particularly at low loads which is probably the case for you presuming your computer is idle most of the time (browsing the internet, running Office is usually close to idle). A high quality good brand PSU, which meets the 80 Plus Certification would likely save quite a bit of energy. Plus cheap POS PSUs are notorious for dying and taking some of the components with them so it may save you some headaches there too. It may cost a bit, but I would look into whether you really need a 600W PSU. As I've said, ignore anyone who tells you to add up the maximum theoretical load of each component. A good place to start would be this SilentPC article (sadly a bit outdated) and the forums. Obviously if you get a power meter, then you will have a rather accurate idea of how much your PCs draws and while you'd want some headroom, I wouldn't give it too much. This is one of the examples where bigger is not necessarily better. (This isn't perhaps quite as much an issue nowadays as it was before the 80 Plus days but it's still the case that you don't really want to be only using a tiny percentage of your PSUs maximum most o
Leaving your computer on while you don't even use it 75% of the time probably won't save energy compared to turning it off. Computers usually have a program where you can shut it off either without pressing the power button, or it will say that it's safe to turn off your computer. I even turn off my computer straight with the power button every time, and with no ill effects. ~AH1(TCU) 19:48, 7 June 2009 (UTC)[reply]
Also, some computers will consume several watts of power while they are turned off. ~AH1(TCU) 19:49, 7 June 2009 (UTC)[reply]

Satellite photos of London from today (5th June 2009)

Hi,

Is there any way of getting these? I realise live satellite feeds probably aren't available but what about very quickly archived pictures? --Rixxin (talk) 11:26, 5 June 2009 (UTC)[reply]

Photos from weather satellites, maybe. What kind of resolution are you looking for? --Tango (talk) 13:28, 5 June 2009 (UTC)[reply]
Honestly? This is sure gonna sound lame. I was interested to see if there was a big 'ol dark cloud over The Palace of Westminster today, 'cos it sure felt like there should be!--Rixxin (talk) 16:43, 5 June 2009 (UTC)[reply]
Well, there was a big dark cloud over the whole of England, pretty much... See here for visible light satellite images of London and the South East of England over today (one per hour). I can't find a close up of Westminster, but the weather was pretty consistent so it doesn't really matter. --Tango (talk) 18:51, 5 June 2009 (UTC)[reply]
The day will undoubtedly come when we will be able to view any part of the earth in real time, and zoom in on places of interest. Wars could be watched as they took place. Imagine what it would have been like to watch the D-Day invasion in real time.. – GlowWorm. —Preceding unsigned comment added by 98.21.107.157 (talk) 18:44, 5 June 2009 (UTC)[reply]
Actually, you could have watched it in real-time, with the right equipment. --98.217.14.211 (talk) 19:57, 5 June 2009 (UTC)[reply]
It's very unlikely that such a thing would ever happen, however such a nice idea it may be. Think about it. If satellite pictures of battles were freely available on the internet as the battles were taking place, they would be available to both sides and would present serious problems (and of course, advantages) to both sides. --KageTora - (영호 (影虎)) (talk) 19:11, 5 June 2009 (UTC)[reply]
You would need millions of satellites, and much more bandwith the entire Internet has today.. --131.188.3.20 (talk) 22:34, 5 June 2009 (UTC)[reply]
Half a century ago it would have sounded ridiculous that everyone was walking around with their own personal mobile phone. You could not build enough transmitters, and where would all the bandwidth come from? Technology will continue to improve apace and if you cannot think of a fundamental theoretical reason to stop such things happening, then they probably will if people want them to. SpinningSpark 00:04, 6 June 2009 (UTC)[reply]
Indeed. If you are willing to wait 50 years, almost anything is possible unless it is explicitly impossible (and maybe even then!). I believe three satellites in geostationary orbit with cameras with amazing resolution could cover most of the Earth, all it needs is for someone to invent the cameras and that could easy happen in the next 50 years. --Tango (talk) 00:08, 6 June 2009 (UTC)[reply]
It's not the satellites or the cameras - providing whole-earth, realtime coverage (as we've discussed recently) requires insane amounts of hardware - but it's not disallowed by the laws of physics. HOWEVER - what is disallowed is the bandwidth you need to get all of that data back down to earth. 500 million square kilometers - 500 trillion square meters. 50 quadrillion pixels. About 500 quadrillion bits with lossy data compression. For a 'realtime' update - you need to send that between maybe 10 and 60 times a second. So 5 quintillion bits per second. That's a truly outrageous amount of data - it would fill a million hard disk drives every second. It's more data than is available on the whole of the internet...every second. This is not merely an engineering difficulty - it's a serious "fundamental laws of physics" kind of a problem. If we had a million satellites - all with tight-beam laser links to the ground so they didn't interfere with each other - then it would be pretty much impossible - but to do it with three satellites! That's just impossible. SteveBaker (talk) 02:52, 6 June 2009 (UTC)[reply]
a) What law of physics are you suggesting is being violated? b) you're making the same mistake you made last time - there is no need to transmit all the data, just the data that someone wants to receive and c) you are making meaningless comparisons - what do hard drives have to do with anything? No-one is asking you to store the data. Really, Steve, you should know better than to cavalierly claim something is physically impossible. --Tango (talk) 12:29, 6 June 2009 (UTC)[reply]
Nanotechnology could be a way to do it. Millions of tiny satellites could be put into orbit. Each satellite would consist of a group of molecules forming a camera and a radio transmitter having a broad beam, Each satellite would be solar powered, with a rechargeable battery for nights. Each camera would be aimed at a certain place on the earth and thus would need only a narrow transmitting bandwidth. The camera would be kept in its earth-aim by viewing certain stars and automatically making aim correction using solar power. The viewer on earth would select the radio frequency of the satellite aimed at the part of the world he is interested in. He would do this by selecting a square on a grid map on his monitor. He could enlarge the picture received, or combine the adjacent pictures of several satellites to cover a larger area. The satellites would not have to be in geocentric orbits, thus preventing overcrowding of that orbit, and permitting good near-polar coverage by using polar orbits with some satellites. Automatic switching would transfer reception from satellite to satellite to maintain the selected view of the earth as the satellites move. Automatic viewing-angle correction would also take place to provide an overhead view or angle view. The satellites could be manufactured cheaply by the millions in a nanofactory. They would be so small and light that they could be put into orbit and replaced at low cost. – GlowWorm.
Steve is being a bit negative here. Updates 60 times a second are not necessary for still pictures to be considered real-time, that's only necessary if you want video. One a minute might do, which would take three or four orders of magnitude off Steve's figures. Even if the aggregate data rate really is ~1018 b/s, each individual user does not want all of that at the same time on one channel, nor is each individual satellite supplying all of that on one channel. This is what networks are for, they are composed of many channels. As for the number of hard drives required to store the data, well you only need them for the data you want to store, you don't have to store any of it if you don't want to. SpinningSpark 12:27, 6 June 2009 (UTC)[reply]
Hm, what about not archiving everything, just send the data on demand. For example, for a city of 1000 you don't need 499500 phone cables just to connect them with each other, because they will never use the system all at the same time, talking to everyone at the same time. In fact, around 30 or so lines are enough to assure a 99.9% service. Not every person on Earth will look at the video at the same time, and even if they did, they won't see the whole surface of the Earth at the same time. Even so, we will require a ridiculous amount of satellites, bandwidth and storage space by today's standards, but significantly less than the numbers discussed so far. --131.188.3.21 (talk) 15:02, 6 June 2009 (UTC)[reply]
The MODIS Rapid Response website provides "near real time" (perhaps a pass per day or so) satellite coverage of most of the planet. The website is here. It takes some work to find what you want, and odds are good it will be cloudy. The images are fairly raw with edge distortion and the like, and the maximum resolution is 250 m per pixel. Still, the OP didn't specify the resolution required, and these are near daily images, so... Pfly (talk) 08:51, 7 June 2009 (UTC)[reply]
Yes - but that's hardly what the OP is asking for. Photos of "London" would be little more than a grey blur with a pixellated river running through it. 250m resolution means that streets and buildings cannot be seen - perhaps the larger parks could be - but there would be very little point in updating those pictures that frequently since nothing much changes at that scale from day to day - or even month to month!
This is London at 25meters per pixel:

But this is all you see at 250meters per pixel:
Pretty much a grey/green blur with a suggestion of a river.
SteveBaker (talk) 18:01, 7 June 2009 (UTC)[reply]
Also, the USGS Global Visualization Viewer gets you images from different satellites near specific geographical coordinates. However, it's unlikely to allow you to see images in much resolution, although it does usually have recent (few months to weeks) images, depending on cloud cover. ~AH1(TCU) 19:42, 7 June 2009 (UTC)[reply]

Is it really true that Planes which cruise at high altitude get hit by lightning?

Hi All,

I just keep wondering if the latest French Aeroplane's accident is really due to lightning. As I understand, at the hights of over 28k feet, there are no rainbearing clouds, in fact (again within my limited knowledge)these clouds are much below. As such there shouldn't be lightning too. Am I correct in my understanding? Would appreciate if people could enlighten me on the same.

Warm Regards, Rainlover_9 Rainlover 9 (talk) 14:22, 5 June 2009 (UTC)[reply]

See upper-atmospheric lightning. Gandalf61 (talk) 14:26, 5 June 2009 (UTC)[reply]
Planes DO get hit by lightning - but not "Upper Atmospheric Lightning)!! Upper-atmospheric lightning is WAY above where a plane flies. A plane will fly at about 35,000 feet - only about 10 km above the ground. The phenomena referenced in upper-atmospheric lightning are more appropriately called "Transient Luminous Events" because they are neither lightning, nor do they take place in the "atmosphere" (largely being above the stratosphere and in the lower boundary layers of the ionosphere). There is no way even a military aircraft would fly at those altitudes (80 or 100 km above the ground). Airplanes can and do get hit by tropospheric lightning when they fly close to or through a storm - and some cumulonimbus clouds might have storm tops as high as 60,000 feet above ground. It is this type of lightning - typically a cloud-to-cloud strike - that results in the occasional airplane strike. Nimur (talk) 14:59, 5 June 2009 (UTC)[reply]
I have seen a photo of lightning striking up from a cloud as well, so that could be a possibility. Also, an airplane flying along might build up its own static charge, and may become a lightning target by itself, similar to how lightning will go from cloud to cloud. 65.121.141.34 (talk) 15:03, 5 June 2009 (UTC)[reply]
As I understand it, lightning has been ruled out as a cause. Planes, in theory, should be safe from lightning for the same reason cars are safe from lightning: the metal skin conducts the charge safely around the passengers and sensitive electronics. This is not always the case, as I know lightning has brought down planes in the past, but I'm fairly certain that all modern planes have protection against lightning. (ref: http://blog.seattlepi.com/aerospace/archives/169983.asp) -RunningOnBrains(talk page) 18:15, 6 June 2009 (UTC)[reply]
I don't think lightning has been ruled out entirely, but it is pretty clear that more than one thing went wrong. These kinds of accidents always have a sequence of causes. There are all kinds of safety features on modern places so they can survive pretty much anything that can happen, but sometimes they can't survive multiple such things happening at once. --Tango (talk) 18:33, 6 June 2009 (UTC)[reply]
The deadliest plane crash caused by lightning was LANSA Flight 508, with 91 deaths. Planes today have better safety features, but that does not completely prevent them from being downed by lightning. Some features include needle-like structures on the back of the wings and tail to repel the static charge away from the aircraft. ~AH1(TCU) 19:38, 7 June 2009 (UTC)[reply]

Is 2 + 2 = 4 in other universes

Hello, while wandering about on the Internet I came across this interesting question on Yahoo Answers:

"Is 2 + 2 = 4 in other universes? Assuming that parallel universes exist, would their mathematics be the same as ours? I.e. could there be a universe in which, for example, 2 + 2 = 5?"

People may be interested in the answers posted: http://au.answers.yahoo.com/question/index;_ylt=AvNVaIojr7pEw1Bn_IyyzxoJ5wt.;_ylv=3?qid=20090603213159AAKkQK1

Since the theory of multiple universes was derived using the maths that we understand, then it follows that the maths on parallel universes MUST be the same. Otherwise, it would make the theory inconsistent. That's my guess, I was just wondering what others might think about this.

203.206.250.131 (talk) 14:29, 5 June 2009 (UTC)[reply]

Unless they use different numbers, then it's probaly going to be the same. If you have 2 things and add 2 more it's going to be 4 everytime. --Abce2|AccessDenied 14:31, 5 June 2009 (UTC)[reply]

Actually I would disagree and say there definitely could be universes where 2+2=5 or even 2=3. The whole idea behind a theorectical multiverse is there are an infinite number of universes in which the laws of physics, logic and math can be different. Time could run backwards, accelerate, or not exist at all. Keep in mind math has never been proven to be (maybe once we get a grand unification theory it will be)the basis of reality but simply a representation which we've created. But either way, even with a G.U.T. this would only prove math is immutable in OUR universe, others could be entirely different. TheFutureAwaits (talk) 14:56, 5 June 2009 (UTC)[reply]

A better way to phrase this question is, "Suppose a hypothetical universe existed where 2+2 != 4. What would be the consequences?" I do not think there are any verifiable consequences of such a statement, so it is a non-scientific question. I think I'll leave my response at that, and we can just wait for SteveBaker to show up and remind everyone that needless philosophizing is wasting quarks... Nimur (talk) 15:03, 5 June 2009 (UTC)[reply]
(ec)Personally i think maths evolved from what you see. If you have two balls and someone else gives you two more balls, you find out that you have four balls. This process is called addition. So, maybe, just maybe, there might be a universe where if you have two balls already and i give you two more, another one automatically materializes and you get 5. This is a physical law in the universe that whenever you give two more balls, an additional one should materialize. In this particular universe, i am inclined to believe that 2+2 will indeed be 5, and hence all our mathematics breaks down. So, if there can be universes with different physical laws, this might well be possible, so i think it cannot be said with certainty that 2+2 is always 4. It can just as well be any other number, though it is hard to visualize in this example how it can be a fraction(half a ball? pi times a ball?) I am assuming decimal systems used in all universes. Rkr1991 (talk) 15:09, 5 June 2009 (UTC)[reply]
It depends on your definitions of 2, +, = and 4. If you define them the same way as we define them, which is not dependant on the universe, then you'll get the same answers. However, life in other universes may well define them differently if different definitions are useful for them. Mathematics and universes are separate things linked by models. We model parts of the universe on mathematical things. To use Rkr1991's example, balls are well modelled in our universe by natural numbers, they might not be well modelled by natural numbers in other universes. That doesn't mean natural numbers are any different, just that they wouldn't be used. There are all kinds of possible number systems in mathematics but we generally only use the ones which are useful for describing our universe. --Tango (talk) 15:20, 5 June 2009 (UTC)[reply]
I disagree with Tango. Even if you define 2, +, = and 4 exactly the same way in our system, the problem posed in my example remains unresolved. As i said earlier, i think maths was derived from what we observe, not hard solid facts. Maths and our universe cannot be separated, and it cannot exist independently. So if we see 2 balls and 2 balls make 5 balls in a universe, then i think our maths indeed breaks down there. Rkr1991 (talk) 15:38, 5 June 2009 (UTC)[reply]
I agree with Tango. If we see 2 balls and 2 balls make 5 balls, we will reasonably deduce that our definitions of 2, +, = and 4 are no longer useful, and we will redefine one of them. But unless we do that, if instead we keep 2 defined (as is most common) as the set {{}, {{}}} and + through Peano arithmetic, 2 + 2 cannot suddenly become 5. It's not like mathematics defines 2 + 2 as "the number of physical objects in the collection resulting from putting two physical objects together with two other physical objects". Correspondence with that physical result is of course the reason for our definitions (that's why Peano chose the axioms he did, etc), but it is not the definition. —JAOTC 15:54, 5 June 2009 (UTC)[reply]
Our definitions of arithmetic are axiomatic, not empirical. Observations don't come into it other than to determine whether those definitions are useful. --Tango (talk) 16:29, 5 June 2009 (UTC)[reply]
Um, no. The formal axioms of arithmetic are chosen because they're true, not the other way around. How we come to know this truth, and whether that can be called "empirical", is a difficult question. But you can't seriously maintain that there's no notion of the truth of a statement like 2+2=4 apart from its provability in a formal system, when discussing provability in that system requires understanding the truth or falsity of more complicated statements. --Trovatore (talk) 16:48, 6 June 2009 (UTC)[reply]
We use those particular axioms for things like counting everyday items because they are useful axioms for doing that - they accurately model the real world (you can call that "truth" if you like, I prefer to avoid the word because it has so many different meanings). We could still consider the axioms in some universe where they didn't accurately model the real world, but it would be a largely academic exercise. If we chose to carry out that exercise, though, we would still get 2+2=4. --Tango (talk) 03:07, 7 June 2009 (UTC)[reply]
We use those axioms because they're true. They accurately describe the behavior of the mathematical objects they are intended to describe, which are part of the "real world", yes, but not the physical world.
The claim in your penultimate sentence relies essentially on the existence of real, though abstract, mathematical objects, which are the same in the hypothesized other universe. If you don't have those objects, you have no way of even formulating what it would mean to "carry out that exercise" in that other universe. --Trovatore (talk) 06:27, 7 June 2009 (UTC)[reply]
It occurs to me that I should say a little more here, to head off a possible misunderstanding of my point. My point is that the axioms themselves are abstract mathematical objects, and the syntactic manipulations of them by which one codes proofs are more complicated relationships between those abstract objects, than the relationship between the abstract objects 2 and 4.
If you don't accept abstract objects of even this simplicity, then you have no grounds whatsoever to claim that it's even meaningful to discuss "carrying out the exercise" in a hypothetical universe in which the physical objects may bear no relationship whatsoever to those we're used to. Hope that makes my point clearer, or at least not less clear. --Trovatore (talk) 06:32, 7 June 2009 (UTC)[reply]
I'm afraid you have a misunderstanding of how mathematical logic works. There is no single mathematical object called "2". Anything which satisfies the axioms for being "2" can be consider to be "2". The axioms come first, then you find a model which satisfies those axioms. The most common model for the natural numbers is to let zero be the empty set () and then the successor of each number is the set containing all the numbers that come before it, so 2 would be "", but there are various other models you could use all of which have an object that can legitimately be called "2". The axioms were chosen because they accurately model the way items in the real world behave (there is an obvious real world concept of there being none of something, so we need a zero, then you can always place another item in a group and get a new number of items, so every number needs a successor, etc. See Peano axioms for details). --Tango (talk) 16:11, 7 June 2009 (UTC)[reply]
My Ph.D. is in mathematical logic, which of course does not in itself refute your claim that I misunderstand it (in fact, I would even agree that it is practically guaranteed that I misunderstand it), but at least recognize that such misunderstandings as I have are not ones uninformed by knowledge of the currently-understood alternatives.
There are two separate points to be addressed here — structuralism versus unique representation on one axis, and axiomatics versus realism on the other. The bit about "no single mathematical object called 2" addresses only the structuralism-related part, and that part I'm not interested in arguing. Sure, any object could be thought of as "2", if it bears the correct relationship to other objects thought of as other naturals. But that is not the same as saying the naturals are characterized by axioms, only that they are characterized by the relationship to other objects. These cannot be captured by (first-order) axioms.
The claim about axiomatics being primary in arithmetic, though, is simply false. Just plain not true. This is easily seen — the ancients had no axioms for the naturals at all, but they still knew what they were. --Trovatore (talk) 20:58, 7 June 2009 (UTC)[reply]
(e/c) Math was originally derived from experimentation, but this is no longer the case. A mathematical theory is made by a group of axioms and is supposed to be interesting. People will find parallels between the mathematical theory and the universe, and use that to make scientific theories. You could have a universe where the strength of gravity is not inversely proportional to the square of the distance, but that just means gravity will be modeled with different mathematics. All that being said, there could be universes where it's possible to build a hypercomputer, and it would be possible to do more sophisticated math. It's not that they'd get different theorems for the same theory (2+2=4 would still be true), it's that they'd be able to use axiom schemas that we cannot. For example, they might be able to find out if every even number above four is the sum of two odd primes by checking every case, something physically impossible in our universe. — DanielLC 16:03, 5 June 2009 (UTC)[reply]
I would like to point out that there are things that appear axiomatic that are in fact dependent on unstated assumptions. Euclid teaches, and people agreed for quite some time, that the interior angles of a triangle always add up to 180º. That's been a basic theorem of geometry for hundreds of years, and is as easy to demonstrate as 2+2=4. Of course, unstated is that only works if you are talking about a triangle on a 2-D plane—if you are sketching the triangle on a 3-D surface, it isn't true at all. A triangle on a sphere can have interior angles that add up to more than 180º. There's an unstated limiting factor in Euclidean geometry—that it only applies to 2-D surfaces—that becomes clear if you actually start, say, drawing large triangles on the ground (and you happen to live on a sphere).
I don't know enough about number theory to say whether there could be something analogous with arithmetic, but it's worth considering. --98.217.14.211 (talk) 17:21, 5 June 2009 (UTC)[reply]
No good mathematician would leave those assumptions unstated, at least in formal work. Everything in mathematics is based on assumptions. Results are always of the form "A implies B" never simply "B". One of the key things that is drilled into any mathematics student, however, is to always state your assumptions. --Tango (talk) 18:01, 5 June 2009 (UTC)[reply]
If 2+2 *always* equalled 5, then the alternate universe's laws of mathematics would likely look a lot like ours (though perhaps a bit more cumbersome). However, consider a universe in which 2+2 sometimes was 4, sometimes 3, sometimes 5, etc. In a sense, we live in such a universe now. The math we use to deal with this is statistics. Two random variables, when added together, can give you different answers each time you observe them. Wikiant (talk) 18:06, 5 June 2009 (UTC)[reply]
In which case they wouldn't model items as numbers but rather as random variables. Mathematics includes all kinds of tools, you need to use the right one for the right job. Just because one tool isn't right for a particular job doesn't mean that it is wrong mathematically. --Tango (talk) 18:10, 5 June 2009 (UTC)[reply]
I would like to leap to the defence of Euclid on this one. Euclid realised perfectly well that the angles in a triangle summing to 180º was not axiomatic but was dependant on the parallel postulate and clearly states this as a postulate in his Elements (postulate 5). It was later mathematicians who through the centuries refused to believe that the parallel postulate could not somehow be proved from other basic postulates. It was only finally admitted that Euclid was right when Einstein discovered that the universe was not, in fact, Euclidean and that other geometries were possible in the real world as well as in the minds of mathematicians. SpinningSpark 23:06, 5 June 2009 (UTC)[reply]
Einstein used non-Euclidean geometry, he didn't invent it. The geometry Einstein used was mostly constructed by Bernhard Riemann, I believe, although several people had done work on the subject before him. Non-Euclidean geometry was known to be relevant to the real world before Einstein came on the scene - we live on the surface of a sphere, which in non-Euclidean. It doesn't matter anyway, you don't need a real world application to prove that the parallel postulate is independent of the preceding four. --Tango (talk) 12:08, 6 June 2009 (UTC)[reply]
I didn't say Einstein invented non-Euclidean geometry, I quite agree, it was around long before that. What his work did do was put an end to the cottage industry of trying to prove the parallel postulate, which at one time occupied the place that attempts to invent a perpetual motion machine occupy now. Very little of the mathematics of relativity are due to Einstein himself, but relativity established once and for all that as well as being unprovable, the parallel postulate was not even true in the real universe (at least on a large scale). SpinningSpark 14:21, 6 June 2009 (UTC)[reply]
The discovery of non-Euclidean geometries put an end to attempts (by legitimate mathematicians) to prove the 5th postulate from the other 4. The real world has nothing to do with mathematics. --Tango (talk) 14:28, 6 June 2009 (UTC)[reply]
Ooooh, well that would depend when you date the discovery of non-euclidean geometries. Some would credit it to Omar Khayyam, in which case do the attempted proofs of Giordano Vitale (1680), Girolamo Saccheri (1773) and Adrien-Marie Legendre (1794) render them not legitimate mathematicians? I think not, and if you define the beginning of non-Euclidean geometry as the point at which attempts to prove the 5th postulate ceased, well then you have a circular argument. SpinningSpark 15:12, 6 June 2009 (UTC)[reply]
I believe Khayyam contributed some of the initial ideas, but he didn't have a rigorous model which obeyed the first four axioms but not the fifth, which is what was required to prove the fifth was independent. I've not studied the history of non-Euclidean geometry in depth, so I'm not sure exactly who had the first such rigorous proof, but I do know if was a long time before Einstein. --Tango (talk) 15:55, 6 June 2009 (UTC)[reply]
We like to think of these discoveries as a sudden revelation and credit them to some one genius in particular. The reality is that it is a gradual process to which many contribute. As Wittgenstein said, the real change in beliefs comes not at the point of this or that discovery but at the point of a paridgm shift. It is made clear in this book for instance, that Saccheri would have been credited with the discovery of non-Euclidean geometry if only he had not continued to publish "proofs" of the 5th postulate. This is an untenable logical position, he discovered non-Euclidean geometry, according to the author, but his discovery is invalidated by his later work. If all his later work was destroyed so we did not know of it, he would be the discoverer, so if I mentally imagine I have not read of his later works he becomes the discover? In any case, as I have already said, I do not dispute this all happened long before Einstein, my point is that it was Einstein that triggered the paradigm shift in belief, and this was because Einsteins work related to the real universe, which is where reality comes in to it. This makes no difference at all to the mathematics, but it does change quite radically the way people are thinking. SpinningSpark 18:36, 6 June 2009 (UTC)[reply]
Yes, there is some confusion over exactly when the first rigorous proof was accepted by the mathematical community, but it was certainly a long time before Einstein. Einstein's discovery that Riemannian geometry was a good model for the universe has nothing to do with mathematics. Mathematicians make decisions based on logic, not empiricism. And, as I've already said, there were aspects of real life that have been modelled on non-Euclidean geometry before Einstein, such as the surface of the Earth. --Tango (talk) 01:19, 7 June 2009 (UTC)[reply]
Are we really debating this? If I have two apples on a table, and place two more apples on the same table, I now have four apples on the table. If I lived in a universe where an extra apple materialized merely because I placed two groups of two next to each other, such a universe would violate the principle of causality so as to be entirely unexplainable at any level. Seriously, why is this even being debated to this level? --Jayron32.talk.contribs 19:00, 5 June 2009 (UTC)[reply]
Your example only violates causality if it is true that 2+2=4. So, your argument boils down to the circular "2+2 must equal 4 because if it didn't, 2+2 wouldn't equal 4." For a possible counter-example, consider this universe wherein subatomic particles and anti-particles spontaneously emerge. This could be interpreted to suggest that 0+0=2. Wikiant (talk) 21:05, 5 June 2009 (UTC)[reply]
Why should another universe obey our universe's laws of causality? It isn't difficult to imagine a universe with two timelike dimensions, which would have very different principles of causality. (I think I read a paper once about such a universe and the conclusion was that complex life almost certainly couldn't evolve in it, but that's not important.) --Tango (talk) 21:15, 5 June 2009 (UTC)[reply]
This reminds me of David Deutsch's concept of "cantgotu" environments in The Fabric of Reality, but I can't remember what if anything he said about the idea of 2+2 not equaling 4. He deals with this general kind of idea, though. 213.122.1.200 (talk) 22:01, 5 June 2009 (UTC)[reply]
No, no, no. You can't use things popping into existance or whatever to envisage a scenario when 2+2 doesn't equal 4. In our universe, 0.9 times the speed of light plus 0.9 times the speed of light equals 1.8 times the speed of light (yes, it really does). However, if you're travelling at 0.9 times the speed of light relative to me and you toss a ball out in front of you at 0.9 times the speed of light, it won't be moving at 1.8 times the speed of light relative to me - it'll be more like 0.99. That's not because 0.9+0.9 doesn't equal 1.8 - it's because the arithmetic operator '+' doesn't apply to speeds as we humans always thought it did. This really shouldn't be a surprise, the '+' operator doesn't apply to a while lot of things!
So if two subatomic particles pop out of nowhere, that doesn't prove that 0+0=2 - it proves that the total number of particles in some volume of space is not a constant - so addition is (again) not an appropriate operation to apply to them.
The point is that things like '+', '2', '=' and '4' are symbols that stand for concepts that have definitions that humans have applied to them. Our definition says that 2+2=4 - not because that necessarily represents something "real" but because that's what we've defined those symbols to mean.
Suppose I were to define the symbol '#' to mean "the sum of two numbers - except when there is a full moon, when it's the product of two numbers" - then 2#2=4 happens to be a true statement...but 3#3=6 isn't always true and 7#9=5 is never true. There isn't any deep inner meaning in what I just said - it's just words and definitions. There is probably no physical property of our universe for which the '#' operator is applicable - but my definition is still a perfectly reasonable one and 2#2=4 no matter what. Perhaps in another universe, placing apples on tables follows the '#' operator - but the '+' operator is kinda useless and considered weird to the people who live there, (who are constantly checking their almanacs to see when the next full moon will be!)
So this universe does whatever it does - the other universe does whatever it does - and whether our definitions for the symbols '2', '+', '=' and '4' are applicable to apples placed on tables in that other universe (or to the way speeds are combined) is a matter that we can't answer. But our definitions still apply - you can't invalidate something that's axiomatic in the system of mathematics you choose to use.
Hence, the answer is a clear "yes" - 2+2=4 everywhere - because we happen to have defined it that way. SteveBaker (talk) 22:27, 5 June 2009 (UTC)[reply]
I'm going to expand upon what's been said above, because hopefully it'll sort out some confusion. Back off a bit - say we're in a parallel universe which doesn't have the same laws of math. First let's start with a quantity. Names are arbitrary, so let's call it "@". That's going to be boring by itself, so we'll make another quantity, and we'll call it "!". Independent quantities by themselves aren't interesting, so let's add an operation: "%". How does this operation behave? It might be nice to have an identity, that is, if we 'percent' a quantity with the identity, we get the other number back. It's arbitrary which one we choose, as they are the same, so let's pick '@'. So we have '@ % @' => '@' and '! % @' => '!'. Okay, but now what about '! % !'? Well, we could say that '! % !' => '@', or even '! % !' => '!', but we can also introduce a new symbol, so let's do that and call it '&'. So '&' is defined as '! % !'. Now what is '& % !'? Keeping it open ended we define it to be '$'. And '$ % !' => '#'. Now, let's figure out what '& % &' is. Since '&' is the same as '! % !'; we see that '& % &' is '(! % !) % (! % !)'. If we say that order in which we 'percent' doesn't matter (associative property), we can rewrite that as '((! % !) % !) % !', or '(& % !) % !' or '$ % !', which we defined earlier as '#'. As long as '! % !' => '&', '& % !' => '$', '$ % !' => '#', and the '%' operation has the associative property, '& % &' => '#'. Hopefully you can see where I'm going with this; the names I gave them were arbitrary. I could have easily have said 0 instead of @, 1 for !, 2/&, 3/$, 4/# and +/%, and you have your situation. If we have defined 1+1=2, 2+1=3, 3+1=4, and addition as associative, then 2+2=4. If 2+2=5, then either 3+1=5, 1+1≠2, or addition is not associative. We certainly could call 3+1, '5' if we wanted, but it would behave exactly the same way 4 does now. It would be the equivalent of writing 'IV' instead of '4' - the name changes, but the properties stay the same. Conversely, with addition being non-associative, why call it "addition"? Associativity is part of what defines addition - if you change that, you have something else.
"Non-standard" arithmetic are used all the time, however. That case earlier, where '!%!'=>'@' (er, 1+1=0)? That's modulo 2 arithmetic. We also have the case where '!%!'=>'!', except we call that multiplication, and we substitute 1 for @ and 0 for ! instead of vice-versa. We also frequently encounter physical situations where 2+2≠4. Take, for example, adding two liters of water to two liters of sand. 2+2≠4 in that case. Mix 2 cups vinegar to 2 cups baking soda, and the result takes up much more than 4 cups. Physical reality doesn't have to match with abstract mathematical constructs - however, instead of redefining 2+2=3 because the sand and water don't measure 4 liters afterward, or saying that 2+2=8 because of the carbon dioxide gas evolved with baking soda and water, we leave mathematics as a "pure" ideal, with 2+2=4, and realize normal addition doesn't apply to those situations. Likewise, the fact that interior angles of a triangle don't add up to 180 on the surface of the earth didn't invalidate Euclidean geometry - it still exists theoretically, we just realize that when doing surveying, we need to use a non-Euclidean geometry. -- 128.104.112.106 (talk) 01:28, 6 June 2009 (UTC)[reply]
Physicist Max Tegmark has proposed a Mathematical universe hypothesis which proposes that every mathematical structure exists physically in some universe. But from a mathematical point of view, it seems to go a bit overboard (ask in the math RD for more explanation). Note, non-standard arithmetic (in mathematical logic) means something different than what 128.104.112.106 seems to think. 207.241.239.70 (talk) 03:41, 6 June 2009 (UTC)[reply]
I believe the anon put "non-standard" in quotes in order to clarify that they weren't using it in the technical sense but just in the dictionary sense. --Tango (talk) 12:14, 6 June 2009 (UTC)[reply]
I thing I can clarify this by answering a similar question: Is there are universe in which you cannot cut a ball into five pieces and reassemble them into two balls identical to the first? Of course there is. You can't do that in this universe. It's not that the math is wrong. It's simply that our universe isn't set theory. It has parallels with it, but if you want to make a perfect model of our universe, it's going to be a bit more complex than that. 67.182.169.172 (talk) 17:21, 6 June 2009 (UTC)[reply]
That depends on your definition of "is"... but we are well into the realms of philosophy there, and we know how Steve feels about that! --Tango (talk) 18:31, 6 June 2009 (UTC)[reply]

Peacock and Peahen

Somebody told me that Peahen gets her eggs fertilized by orally shallowing the semen of the peacock.Is it true? —Preceding unsigned comment added by 202.70.74.155 (talk) 15:32, 5 June 2009 (UTC)[reply]

No. --Stephan Schulz (talk) 16:11, 5 June 2009 (UTC)[reply]
Fowlatio? Edison (talk) 01:59, 6 June 2009 (UTC)[reply]

Egg protein

Since eating raw eggs is not better than eating cooked eggs, what way should i cook them so that minimum protein is lost in the process of coooking (i.e. boil, fry etc.)? —Preceding unsigned comment added by 116.71.42.218 (talk) 18:00, 5 June 2009 (UTC)[reply]

This is a confusing question! If raw is not better than cooked, then why are you concerned about loss during cooking? As for outright loss during cooking, everything is still there except for maybe some small amounts that get stuck to the pan or leached out into the grease or other cooking medium, so something like hard-boiled would keep everything still in there. DMacks (talk) 18:04, 5 June 2009 (UTC)[reply]
The only protein loss during cooking would be the little bits that brown along where the egg meets the pan (see Maillard reaction). These are likely so small as to be insignificant, so you would be safe frying the eggs. As far as I am concerned, hard boiled eggs serve little purpose except perhaps to make deviled eggs or egg salad. I find them pretty bland and unpalatable. But if your concern is making sure you get every molecule of protein out that was there originally, taste be damned, then go with the hard-boiled method. --Jayron32.talk.contribs 18:28, 5 June 2009 (UTC)[reply]
Cooking will denature protein, but you will not lose the protein (specifically the amino acids). -- kainaw 18:33, 5 June 2009 (UTC)[reply]

To DMacks: I asked a question a couple of weeks back if raw eggs have any advantage over cooked ones and I was told no. And so if I work out and want max protein from the eggs I should hard boil them? I dont care about taste. —Preceding unsigned comment added by 116.71.59.87 (talk) 19:45, 5 June 2009 (UTC)[reply]

Kainaw already answered the above question, didn't he? Tempshill (talk) 22:10, 5 June 2009 (UTC)[reply]

Has anyone actually got any for the Euro elections? —Preceding unsigned comment added by 86.128.217.5 (talk) 22:43, 5 June 2009 (UTC)[reply]

I'm guessing you're interested in UK results: European Parliament election, 2009 (United Kingdom) should gradually accumulate that information - our editors usually get results up within minutes of it being announced...for an encyclopedia, we make a better news site than most proper news sites! There seem to be some results and recent polls there - but you'll probably want to hit reload every now and again. On the remote off-chance that it's not the slowly unfolding spectacular train wreck in the UK that you're interested in, there are links for results in the other countries at European Parliament election, 2009. My brother in law (Tony Goldson) just got elected to the Suffolk County Council with a landslide result - but no sign of that result here! SteveBaker (talk) 02:40, 6 June 2009 (UTC)[reply]
They are avoiding publishing any results (which seems to include exit polls) until Sunday evening to avoid influencing the votes in other countries (the Netherlands, for the second time in a row, has chosen not to do this, and may end up in a bit of trouble over it...). --Tango (talk) 13:17, 6 June 2009 (UTC)[reply]
Just out of interest, who is the "they" who are avoiding publishing, what, if anything, is enforcing this, and under what statutes, European or Dutch, may the Netherlands end up in trouble? Tonywalton Talk 22:35, 6 June 2009 (UTC)[reply]
You're not supposed to ask questions I don't know the answers too... There are some kind of European laws about how these elections are supposed to take place. Of course, the Netherlands is a sovereign country, so there isn't a great deal anyone can do about it, but they'll get told off by the European Commission and that sort of thing. I don't know if those laws apply to the media (who handle the exit polls), but I would guess it's just a gentleman's agreement that stops the exit polls being published. --Tango (talk) 23:27, 6 June 2009 (UTC)[reply]
So what's the point of the exit polls? Axl ¤ [Talk] 18:16, 7 June 2009 (UTC)[reply]
They are quicker, so you can get results out straight away once the embargo is removed, rather than having to wait for the count to finish. I'm not sure there were any exit polls in the UK, though. --Tango (talk) 00:51, 8 June 2009 (UTC)[reply]

work

Why would work be defined as a scalar rather than a vector? —Preceding unsigned comment added by 70.52.45.55 (talk) 23:29, 5 June 2009 (UTC)[reply]

Work done is synonymous with energy. The energy required to do something is equal to the sum of the energies required to do each part, they don't cancel out. --Tango (talk) 23:39, 5 June 2009 (UTC)[reply]
It does seem weird - if your "work done" is pushing a block up an inclined plane or something you kinda feel like it ought to be a vector...but remember that you could be doing work to increase the air pressure in a balloon - what would the direction be in that case? Suppose it was electrical work - charging a battery or something - again, no direction. SteveBaker (talk) 02:22, 6 June 2009 (UTC)[reply]
Lets just assume Work to be a vector. Say you're pushing a block of wood up an inclined plane. But the catch us that you're not applying the force along the inclined plane. Say you're applying the force directly horizontally, but the block still moves up. SO now, what would be the direction of your work? Would it be along the inclined plane, where the block actually moves, or horizontally, in the direction where you actually pushed? If we assign a direction for Work, it results in more confusions, and essentially loss of physical significance as being equivalent to Energy. So, it is best to keep Work as a Scalar. Rkr1991 (talk) 04:26, 6 June 2009 (UTC)[reply]
Or as one of my college professors would put it: "Because that's the way the math works out". Work defined is the dot product of two vectors, which always results in a scalar. -RunningOnBrains(talk page) 18:20, 6 June 2009 (UTC)[reply]
Yes, but that just changes the question to "why is it defined as the scalar product?". --Tango (talk) 18:29, 6 June 2009 (UTC)[reply]
If work (and by extension, energy) was a vector, there'd be no law of conservation of energy. When you fire a gun, more energy goes with the bullet than the gun, so there would be energy in the direction of the bullet. If you used the same gunpowder in a grenade, the energy would move in all directions and cancel itself out. Thus, more energy is extracted from the gunpowder one way than another way. Also, heat involves particles moving in all directions, so it would have zero energy. That would mean that anything that turns energy to heat destroys energy. — DanielLC 15:36, 7 June 2009 (UTC)[reply]


June 6

Jellyfish as pets.

An IP recently asked me if Jellyfish could be kept in a houshold aquarium. I told him/her basically that I didn't know for sure. I offered some in depth, but basic, guidance on how to do it, while reinforcing that I think it is a very bad idea to try. The whole reply can be seen at my talk page. After replying, I realised that while I may not have the answers, someone else might. Basically I have to questions:

  • Can jellyfish be kept in a household aquarium?
  • Was my reply right, wrong, or a little of both?

Thanks for any guidance on this.Drew Smith What I've done 05:58, 6 June 2009 (UTC)[reply]

Perhaps this web site can help.--Lenticel (talk) 06:13, 6 June 2009 (UTC)[reply]
I recently was at an event where someone was selling jellyfish kits for several hundred dollars each. I'll offer some observations from what I saw. This doesn't mean that his "kit" was actually a good idea or the optimal solution. He also seemed to be offering only one species of jelly, so these observations may not apply generally.
  1. The largest tank on display was not huge, perhaps only 80 gallons and held three large jellies and basically nothing else. No scenery, no reefs, no other fish, nada. Maybe this was an open water species and preferred that; I don't know.
  2. The smallest tank was perhaps 15 gallons and held one small jelly, and nothing else.
  3. The "ground" in his tanks seemed to be made of rounded plastic stones roughly the size of a thumb but somewhat fatter.
  4. Most of his tanks were vertically oriented, i.e. they were taller than they were wide. At the largest scale this was perhaps 3:1. All of the tanks were rounded and had no sharp corners on their vertical face.
  5. All of his tanks had heavy duty aerators. Noticeably stronger than I have ever seen on aquariums of similar size. These were mostly enclosed, presumably to prevent the jelly from swimming over the air flow.
  6. The type of jelly in question had very short tentacles.
I wasn't interested enough to ask about care and feeding, so I don't know anything about the water quality or nutritional requirements. I did see some sort of food packs though, so presumably they can eat things other than live/recently caught food. Hope this helps. Dragons flight (talk) 06:47, 6 June 2009 (UTC)[reply]
This helps, in that keeping jellies as pets is more common than I thought. On the other hand I have never seen a "kit" that actually keeps its intended inhabitants alive for even a fraction of its lifespan. Goldfish can live up to forty years in the wild, and seldom live more than a few months in "kits".Drew Smith What I've done 09:21, 6 June 2009 (UTC)[reply]

Physical strength requirements of Formula 1

Not sure if this is more suited to the Entertainment or Science section, so please feel free to move it

During today's qualifying for the Turkish GP a commentator stated that 10kg of weight would add 0.3 seconds per lap. Obviously that is quite a lot of time (adding about 21 seconds in a 70 lap race). This has made be wonder about the impact of keeping down a driver's weight and the trade offs that might entail. As an example, according to this page Jenson button weighs about 68kg, whereas I weigh about 95kg (I shouldn't, but I do!). At 0.3 seconds per 10kg per lap, I would, with the same driving skill, be 0.81 seconds slower per lap, or 56.7 seconds slower for a 70 lap race. So it would seem that, no matter how skillful, somebody weighing 95kg could never be successful in F1. This makes me wonder why successful F1 drivers do not tend to weigh even less. The jockey Willie Carson, seems to have weighed about 49kg, which would give him roughly a 0.6 second per lap advantage, or 42 seconds per race - presumably a woman would have even greater potential weight wise. Obviously skill is the overriding factor, but I cannot see why a woman, or a much lighter male would not be capabale of the same level of skill. So, I wonder if it is about the physical demands of F1. Is it that a degree of strength is required, that would override the weight advantage of a woman or a lighter male? I would have thought that riding a horse required greater strength than driving a Formula 1 car, and that jockeys must have a very good strength to weight ratio, but obviously there are other factors at play here, because Formula 1 is not full of drivers with the stature of jockeys. I would appreciate any thoughts on this. Thanks Chuny Beetroot (talk) 12:30, 6 June 2009 (UTC)[reply]

You do need to be very physically fit to withstand the g-forces. I'm not sure how strong jockeys need to be, but probably not as strong as a F1 driver - I can't see why they would need much strength. --Tango (talk) 13:08, 6 June 2009 (UTC)[reply]
Formula1.com says "cars must weigh at least 605kg (including the driver), but traditionally teams build the car to be considerably lighter and then use up 70 kg of ballast to bring it up to weight." So that means the weight of the driver is mostly irrelevant (all it does is given the team less ballast to move around to change the balance of the car for specific tracks). This reply (I don't know how reliable it is) claims quite a diversity in F1 driver heights (and surely weights, as the tall ones listed aren't beanpoles and the shorter ones noticeably chunky). That said, this article claims KERS promotes smaller drivers. 87.112.85.8 (talk) 13:59, 6 June 2009 (UTC)[reply]
On the official Formula 1 site the Toyota team doctor says, among other things, There is no other sport in the world which compares to the demands Formula One puts on the heart. SpinningSpark 14:07, 6 June 2009 (UTC)[reply]

Might one find one or more Higgs bosons at the center of a Black Hole? ---- Taxa (talk) 12:52, 6 June 2009 (UTC)[reply]

If our theories are correct, there are Higgs bosons everywhere, they are just difficult to see. We can't say what is at the centre of a black hole, it is a singularity, which is scientist-speak for "we haven't got a clue". --Tango (talk) 13:12, 6 June 2009 (UTC)[reply]
My thinking is somewhat askew from the thinking of others. I was trained specifically to think outside the box. I have no problem, for instance, with 10/zero(0)=10. Its logical to me that when you divide by zero nothing happens. I apply the same logic to the center of a Black Hole and to a Higgs boson . It may not be possible for anyone other than myself to consider that not having a clue is not an acceptable answer unless you explain why you do not have a clue. ---- Taxa (talk) 13:42, 6 June 2009 (UTC)[reply]
I'm afraid that isn't "thinking outside the box", that is just being wrong. If 10/0=10 then we can multiply both sides by zero and get 10=10*0, but anything times zero is zero, so you have 10=0, a contradiction. Thinking which is unconventional can be a good thing, but it still has to be logically consistent. We don't have a clue about what happens at the centre of a black hole because the mathematical model we use to describe black holes breaks down at the centre and gives a result that doesn't make sense (ie. various infinite physical values, which aren't consistent with how those values are defined). Hopefully someone will come up with a theory that can explain the centre of a black hole, but they haven't yet. If you have a theory which explains black holes in terms of Higgs bosons, feel free to write it up and submit it to a peer reviewed journal, but I doubt it will be accepted because I suspect you don't actually understand what a Higgs boson is (I'm not clear on the details myself, but I fail to see any way that their presence in black holes could resolve the singularity, if they exist they will be there because they are everywhere, that doesn't help anything). --Tango (talk) 14:17, 6 June 2009 (UTC)[reply]
Not quite. The problem is that you are still thinking inside the box. You still see the division sign (/) as operational. By outside the box I no longer see the division sign as operational when a zero value is below it. What you see is that 10 is the result of an arithmetic operation. What I see is that no arithmetic can or has taken place. Its as if I were screening the values with an "if, then" statement and if the value under the division sign is zero then no arithmetical operation can take place. You see the division sign instead as an arithmetical operator that is turned on not matter what the values are because you are still thinking inside the box. You are unable to recognize that the division sign itself, its use or operation is not always valid, that there is a chink in the division sign. -- Taxa (talk) 19:23, 6 June 2009 (UTC)[reply]
If you open your mind to much... [13]. What you do by "thinking outside the box" is redefining the / sign to mean something different from what it usually means. I can just as well claim that 2+2 = 5 (by redefining "=" to mean "bigger than" and "+" to mean concatenation of the decimal representations of the arguments. --Stephan Schulz (talk) 19:36, 6 June 2009 (UTC)[reply]
Taxa, you said "Its logical to me that when you divide by zero nothing happens." so don't come now with "By outside the box I no longer see the division sign as operational when a zero value is below it." Divide by zero means divide by zero. It is by definition operational. you cannot arbitrarily redefine the meaning of words like division and call if "thinking outside of the box". That's no different than saying that instead of deviding by rezo I will replace it with my favorite color, 10/0=blue=humpty dumpty=dejavu. Even when thinking outside of the box things still must make sense, otherwise it's just plain rubish. Dauto (talk) 19:49, 6 June 2009 (UTC)[reply]
Have you observed that 2+2=5? My observation of the function of the division sign is based upon empirical evidence that the division sign is not operational dependent upon the value which is below the division sign or in other words that the division sign is turned off or no longer functional dependent entirely upon the value of the divisor. -- Taxa (talk) 19:58, 6 June 2009 (UTC)[reply]
Mathematics is not an empirical subject. You can't "observe" 2, it is an abstract concept. You can define the "/" symbol in a different way to me if you like, but don't expect anyone else to use your definition because it isn't useful. Division is useful because it obeys certain rules (like a/b=c => a=bc) that let us work with it in useful ways, if you redefine it in a way that means those rules no longer hold, then it isn't useful any more. This isn't about being open minded, it is about pragmatism. There are all kinds of mathematical objects that can be defined (infinitely many, in fact, but don't ask me which infinity), some of them are useful, so get used, most aren't useful, though, so, for the most part, get ignored (pure mathematicians play around with things that aren't necessarily useful to the real world (until a scientist comes along and finds a use for them - scientists are amazingly good at that), but they are generally useful to something, even if they are just ascetically pleasing for are useful for making mathematicians happy). --Tango (talk) 20:12, 6 June 2009 (UTC)[reply]
My great great grandfather found that he could use a simple pan to fry fish, pan for gold, keep the rain off his head. Unfortunately he learned the hard way that his pan could not stop a bullet. Thanks to empirical evidence that his most valued multipurpose tool could not serve to save his life in the capacity of armor I now know that it is a function the pan is incapable of performing and not one which I would want to try. The division sign is merely a tool and its inability to function when the divisor is equal to zero is quite enough for me to reach the conclusion the division sign is not capable of this function. -- Taxa (talk) 21:08, 6 June 2009 (UTC)[reply]
Hmmm... there are some things best not tested by direct empirical observation! I don't see the relevance of that anecdote, though... We know the division operator isn't capable of functioning when the second parameter is zero, that's why we just say you can't do that. You're saying that it can function and functions as the identity, which isn't consistent with the rest of its behaviour. --Tango (talk) 22:07, 6 June 2009 (UTC)[reply]
No you have it right now. "...the division operator isn't capable of functioning when the second parameter is zero..." What many people do not realize is that it is the function of the division sign that is the problem rather than the value of zero. -- Taxa (talk) 22:47, 6 June 2009 (UTC)[reply]
When did I have it wrong? Could you give an example of someone that doesn't realise that? I'm not sure it is appropriate to distinguish between a problem with one or the other, rather than saying it a problem with how they interact, but if you had to choose one or the other then the problem would be with division, simply because you define zero first. --Tango (talk) 23:12, 6 June 2009 (UTC)[reply]
By accepting that the function of division breaks down when zero is the divisor then you can accept 10/0=10. If you insist that the function of division does not break down then you will argue that 10 divided by zero can not equal 10 because 10 times zero does not equal 10. -- Taxa (talk) 23:31, 6 June 2009 (UTC)[reply]
When you accept that you can't use the regular definition of division to handle division by zero you have a choice. You can leave it undefined, or you can arbitrarily define it to equal something. Either gives you a valid mathematical function, but the former turns out to be more useful. You can define 10/0=10 if you want to, but there is just no reason to do so. It doesn't help you in solving any problems, which is what maths is all about. --Tango (talk) 01:04, 7 June 2009 (UTC)[reply]
When I am writing a workaround to deal with some practical division by zero problem in C, I find that MAX_INT (or something similar, the largest allowable value) is more commonly a useful answer than "nothing happens". Sometimes MAX_INT but keeping the sign of the number being divided. Sometimes applying a sign to the number from some other variable. Other times "nothing happens" might suit me, or zero. So if you're going to argue from practicality, there isn't a definite answer to be found there either. 81.131.14.50 (talk) 00:53, 7 June 2009 (UTC)[reply]
Taxa, I think that rather than thinking outside a box, you may be still trapped inside one. When we learn mathematics, it is usually as an "operation" - an action. You start with two apples, you add three apples, and the result is five apples. That's the thought process you're applying when you state 10/0 = 10. "You start with 10, you divide by 0, which doesn't change anything, so you're left with what you started with, which is 10." When you really get into mathematics, however, you need to break out of this action-oriented box, and view mathematics as relationships, rather than actions. It's no longer "10 divided by 5 is 2", but also "5, when divided into 10 is 2", and "2 is related to 10 by the division with 5". It's this relationship view which fuels Tango's assertion that 10/0 = 10 implies 10 = 10*0. The relationship of 10, 5, and 2 in "10/5 = 2" is the same as the relationship of 10, 5 and 2 in "10 = 2*5", because of the relationship between division and multiplication. You could break that relationship, but then it's no longer multiplication and division, and furthermore, as Dauto mentions, you could just as easily say that "10/0 = blue" or "10/0 = a cup of weak tea" as "10/0 = 10", because the power, nay, the whole point of division as a concept is its relationship to multiplication and other mathematical constructs. Part of that relationship is that "X/Y = Z" implies that "X = Z*Y". You certainly are free to come up with a mathematical operation where "10 op 5 = 2" and "10 op 0 = 10". It wouldn't be division, though. (Mathematicians make up new mathematical operations all the time, though they usually call them functions. And again, although functions are usually taught as actions, they're more appropriately viewed as a relationship, as a mapping of a set of numbers onto another set of numbers.) -- 128.104.112.106 (talk) 00:27, 7 June 2009 (UTC)[reply]
I don't think using the apple analogy is fair when dividing by zero, as even dividing by fractions is counter-intuitive (10 apples divided by 0.5 people leads to 20 apples each). However, I would like to ask how this would apply to molecules, i.e. with the law of mass action, specifically chemical equilibria? For example, let's say we have HCl + H2O → H3O+ + Cl-, the HCl has fully dissociated (in this case there is one of each molecule, not one mole of each molecule). Thus the acid dissociation constant would be: KA = [H3O+][Cl-][H2O][HCl], which leads to 1*1 / 0*0. How is this solved? (Sorry if this is too off-topic and should be a new thread). --Mark PEA (talk) 16:46, 7 June 2009 (UTC)[reply]
The raction can temporarily reverse itself. [H2O] and [HCl] should be seen as the (non-zero) probabilities that at any given time there will be a H2O and/or a HCl present. Dauto (talk) 22:37, 7 June 2009 (UTC)[reply]
Okay so I should have stated that I'm assuming that after the reaction, the two products will never collide again (I tried to state it by saying there was only one of each molecule). I guess you will argue that on the reactants side there was only two molecules, so if they are capable of colliding why not the products, let's assume there is something surrounding the molecules which will attract their charge (effectively "solvate" but in a heterogeneous situation, maybe some complex situation à la Maxwell's demon). Anyway 128.104's response below answers my question. --Mark PEA (talk) 23:09, 7 June 2009 (UTC)[reply]
I used the operation on apples not to show intuitiveness, but to demonstrate how mathematics is usually taught. Your example of 10/0.5 = 20 uses the same "action oriented" viewpoint, illogical as the situation may be. "You start with 10 apples, divide them between 0.5 people, which results in 20 apples per person." The illogicality only points further to the fact that division has a broader life as a relational statement, rather than a procedural one. It's illogical because "dividing between 0.5 people" makes no physical sense. On the other hand, with division as a relationship, sense is restored "if there are 10 apples for every 0.5 people, then there is 20 apples for every 1 person." For your second point, you can only calculate equilibrium constants if the system is in equilibrium - you can't say that one hydronium and one chloride ion are "in equilibrium" with HCl and water. This is a similar situation to the original question! The fact that you are dividing by zero indicates that you've extended the mathematical models further than they are valid. If you just have two molecules, you can't derive an equilibrium constant, because it is not in equilibrium. Likewise, we can't say what happens at the singularity in the middle of a black hole, because our current models all break down. On the other hand, if we're happy saying (1*1)/(0*0) = 1, then we get a KA = 1 for HCl, which will result in no end of trouble when you're back to dealing with liter solutions of HCl, as the true value is closer to 10 -7. -- 128.104.112.106 (talk) 22:31, 7 June 2009 (UTC)[reply]
Yes, sorry, my question really was... If we discover an acid that fully dissociates, what is its KA value? (because you can't divide by zero) I guess you just don't give it one, or I am effectively asking what the KA of a proton is. I completely agree on your final point that 1/0 doesn't = 1, or any x/0=x, it was the reason why I brought up equilibrium constants. --Mark PEA (talk) 23:09, 7 June 2009 (UTC)[reply]
I think you (Tango) must have meant gravitational singularity but you linked to the dab page by mistake. As a mathematician, do you also think that mathematicians speaking of a mathematical singularity is also to be interpreted as "haven't got a clue"? SpinningSpark 13:57, 6 June 2009 (UTC)[reply]
Not by mistake, just by laziness! In mathematics, singularities are well defined places where something goes wrong and are well understood. In physics, a singularity is somewhere where our mathematical model indicates there is a mathematical singularity, which generally means our model is flawed. --Tango (talk) 14:17, 6 June 2009 (UTC)[reply]
The mathspeak for "haven't a clue" is indeterminate. Dmcq (talk) 14:36, 6 June 2009 (UTC)[reply]
Not really, an indeterminate isn't something we don't know the value of, it's something that doesn't have a well defined value. I think "conjecture" would be the closest mathematicians have (basically it means "guess"). --Tango (talk) 15:07, 6 June 2009 (UTC)[reply]
See also independence (mathematical logic). A conjecture at least might be provable from existing axioms if someone clever enough comes along. Something independent can't be proved without new axioms. The continuum hypothesis was a conjecture for a long time, but proved independent in the 1960's, putting it closer to "haven't a clue" (or maybe "meaningless") territory. 67.122.209.126 (talk) 18:12, 6 June 2009 (UTC)[reply]
Something which has been proven independent of an axiom system isn't something we don't know about. We know everything there is know, the result is just neither "true" or "false". I think conjecture is the better analogy. Conjectures could, in time, but proven or disproven (or proven independent), just as gaps in scientific theories could, in time, be filled. --Tango (talk) 18:29, 6 June 2009 (UTC)[reply]
I was thinking of imdeterminate as in indeterminate form. the limit of x/sin(x) as x goes to zero gives the indeterminate form 0/0. In this case one can find the value 1 by other means but anything is possible. But yes conjecture is very good. And independent and undecidable and uncomputable and incomplete too... I think maths is well on the way in developing a theory of types of "haven't the foggiest". Soon we'll have terms like meta-controversial for instance and know exactly what they mean. :) Dmcq (talk) 20:12, 6 June 2009 (UTC)[reply]
0/0 is indeterminate, which doesn't mean we don't know what it equals, it means it doesn't equal anything. isn't indeterminate, it is simply "1". If you solve it and get an indeterminate form as the answer that just means you've done it wrong and need to find a method which actually works. --Tango (talk) 20:16, 6 June 2009 (UTC)[reply]
Pssst, you mean as it goes to 0. Dragons flight (talk) 20:52, 6 June 2009 (UTC)[reply]
I was just testing to see if you were paying attention! (Fixed now.) --Tango (talk) 21:08, 6 June 2009 (UTC)[reply]
Hi there, I plugged into Wolfram|Alpha and got a nice graphical result. So 0/0 sorta converges to 1 with that equation but it doesn't equal 1. How weird. ~~ Ropata (talk) 07:18, 11 June 2009 (UTC)[reply]

Electrical arc

Is an Electric arc related to electrical ionisation as described here [14]. Thank you. Clover345 (talk) 17:16, 6 June 2009 (UTC)[reply]

Yes. --Dr Dima (talk) 21:17, 6 June 2009 (UTC)[reply]

Unknown garden plants for identification

Can anyone help me identify these please, all found in an english garden? This rather elegant garden plant http://i599.photobucket.com/albums/tt74/liverpoollarry/unknownplant1.jpg also had a more or less flat group of flowers above the frame. Photo taken a few weeks ago.

The small plant above the ruler in this scan http://i599.photobucket.com/albums/tt74/liverpoollarry/unknownplants2.jpg is part of a rockery-type plant, probably perennial, that I saw growing in low spreading mats with numerous flowers, and also growing out of cracks in the side of walls. When fresh the ends of the petals on the flower bend out more. It may be a Camanula or Bellflower, but they differ considerably in appearance and I want to try to identify its species or variety.

The plant beneath the ruler may be just a weed - but lots of them have suddenly appeared grouped together. Thanks. 84.13.50.195 (talk) 18:32, 6 June 2009 (UTC)[reply]

The first plant is an Aquilegia. --TammyMoet (talk) 19:38, 6 June 2009 (UTC)[reply]
The second plants: Campanula is latin for Bellflower. I'd say it's one of the Campanula family. The plant below the ruler is one of the Vetches, hard to say which - could be a Black Medick or a Hop Trefoil. It's a weed! --TammyMoet (talk) 19:43, 6 June 2009 (UTC)[reply]

Thanks. After searching around some more I think the middle one is probably Campanula portenschlagiana, variety Hoffman’s Blue, common name Dalmation bellflower. The final one seems to be, as you say, Hop Trefoil, Trifolium campestre. Edit: now I think the middle one is more likely to be Campanula muralis. The c. port... seems to be a much bigger plant - difficult to tell the size in photos. 78.149.117.117 (talk) 20:06, 6 June 2009 (UTC)[reply]

what kinds of companies use corporate astrologers?

Googling "corporate astrologer" (in quotes) shows some 22,000 results. So my question is about what kinds of companies use these servicse? Do any very large companies (market cap in the billions) use services like this? 94.27.141.190 (talk) 18:50, 6 June 2009 (UTC)[reply]

If they do, I doubt they would admit to it! --Tango (talk) 18:59, 6 June 2009 (UTC)[reply]
While only superficially comparable with astrology the article on Feng_shui cites sources that mention Donald Trump, Walt Disney, Singapore Polytechnic and the New York College of Health Professions inter alia as taking the subject seriously. Cuddlyable3 (talk) 19:06, 6 June 2009 (UTC)[reply]
Feng shui — as a decorating style — will generally produce an appealing aesthetic though, so even if you don't care for the mystical underpinnings, you might still choose to consult a feng shui designer. Dragons flight (talk) 19:59, 6 June 2009 (UTC)[reply]
I'm kind of skeptical about this, if only because if you ask ten feng shui "experts", you're likely to get ten different aesthetics. (Bullshit!, as I recall, demonstrated this rather well.) I'd say the appeal of the final aesthetic depends more on the decoration sense of the individuals in question than the principles of feng shui. -- Captain Disdain (talk) 07:34, 7 June 2009 (UTC)[reply]
This is more trivia than an answer to this question, but I'm sure I read somewhere about Microsoft and Apple competing in who had "the best" corporate astrologer. Tbh I think its no more than a psycological effect, but no one asked for my opinion.Alaphent (talk) 19:50, 6 June 2009 (UTC)[reply]
Microsoft and Apple! Couldn't have asked for a better response. Cite, please! 94.27.141.190 (talk) 19:52, 6 June 2009 (UTC)[reply]
Like I say, trivia only. It was read in on an online economics magazine, I've forgotten which, but it may have even been tongue in cheek. Sorry.... Alaphent (talk) 20:53, 6 June 2009 (UTC)[reply]
My guess is that only companies that sell horoscopes, or have some motive for promoting astrology, would pay for astrology services, since there's no credible scientific basis for the "predictive" results generated. Perhaps it's useful public relations in some segments, but that's marketing and not Science. --Scray (talk) 20:58, 6 June 2009 (UTC)[reply]
It is always good to cite sources on the refdesk. The above answer thinks too highly of people using logic, as can be illustrated by feng shui and Nancy Reagan#Influence in the White House. I have no sources to cite on corporate astrologers, but I am not going to sit and speculate on the refdesk that corporations make decisions based on "Science". Tempshill (talk) 23:44, 6 June 2009 (UTC)[reply]
There's also a huge gap between corporate decisions and individual decisions made by people on corporate boards. It's possible that someone is able to make his/her way into a position of power and decision-making - such as a CEO or president - and once there, make decisions based on astrology or coin-toss or any other technique. However, I suspect that such decisions would be worse than informed, scientific decisions, and would thus yield in undesired results. The corporation would quickly decide to remove such an individual, who is free to believe what they want as long as they don't do it with shareholders' money. Nimur (talk) 04:46, 7 June 2009 (UTC)[reply]
I notice that all the corporate astrologers on the first page are based in India. This seems excessive, even considering the possibility that India is renowned in corporate circles the world over for producing the best astrologers. So perhaps the answer is Indian ones. 81.131.14.50 (talk) 00:03, 7 June 2009 (UTC)[reply]
I'm from India, and yes, you'll stumble upon an astrologer here whenever you trip. But most of these men like to advertise themselves as corporate astrologers. True, no company is started here without a bhoomi pooja under some astrologers and swamis, and yes, there are a couple of high profile people who give their ""expert guidance" to companies for large sums, but nevertheless, you don't see many companies recruiting astrologers for their services. Say if they're going to start a new business, they might consult a family astrologer, or when they are about to open a new building, they might ask when is the "good time". But i'm very doubtful about corporate astrologers. Rkr1991 (talk) 04:02, 7 June 2009 (UTC)[reply]

The Mouse vs. Elephant Effect

I was watching a nature show, and a cheetah cub—the fearsome little devil—made a playful charge at a herd of wildebeest—which resulted in all of these 300+ pound animals fitted with horns making a run for it. I see this kind of embarrassing herd behavior all the time on nature shows. Obviously in some instances herd behavior has evolutionary advantages. But, when legions of gargantuan animals flee from some dogs—is this an example of a maladaptive behavior resulting from individuals evolving at the cost of the herd? i.e. it seems like it would make more sense for the prey to band together and confront the outmatched predator.

Alfonse Stompanato (talk) 21:54, 6 June 2009 (UTC)[reply]

Running away is generally the best way to avoid coming off worse in a fight. Is there any real disadvantage to running away? --Tango (talk) 22:03, 6 June 2009 (UTC)[reply]
Yes, there is, but it's a small disadvantage really. You can't feed when you are running away, and you spend some energy on running away, too. You are right, it is still much better than to risk injury or death. That is actually the reason why mammalian sensory system is "tuned" to make much more False Positives than False Negatives on identification of predators or of threats in general. If you make a False Positive, you stop grazing for five minutes, and then resume. If you make a False Negative, your brain stem is in someone else's stomach. --Dr Dima (talk) 22:36, 6 June 2009 (UTC)[reply]
But wouldn't a cooperative defense be so effective as to both conserve energy and prevent one of the herd from being picked off? The herd runs—and the young ones get taken out. I have actually seen some footage of aggressive behavior coming from water buffalo—they encircled their young, stood their ground, and actually gored some lion cubs that the pride hid in the tall grass (the herd stumbled upon the pride in this case). But, I guess my problem is that flight seems over represented in situations where the prey is greater than the predator in both size and number. Alfonse Stompanato (talk) 01:01, 7 June 2009 (UTC)[reply]
Yes Alfonse, you are right. Cooperation works better than "every one save yourself", but cooperation requires coordenated actions which require comunication. That's alot to ask from volution. Meanwhile the one wilderbeast that thinks twice before running is the one that ends up eaten. Just because a behaviour is more eficient doesn't mean that evolution will find its way there. Nevertheless some species do evolve cooperation as a defense. Dauto (talk) 03:36, 7 June 2009 (UTC)[reply]
It is faster to run, without thinking, than to take the time necessary to evaluate the situation, and perhaps conclude that flight is not necessary. There also may be advantages to running, such as exercise, and being physically prepared for the next time -- when flight may actually be very necessary. Also, the animals fleeing may be doing more than meets the eye. It may be akin to a training exercise, in which they practice their scattering techniques, and develop group dynamics that will be protective of the youngest or even the weakest. Bus stop (talk) 03:47, 7 June 2009 (UTC)[reply]
I'm pretty sure cheetahs raise their young as with most mammals. I'm guessing then if there was a cheetah cub, there would normally be cheetah parent/s close by. I don't know the details of what you were watching, perhaps the cheetah cub was being raised by humans or whatever but clearly it's unresonable to expect wildebeast to be able to appreciate such differences when such a thing is extremely abnormal. In other words, while it may seem odd for wildebeast to run away from a cheetah cub because of the risks from the cub, running away from a cheetah cub because cheetah cub usually means cheetah parent/s are close by makes far more sense Nil Einne (talk) 07:24, 7 June 2009 (UTC)[reply]
The cheetah can outrun the wildebeest but must select a young or ill animal for attack and is quickly exhausted. The defensive strategy of the wildebeest includes stampeding as a herd. This puts a predator in danger of being trampled and keeps young animals inaccessible, and it is a well adapted behaviour. It makes better sense than banding together against predator(s) that could then gather and harry the herd indefinitely. What the OP calls a playful charge by a cheetah cub is a serious activity by which it learns how to get close to a herd and how the herd reacts when spooked. A cub is likely to get closer to the herd than an adult so the alarm to the herd is correspondingly greater. BTW Mice do not prey on elephants and a mouse vs. elephant effect is unsourced. Cuddlyable3 (talk) 09:17, 7 June 2009 (UTC)[reply]
It's a matter of evolution: Suppose you are one of the Wildebeast - and suppose that the herd's instinctive strategy is to stand their ground and try to defeat the cheetah. By standing your ground - you run a small risk that the cheetah will kill you before the rest of the herd manage to bring it down. So a slightly better strategy for you, personally - would be to run away while the rest of the herd deals with the cheetah. This produces a small evolutionary benefit for those who possess the "run away" gene. Over hundreds or thousands of generations, evolution would ensure that sooner or later, the gene for the instinct to stand-your-ground would be out-evolved by the gene for running-away. Hence, Wildebeast evolve to run away. More notably, consider the opposite position: Suppose the whole herd has the gene for running-away (as real Wildebeast do)...what happens to the individual who gets the mutated "stand-your-ground" gene? Sadly, he dies the very first time a cheetah feels hungry. There is no way for that gene to get passed on. SteveBaker (talk) 01:26, 8 June 2009 (UTC)[reply]
See African Buffalo for an example of an animal that (sometimes) doesn't run. They aren't built for speed as wildebeest are. You can now argue whether wildebeest evolved to outrun cheetah or whether they survived because they could outrun them. Animals that flee from predators in a herd or flock have a higher survival rate than those that don't all act together, because most predators track motion and when there's too much going on they can get confused and become unable to track one individual. (Imagine you're in a crowd of waiters carrying trays of food and just when you've made up your mind to try some chocolate cake there's a juicy steak coming past. :-) Deciding "why" in animal behavior can usually be argued with some clever reason, but the likelihood of that being the actual cause is rather dubious. 71.236.26.74 (talk) 14:27, 8 June 2009 (UTC)[reply]

what would convince you of my ability to remote view?

I'm reading this article on "remote viewing" and I just wonder based on it why research universities the world over didn't either refute or verify the "paranormal" effect beyond a shadow of a doubt as soon as they learned Stanford was doing it? It is so easy to verify, isn't it? I mean, what would convince you of my ability to remote view? Isn't it enough if we pick a dictionary of ten thousand objects that are easy to draw (different colors, etc, to bring it up to 10k), you roll some dice and draw the one that comes up, while I tune into what you're drawing from thousands of miles away? I should get it right one time in ten thousand, getting it right 20% of the time should cinch the thing, shouldn't it? If not, what would convince you of my ability to remote view? —Preceding unsigned comment added by 94.27.141.190 (talk) 22:54, 6 June 2009 (UTC)[reply]

"Research universities" typically have serious research to do. But the James Randi Educational Foundation has debunked claims of PSI powers for ages - in fact, they will pay you ONE MILLION DOLLAR if you can prove, under reasonable experimental conditions, that you have any kind of paranormal abilities. --Stephan Schulz (talk) 23:06, 6 June 2009 (UTC)[reply]
Stanford is a very serious research University! My quesiton is why don't Princeton, Harvard, Yale, MIT, Columbia, and Brown -- the other research Universities of the same calibre...there aren't that many of them! -- try to either definitively verify or refute the claims?
Sometimes they do. There isn't much point, though. The people making the claims never accept the results and people that would accept the results don't believe the claims in the first place. Unless there is some evidence to support the claims, there is no reason to refute them - you can't spend limited research funding refuting every random thing somebody claims. There have been studies done on various types of ESP, etc., and the results have always been the same. There is no reason to suspect the results of a new study would be any different and there are better things to spend the time and money on. --

Tango (talk) 01:12, 7 June 2009 (UTC) The onus is on the OP to define in advance what remote viewing ability s/he claims to have. After that we would need to agree on a test that A) the parties are willing and able (practically and economically) to carry out, B) can give a result with high statistical confidence and C) includes precautions to eliminate unintended influences. "Beyond a shadow of a doubt" is a subjective not an objective measure, and it may reflect ignorance as readily as evidence. If I roll ordinary 6-sided dice the OP should be able to guess the result 16-2/3% of the time. Getting that right 20% of the time doesn't cinch anything. Cuddlyable3 (talk) 08:36, 7 June 2009 (UTC)[reply]

What am I currently consuming? I'll give you a hint: it's not a crustacean. -- Consumed Crustacean (talk) 23:11, 6 June 2009 (UTC)[reply]
Very funny. I'm no medium. 94.27.141.190 (talk) 23:54, 6 June 2009 (UTC)[reply]
By the way if I told you "a starbucks coffee" with lots of whipped cream and chocolate shavings and other assorted crap, and it turned out to be wrong, that would not be very good evidence that against "remote viewing". I want to know why research universities don't refute or verify the claims of Stanford (isn't it an ivy leage school?) in a scientifically conclusive way? —Preceding unsigned comment added by 94.27.141.190 (talk) 23:57, 6 June 2009 (UTC)[reply]
Actually, we're quite proud of the fact that Stanford is NOT an Ivy League school. We're a bunch of west-coast types - we're much too scruffy to fit in at a place like Harvard. (I have personally been told that I was not permitted to enter the Harvard library!) Though we don't have four hundred years of legacy, our fine institution is perfectly content to have a particle accelerator, a functioning radar telescope, and a Republican Think Tank six blocks away from an enchanted broccoli forest commune. Take that, east-coast intellectual types! And yes, it is very important to point out that our sister institution, SRI International, was responsible for the research into parapsychology; this research took place after the two institutions diverged. Nimur (talk) 01:11, 7 June 2009 (UTC) [reply]
The assertion that it hasn't been refuted is most likely wrong. This kind of crap has been proven to be exactly that over and over again, and yet people either come up with excuses that conveniently explain why it didn't work or simply ignore it. I mean, take homeopathy, for example -- there's no shortage of research that clearly shows it doesn't work, but that doesn't keep people from insisting that it does. The problem here is that people who want to believe in this kind of stuff really aren't swayed by reasonable arguments or scientific proof. One reason why universities don't waste time on researching this stuff is that there's no reason to assume it works, and plenty of evidence to indicate that it doesn't. If a really, really convincing remote viewer suddenly popped up, I'm sure that would be a different story. As it is, though, most scientists prefer to work on things that aren't completely ridiculous. (Or which, at least, are of interest to them, which may not be the same thing...) -- Captain Disdain (talk) 00:23, 7 June 2009 (UTC)[reply]
Stanford Research Institute is not Stanford University It is an independent research group. As for the 1970's work on ESP, methodological failures can produce amazing results. There is a long and well documented history of poor methodology at various labs producing amazing ESP results. The article SRI International says "A lengthy exchange ensued, with the external researchers finally concluding that the failure of Puthoff and Targ to address their concerns meant that the claim of remote viewing "can no longer be regarded as falling within the scientific domain." Edison (talk) 01:02, 7 June 2009 (UTC)[reply]
Stargate Project appears to have some related information, too. Tempshill (talk) 01:04, 7 June 2009 (UTC)[reply]
Edison is right on on this. SRI is a well-known case of physicists being hood-winked, and is not really part of Stanford University. Randi got involved with SRI started appering to be taken in by people like Uri Geller if I recall. --98.217.14.211 (talk) 02:21, 7 June 2009 (UTC)[reply]
Here is their paper, Information transmission under conditions of sensory shielding, published in the journal Nature in 1974. Nimur (talk) 03:43, 7 June 2009 (UTC)[reply]
I can recall a story in Snake Oil Science about the SRI. The anecdote was something about how the fellow in charge of some line of research (on remote viewing IIRC) presented that he had higher-than-chance results. When pressed, he admitted that there were some trials that didn't have that result, but because the people involved must have been intentionally screwing up the test, he stashed the data away and didn't count it. -- Consumed Crustacean (talk) 04:28, 7 June 2009 (UTC)[reply]
In direct response to the original question, "what would convince...", I think the starting point is to define remote viewing. In the paper I linked above, it was not really clear what "counted" as a successful remote viewing. For example, although I'm located in California, I can close my eyes and visualize a major landmark, say Eiffel Tower, in my mind. I can describe a lot of things about it. Because I'm pretty good with time-zones, I can guess the time of day, and because I check the international weather forecasts reasonably often, I can even tell you what kind of day it is in Paris. Have I "remotely viewed" anything? What level of correct information is necessary to count as a remote view? What level of incorrect information is necessary to throw out the "viewing" as unsuccessful? What information is necessarily provided or withheld? If you can develop a systematic framework for these questions, then you can run a double-blind experiment to test whether or not you have satisfied those criteria. What is fundamentally the limiting case is that you will not succeed at any such experiment if the criteria are sufficiently well defined. That is why the 1974 Uri Gellar research is discredited. The experimenters jumped right in and started counting P-norms, decibels of EEG attenuation, and so forth - without even defining what counted as a successful "view"! They did go to great lengths to preclude the possibility of Mr. Gellar knowing what the drawing should have been. But they forgot to define a valid judging criteria for the post-experiment comparison. So, they ended up with a bunch of line drawings, and they had some that really do look stunningly similar to the "target" image - but they never really stated "how similar" is an acceptable drawing. Even worse are the "descriptions of remote places" trials. In these experiments, the subject describes a remote place. Later, judges decide if the description matched the remote place. But the judgement criteria is not spelled out in gory scientific detail. If these details were present and explained in the experimental setup, the experiments would be repeatable and could be subject to scientific testing. As they stand right now, the experiments are subjective and not scientific. Nimur (talk) 05:41, 7 June 2009 (UTC)[reply]
The onus is on the OP to define in advance what remote viewing ability s/he claims to have. After that we would need to agree on a test that A) the parties are willing and able (practically and economically) to carry out, B) can give a result with high statistical confidence and C) includes precautions to eliminate unintended influences. "Beyond a shadow of a doubt" is a subjective not an objective measure of confidence, and it may reflect ignorance as readily as evidence. If I roll ordinary 6-sided dice the OP should be able to guess the result 16-2/3% of the time. Getting that right 20% of the time doesn't cinch anything. Cuddlyable3 (talk) 08:36, 7 June 2009 (UTC)[reply]
There are several things that have to be nailed down - but I think the OP's original statement would work pretty well. We have two identical books of (say) one hundred very diverse pictures, which are numbered - we agree that someone a suitable distance away uses a random number generator to pick a picture from the book and at 12 noon, GMT concentrates on it or stares at it or whatever the remote viewer claims is necessary. Meanwhile, the viewer has been kept alone in a windowless room with a clock from before the time that the random numbers were generated until they asserts that they have 'picked up the image', sketched what they saw and (since interpretation of that image is iffy) - we'll stipulate that the viewer must then look through their copy of the book and nominate between one and three photos that match what they saw. Once they have listed between one and three numbers, these will be communicated to the experimenters and that round of the experiment is over. The experiment will be repeated ten times. The results are then assessed with standard statistical tools to determine the probability of this happening by chance - and only if the result is that there is a less than 0.1% percent chance of this happening by chance alone do we give this effect any credance. I'm pretty sure that if someone were to agree to terms like this then the Randi institute folks would be happy to test that person and award them a million dollars for passing the test. SteveBaker (talk) 17:40, 7 June 2009 (UTC)[reply]
Incidentally, that would require them to get at least 4 out of 10 correct (assuming I can do basic probability, which is a big assumption... it took me two attempts!). --Tango (talk) 18:26, 7 June 2009 (UTC)[reply]
Actually, I'm not much of a statistician, but I'm not sure one set of 10 would be enough (unless they are claiming 100% reliability, which they rarely do, they just claim to be better than random chance). I think you need to do several sets of 10 and then do some fancy statistical analysis. Do we have a statistician in the room? --Tango (talk) 18:30, 7 June 2009 (UTC)[reply]
Imagine if Maxwell's Equations only worked "better than random chance." You'd pick up your mobile telephone, and 3 out of 10 times, its batteries would undergo the statistical thermodynamic processes necessary for chemically induced voltage. Electrons would statistically drift down the conductors with a 1 in 5 outcome; and they would flow into a transistor amplifier and 1 out of 6 times, amplification would occur... more electrons would flow into an antenna and induce an electromagnetic perturbation on the surrounding air... depending on convection patterns, the air near the antenna would perturb the dielectric constant 1 in 2 times, allowing effective coupling between the antenna and the radio channel... oh wait, these things are already statistical. Using statistics is perfectly fine to describe physical laws, if there is a physical law to describe. We're able to build machines and cope with statistical chances because when we scale up the size, the law of large numbers kicks in and we have a nearly deterministic system. Scientists are perfectly happy to use statistics, but we don't like making crap up. That's why we don't believe in psychokinesis or remote viewing. Nimur (talk) 19:09, 7 June 2009 (UTC)[reply]
It's perfectly legit to be impressed by a result that's consistently "better than random chance". Even if Remote Viewing does not work with 100% reliability (there are human factors involved.), or even if it doesn't work by the same mechanism that its proponents put forth (Extraordinarily likely, even if RV is real!) it would still be an exciting discovery, well worth The Amazing Randi's million bucks.
If I could predict lotto numbers "better than random chance" the Lottery Commission would absolutely be interested in finding out how I was doing it, they wouldn't wait until I'd revised my method to 100% reliability. APL (talk) 16:27, 9 June 2009 (UTC)[reply]
I agree with APL and indeed I would be disturbed if any scientist completely ignored something that was statiscally shown to be not due to random chance simply because the result wasn't impressive enough. Generally speaking, in most science anything which is not due to random chance is usually considered worthy of further analysis (i.e. it is often the null hypothesis that a difference is simply due to random chance and it is only when it is not that the result is worth further investigation). I'm pretty sure that the JREF for example only requires someone to be able to do something good enough that it is not due to random chance. And I suspect many sceptics likewise. Of course, it's not possible to prove something wasn't due to random chance, you can just say it is highly unlikely so if the difference is minor, the result may be mostly ignored for reasons of timing, funding etc (and for something like the JREF they'd likely need many trials before they're convinced you are really capable of predicting something better then random chance if the percentage is small to avoid the risk you just got luck) but it doesn't mean the result isn't of potential significance. A good real example of course is with drugs. No drug testing agency I know of requires a drug to be 100% effective. Nor does any company not release a drug simply because it is not 100% effective. The primary thing they require/expect is that the drug must be better then a no drug or perhaps an existing drug. And indeed many patients are perfectly happy with drugs that are not even close to 100% effective when the alternative is worse. (Of course this depends on the condition and side effects, a drug which reduces chance of pregnancy by 10% and has serious common side effects like death will be of little interest, a drug which reduces chance of cancer metasasing by 10% and has few side effects is likely to be of great interest.) Nil Einne (talk) 19:50, 9 June 2009 (UTC)[reply]

1 in 2 times allowing effective coupling between the antenna and the radio channel...:: You just described AT&T cell service! —Preceding unsigned comment added by 94.27.225.206 (talk) 20:48, 7 June 2009 (UTC)[reply]

June 7

bound photons

Spontaneous emission of bound photons from relativistic free electrons. What is a "bound photon"? Please provide an answer that can be understood by a biologist. --JWSchmidt (talk) 01:57, 7 June 2009 (UTC)[reply]

From a really quick glance at the referenced paper it looks like the "bound photon" is a photon in one of the cavity modes, that is, a photon emitted in a discrete rather than a continuous spectrum. Cavity modes must obey the boundary conditions imposed by the cavity walls, and therefore may not, generally speaking, have just any wavelength (and any energy), but only one of the discrete -- but infinite -- list of allowed wavelengths (and thus allowed energies). Generally, a particle is said to be bound when it cannot escape to infinity in any direction. An energy spectrum of such particle is discrete, at least as long as the potential it is bound in is not changing too fast or the lifetime of the particle is not too short. --Dr Dima (talk) 02:25, 7 June 2009 (UTC)[reply]
Thanks. Cavity quantum electrodynamics talks about cavity photons functioning as part of a quantum computer. Would such photons be "bound photons"? --JWSchmidt (talk) 02:45, 7 June 2009 (UTC)[reply]
Yes, I think they would be. --Dr Dima (talk) 03:56, 8 June 2009 (UTC)[reply]

Many-worlds Question

Is it possible that beings from another universe can come into ours and interact with us? Or if quantum immortality is true, then won't a person visit us, or has already visited us (even me personally)? —Preceding unsigned comment added by 66.66.149.43 (talk) 04:41, 7 June 2009 (UTC)[reply]

No. Nimur (talk) 05:00, 7 June 2009 (UTC)[reply]
There is nothing in physics which would prevent this from happening, because what you are describing is beyond the purview of physics (which only deals with our universe, even if other universes exist). It is on the same level as asking "is it possible that God exists?" Of course it's possible, but physics has nothing to say on the matter.-RunningOnBrains(talk page) 05:07, 7 June 2009 (UTC)[reply]
Mind me asking what the heck is quntum imortality? Dauto (talk) 06:19, 7 June 2009 (UTC)[reply]
Why not read our article Quantum immortality? Nil Einne (talk) 08:36, 7 June 2009 (UTC)[reply]
"Quantum immortality" is an incorrect and backwards reading of the anthropic principle. Its premise is something like this - "if a being is conscious, then all previous incidents which might have rendered it no longer conscious did not occur." For some reason, which is the really illogical part, it then concludes, "no future event will occur that will render this being unconscious (dead)." Somehow, this is linked into the idea of a quantum non-deterministic universe, by way of superposition of all possible outcomes of every possible event - and self-selection (by anthropic principle) of only those events which maintain the being as conscious (living). This doesn't really have anything to do with "quantum" anything. I'm not even really sure why this theory gets propagated - it's sort of a half-baked, nonsensical idea. But, like perpetual motion, immortality seems to spark some kind of fury in non-scientists, who hope to cherry-pick the parts of science that will somehow support their wishful thinking. Nimur (talk) 14:41, 7 June 2009 (UTC)[reply]
I think you are the one misreading it. The theory is more like: If the "many worlds" interpretation of quantum mechanics is true - then every quantum-mechanical event that can turn out in more than one way results in multiple universes in each of which the event happens differently. Since all events are ultimately quantum mechanical in nature - there is a universe in which you personally died because you were killed in a car wreck that you narrowly avoided in this universe. There is another in which you died due to a childhood disease. There are in fact an infinite number of universes in which every possible fate that could befall you has indeed befallen you.
Now - marry that (highly theoretical, and possibly dubious) hypothesis with the "anthropic principle". The 'strong' anthropic principle says that the universe is conducive to your existance BECAUSE if it was not, you would not be here to observe it. So, (for instance) it has been remarked that if the earth did not have a large moon - or if the charge on the electron differed by 1% or if water didn't expand a little as it froze - then life on Earth would be impossible. Why are we so amazingly lucky in all these respects? Well, the anthropic principle points out that in all existances in which there is intelligent life that is able to observe these conditions - the conditions must be conducive to that life.
OK - so put the many worlds and anthropic principles together - and you say that since (by definition) the only universes in this infinite number of universes in which you are able to note that you are still alive are the ones where you happened (by sheer chance) to have had all of the prior quantum events come out in favor of you living. Taken to an extreme - you cannot die because there will always be a universe in which by an AMAZING sequence of lucky breaks and one in a quadrillion chances - you happen to survive. Hence, you PERSONALLY will be immortal - although those around you will not (although they'll all be immortal in some other universe).
However, those who think this is "wishful thinking" need to carefully consider what this really implies. Suppose you are really sick - something painful and incurable. With this combination of hypotheses, you can't die. No matter what - you'll be in a universe where (though in continuous and nearly unbearable pain) - you can't commit suicide. You put a gun to your head and pull the trigger to try to end your misery...the gun will jam - or a freak gust of wind will blow the gun out of your hand. No matter what you do, there is no way out. Given you'll live forever - it's certain that you'll lose limbs, become blind and deaf - be in continuous anguish...for all eternity. If ever there was something close to old-testament hell - then Quantum Immortality is it.
(Going off-topic here.) Steve, I thought you said you worked your way through the whole Bible once? Where exactly did you find this "old-testament Hell"? There are a few lines in the Book of Daniel, but other than that Hell seems to be almost entirely a new-testament concept, as far as I can tell. The OT God might have done a bit of smiting, but once the smitten were dead, the narrative appeared to stop. --Trovatore (talk) 21:11, 7 June 2009 (UTC)[reply]
I don't remember much of whatever I read of the bible but based on the behaviour of God in the OT I would be just as worried about the OT heaven as I would be the (largely undisscussed?) hell. Let's face it, that God was psychotic.Nil Einne (talk) 19:55, 9 June 2009 (UTC)[reply]
I sure hope there is an error in the theory somewhere!
SteveBaker (talk) 17:10, 7 June 2009 (UTC)[reply]
Also, not every divergence into "many worlds" will result in an explicit separation between "living" and "dead" - there are hundreds of trillions of quantum "decision points" which occur all the time, and most of them have no impact on whether or not you will be conscious and alive at the next "time step." (Think about every photon of sunlight colliding with every molecule of nitrogen on its way down to earth - most of the time, whichever way the stuff scatters out, you live anyway). But, the many-worlds interpretation suggests that the universe necessarily schisms at each of those. So, there should be exponentially increasing numbers of universes, most of them pretty similar, with an exponentially schisming "thread" of consciousness. Which one is the "real" you? This whole infinite-universe-schisming methodology is sort of silly, especially since it yields absolutely nothing predictive or otherwise scientifically helpful in understanding our universe. Nimur (talk) 18:30, 7 June 2009 (UTC)[reply]
And it also does not follow from logic or basic understanding of quantum mechanics. Quantum mechanics describes statistical probabilities for certain state transitions, rather than deterministic behavior. However, the common interpretation that there is a "small but nonzero chance" that anything can happen is simply incorrect. There is a probability density function for any physical observable, and that probability can and is often zero for certain outcomes. So, "immortality" implies that at no time will you ever encounter any situation which has a "dead-end" on the constantly schisming set of outcomes - which is sort of silly also, given that we observe living creatures dying all the time. Changing the quantum state of a bunch of atoms must still follow rules. Nimur (talk) 18:36, 7 June 2009 (UTC)[reply]
I think you really need to start by defining "universe". I would (roughly) define "universe" to mean a collection of causally connected things (you may not have everything causally connected to everything else, but there needs to be at least a connection via other things). By that definition, any being that can enter our universe is part of our universe. So, by definition, the answer to your question is "no". --Tango (talk) 16:17, 7 June 2009 (UTC)[reply]

Thanks for the explanations. I can't help but think that calling that wishful thinking is too generous. Hogwash comes to my mind. Dauto (talk) 22:29, 7 June 2009 (UTC)[reply]

OK, great points all around first of all. To me, quantum immortality makes sense. But because I can personally theoretically live forever, then that means that everything works like that, right? Since everything is made up of quantum particles then everything should have "quantum immortality" in the sense that anything can happen to it, it's just that in this universe we're observing it doesn't. But wouldn't it follow that if anything is possible then wouldn't someone from another universe be able to travel to this universe and say "hey! I'm from another universe!"? Because it's a possibility. So if the theory is true, then how come this hasn't happened OR is it completely probable that it can happen, it just hasn't happened yet? —Preceding unsigned comment added by 66.66.149.43 (talk) 00:22, 8 June 2009 (UTC)[reply]

A possible answer is that it is physically impossible to travel between universes. Everything which can physically happen, in some universe somewhere, does; but not anything else. A more gratifying answer is that it is physically impossible unless a time machine exists at the destination. (That is to say an inter-universe travel machine, universes and moments in time being the same thing.) 213.122.32.39 (talk) 03:52, 8 June 2009 (UTC)[reply]
Another thought which just occurred to me is that the emergence of a traveler into just one universe, and not any of the very similar alternate versions of that universe, is by definition an extremely unlikely event. ("The same across universes" is the definition of likely.) So if time machines have perfect accuracy, then you'd pretty much never see anybody arrive, just because of probability. Then again I suppose you have to multiply that tiny probability by the number of possible visitors, which is very large since it includes everybody in all possible futures (even though most of them wouldn't want to come here), so in conclusion, meh. 213.122.32.39 (talk) 04:22, 8 June 2009 (UTC)[reply]

Speeding up formal education

What research has been done into increasing the rate at which students can complete their formal education (e.g. decreasing sleep and leisure requirements to allow a greater course load, increasing retention from lectures, increasing comprehension-adjusted reading speed, increasing learning ROI on practical exercises)? Is this likely to become more of a priority if the demand for highly skilled workers continues to increase? NeonMerlin 04:42, 7 June 2009 (UTC)[reply]

I think a lot of research has been done on time-to-productivity. In my fields, there is an always-raging debate between the need for advanced degrees versus simple bachelors' degrees - in other words, which is more productive, two years of on-the-job training, or two years of post-graduate study towards a Master's Degree ( ... or six years of post-graduate training for a Ph.D.)? There's a lot of academic debate about this as well. Most academic research into education is not about "speedup" as much as it is about improved retention and comprehension. There's a lot of research on "force-multipliers" - such as using technology to allow one teacher to impact more students. I think this is the most practical framework you will find for "speeding up" formal education. Also, the military has developed a huge body of rapid training - take a look at AKO or NKO. I hear that the language courses for many overseas deployments claim to teach "basic fluency" from zero prior knowledge in the span of six or eight weeks (though this may be a dubious claim). Nimur (talk) 04:59, 7 June 2009 (UTC)[reply]
The question that comes to mind is "..and then what?" Formal education should not be a goal to be completed in and of itself. Used to be a time where it was thought to prepare you for a career in your later life. Standard testing (which works for some) has created a slew of students who are not prepared for anything but solving standard tests. There are "robot hack" jobs that require nothing else. The worrysome thing is that ingenious and creative minds that industry would need fall through the hiring cracks. As a consequence technological advances and inventions needed to stay ahead of the curve aren't happening. (And we'll all go wondering why the next economic crash happened.) If you don't spend time-off from studying for "leisure requirements" how do you ever build up a social network. Your course results will only get your resume past the software keyword engine in hiring. If you don't show sufficient social skills during the interview you won't get hired. (OR: Just last week my hubby's group turned down the more qualified applicant because they all agreed he appeared to be hard to work with.) Word of mouth is a far more effective tool for getting contracts or getting hired than any other I know of. Most (better) universities check applications for extracurricular activities. 71.236.26.74 (talk) 19:26, 8 June 2009 (UTC)[reply]

Magnetic force based "black holes"

Is there a "Black hole" based on the force of magnetism rather than gravity? -- Taxa (talk) 04:58, 7 June 2009 (UTC)[reply]

No. Nimur (talk) 04:59, 7 June 2009 (UTC)[reply]
Black holes form by a runaway collapse of matter, i.e. increasing density causes further increase in density ad infinitum. Since there is no phenomenon in physics by which a magnetic field could intensify itself, a "magentic hole" is impossible. -RunningOnBrains(talk page) 05:03, 7 June 2009 (UTC)[reply]
but the magnetic force is much stronger than the gravitational force (brian greene: "a puny magnet can pick up a paper clip, overcoming all of the massive Earth's gravity" [paraphrased]. In fact gravity is some e-42 as strong as magnetic force!).
So it is a valid question, could we collapse a black hole by magnetic compression? 79.122.74.190 (talk) 07:35, 7 June 2009 (UTC)[reply]
No, as Runningforbrains said, the issue is not the strength of the force, but the feedback loop that is set up. Gravity results in an increase in density, causing increase in gravity. There is no mechanism by which the magnetic force could alter its magnetic domain and cause an increase in force, without the external application of energy.YobMod 08:16, 7 June 2009 (UTC)[reply]
My question is whether a traditional, gravity-based black hole, could be created through the initial compression of the proper mass down to the proper size not due to Gravity, but through a magnetic force? The point is that without the magnetic compression, the mass would not be critical enough (small enough volume) to turn into a black hole, but with the magnetic compression, the mass becomes small enough to pass the critical point and become a blcak hole. Would this be possible? 79.122.44.114 (talk) 10:23, 7 June 2009 (UTC)[reply]
Sure. It doesn't matter how the necessary density is achieved, just that you get enough mass into a sufficiently small volume. Some physics models predict the possibility that we will even be able to create micro black holes using the Large Hadron Collider. (In the LHC, very large amounts of energy – effectively mass, by E=mc2 – get packed into a very tiny space when highly energetic particles collide. There's an outside chance that sufficient density can be achieved to make tiny, temporary black holes in this way.) TenOfAllTrades(talk) 15:38, 7 June 2009 (UTC)[reply]
Why would the black holes from the LHC be temporary? Once they are formed, doesn't anything that come within their event horizon just increase their mass (remaining in a point volume), making them ever bigger and bigger black holes? If not, then what's to stop whatever process would end these black holes from ending (on a larger scale) normal black holes? Thanks! 94.27.225.206 (talk) 15:53, 7 June 2009 (UTC)[reply]
See Hawking radiation. In theory, the smaller the black hole, the shorter its lifespan. A black hole created by the LHC would likely evaporate before it even gobbled up a single atom. This is why actual physicists are not concerned by the possibility of mini-black holes, even if a few botanists and biochemists are. -RunningOnBrains(talk page) 16:29, 7 June 2009 (UTC)[reply]
The magnetic force is not stronger than the gravitational force. They have different units. It's like saying a pound is heavier than a foot is long. The closest you could get is using Planck units, in which case the gravity and the electromagnetic force both have a strength of one. — DanielLC 15:29, 7 June 2009 (UTC)[reply]
Daniel, you should be more careful with what you say. I can't think of a single way in which your statement that the electromagnetic force may have a strength of one would make any sense. The most natural way to measure the strength of the electromagnetic force is through its coupling constant which is adimensional. Dauto (talk) 17:53, 7 June 2009 (UTC)[reply]

Muscles + Oxygen = Energy

Would any of you know of an exercise or a few types of exercises to make your muscles require less oxygen?

Sleep. Cuddlyable3 (talk) 09:58, 7 June 2009 (UTC)[reply]
I don't think there's anything that makes them require less oxygen, but I know anaerobic exercise makes it so you can get oxygen there faster. — DanielLC 15:24, 7 June 2009 (UTC)[reply]

Intelligence as absence of stupidity

It is often said that "intelligence has its limits, but stupidity knows no bounds." This suggests that intelligence is merely the absence of stupidity, just as cold and darkness are the absence of light and heat (hence there is a minimum possible temperature but no maximum possible temperature), and that as a perfect vacuum would be perfectly dark, so too would it be perfectly intelligent. Is there a theoretical elementary particle that carries stupidity? Is it possible that some or all Standard Model particles carry it? NeonMerlin 07:07, 7 June 2009 (UTC)[reply]

Um what? Particles carrying stupidity??? Intelligence is present in a vacuum??? How do you even define intelligence and stupidity let alone carry them in particles or observe them in vacuum? I think your confusing a cute, quaint or fun comment with some sort of deep scientific statement. I doubt even many philosophers will make such a suggestion Nil Einne (talk) 07:14, 7 June 2009 (UTC)[reply]
It's also said, among other things, that a stich in time saves nine, but to hear that and start wondering what kind of a garment the stitches are on is to miss the point of the proverb. These aren't some kind of universal and literal hard facts, you know. -- Captain Disdain (talk) 07:28, 7 June 2009 (UTC)[reply]
Sayings do hold deep truths but may have to be slightly adjusted to be exactly right. For instance one can't have one's cake and eat it, unless one does them in that order (and I'm not going to think of the other alternative). A bird in the hand is worth two in the bush, unless you're talking about women. If pigs could fly a lot of biology and physics would have to be rethought but I'm sure we'll manage it one day with genetic engineering. I think in this case the saying is referring to the fact that for instance an intelligent person would say something intelligent and then stop whereas an idiot could go on wittering for ever. And furthermore.... :) Dmcq (talk) 07:49, 7 June 2009 (UTC)[reply]
It simply means that stupidity is not a conserved quantity. SpinningSpark 09:26, 7 June 2009 (UTC)[reply]
Intelligence is an attribute that can be defined and tested in various ways. There is no consensus on its limits. The word stupidity is not as suggested the opposite of intelligence but a disparaging term for negligent thinking. Persons that have high intelligence by some criteria, e.g. IQ score, are not immune to stupid actions. The OP offers hypotheses about a particle nature of stupidity that fail the scientific method because they lack falsifiability or a useful prediction. Cuddlyable3 (talk) 09:53, 7 June 2009 (UTC)[reply]
I'm sure I saw an advertisement that said there was a clever-dumb balance so it must be a conserved quantity. Advertisements are covered by the advertising standards agency and must be truthful. Dmcq (talk) 14:23, 7 June 2009 (UTC)[reply]
I presume you are referring to the toothless Advertising Standards Authority (United Kingdom) created by the advertising industry themselves whose idea of "self-regulation" is the occasional meaningless ticking off. If you believe anything they say . . . Stupidity behaves rather more like the (unconserved) entropy equation;
Where Q is intelligent actions and T is the "intelligence temperature" - average intelligence per individual. SpinningSpark 13:18, 8 June 2009 (UTC)[reply]
Does anyone else see how ironic this question is? --Shaggorama (talk) 22:52, 7 June 2009 (UTC)[reply]

Heat death, temperature and entropy

The article about Heat death says "The heat death is a possible final state of the universe", and that is a state in which "it has reached maximum entropy" and 'In a "heat death", the temperature of the entire universe would be very close to absolute zero'. The article about entropy says "When heat is added to a system at high temperature, the increase in entropy is small. When heat is added to a system at low temperature, the increase in entropy is great". So In heat death, the entropy becomes maximum and temperature is the lowest, but if heat is added to a system (I think that means a rise in temperature), the entropy increases. Then what is the high entropy in the heat death comprised of? I have been guessing that high entropy usually means high temperture. Like sushi (talk) 10:37, 7 June 2009 (UTC)[reply]

You can think of it this way: when you transfer some heat from a high-temperature system to a low-temperature system, the high-temperature system loses a relatively small amount of entropy while the low-temperature system gains a relatively large amount, so overall entropy increases. The entropy is (locally) maximum when the temperatures are equal. Adding heat would increase the entropy, but the heat has to come from somewhere (first law). If the whole universe is at the same temperature then the loss in entropy where the heat was removed would exceed the gain in entropy where it was added, which is forbidden by the second law. -- BenRG (talk) 14:20, 7 June 2009 (UTC)[reply]

Wet matches

I left a box of matches outside last night and they got soaked in the rain. When I try to strike them, the head of the match just crumbles into a wet mush. If I leave them and the box out in the sun today, might they dry out and be useable again? Frank Bruno's Laugh (talk) 11:18, 7 June 2009 (UTC)[reply]

Sure, why not. Try it and let us know tomorrow. Avoid the temptation of putting them in the oven on a low setting though (for obvious reasons!) 94.27.225.206 (talk) 12:41, 7 June 2009 (UTC)[reply]
Yep, that worked fine. The charcoal that go wet is still a bit mushy! :) I did consider putting the matches in the electric oven on a low setting (starting with one), thinking that it wouldn't have been hot enough to ignite them, but I guess that wouldn't have been a good idea? Might it work with the charcoal? Frank Bruno's Laugh (talk) 15:01, 7 June 2009 (UTC)[reply]
I'd be surprised if they still worked. In my experience regular matches don't have a lot of tolerance for weird environments (and degrade rather quickly with age or humidity). That being said, matches are pretty cheap, so it's not exactly the end of the world. --98.217.14.211 (talk) 14:59, 7 June 2009 (UTC)[reply]
They actually seem in pretty good shape, so I guess I got lucky (they did get a real soaking) Frank Bruno's Laugh (talk) 15:05, 7 June 2009 (UTC)[reply]
If you're taking some matches somewhere they run the risk of getting wet (e.g. a camping trip), a tip I once read is to dip them in melted candle wax. I can't say I've ever tried it, and I imagine a cheap lighter would be a better bet. 87.81.230.195 (talk) 23:58, 7 June 2009 (UTC)[reply]
Yes, yes. Before use, just melt the wax off with a lighter. 79.122.119.18 (talk) 06:21, 8 June 2009 (UTC)[reply]
I see what you did there. Tempshill (talk) 17:41, 8 June 2009 (UTC)[reply]
I have dipped old-style kitchen matches (the old-fashioned strike-anywhere kind) into wax and that made them completely waterproof and they would still light just fine without having to melt the wax away or anything. I haven't tried it with safety matches. 207.241.239.70 (talk) 19:12, 8 June 2009 (UTC)[reply]

Human blood

What are the risks with drinking (fresh) human blood? Vimescarrot (talk) 14:01, 7 June 2009 (UTC)[reply]

Oh, and the possible (dietary) health benefits? Vimescarrot (talk) 14:07, 7 June 2009 (UTC)[reply]
Maybe I'm stating the obvious but a dietary benefit would be increased iron intake. I'm guessing that there are no immunoreactive issues as the antigens should be metabolized and/or not absorbed by the gastrointestinal tract. --Mark PEA (talk) 15:32, 7 June 2009 (UTC)[reply]
It might be worth pointing out here that increased iron intake is not always a benefit. Iron is a necessary nutrient, but it's also somewhat toxic, and there is apparently no dedicated mechanism for getting rid of the excess. There is some information at our articles on human iron metabolism and iron overload.
A few years ago there was a minor splash made by the observation that many of the risk factors for heart disease have in common that they are associated with higher levels of iron in the body, and it was hypothesized that iron was a contributing cause to heart disease. I can't find an article about that in WP, and I don't really know how the idea has fared, though I imagine we'd have heard more about it if there were good evidence for it. --Trovatore (talk) 20:01, 8 June 2009 (UTC)[reply]


There are obvious risks of the transmission of disease. I think that's probably about it. You could overdose on iron (and maybe other constituents) if you drank too much of it, but that basic principle applies to anything. --Tango (talk) 16:35, 7 June 2009 (UTC)[reply]
There's also likely a risk of criminal charges, depending on who's blood it was and how you acquired it :-D -RunningOnBrains(talk page) 17:03, 7 June 2009 (UTC)[reply]
You're not planning to eat your bone tumor, are you? Nimur (talk) 18:20, 7 June 2009 (UTC)[reply]
Sorry, but I must note that this line made me laugh for no less than 20 seconds. Tempshill (talk) 04:29, 8 June 2009 (UTC)[reply]
Bone tumours aren't made of blood. On a slightly related note...What about the, uhh...various components of what comes out of a female during menstruation? (I'm given to understand that it isn't just blood...) Is that any more risky? Also, what about eating a tumourous bone? That last one is just for laughs, really. The first one, I'm specifically thinking of oral sex. Vimescarrot (talk) 19:16, 7 June 2009 (UTC)[reply]
What are you, 12? Menstrual fluid doesn't get ingested during oral sex, mmmmkay? 94.27.225.206 (talk) 20:12, 7 June 2009 (UTC)[reply]
I see no reason why that isn't possible. Vimescarrot (talk) 20:33, 7 June 2009 (UTC)[reply]
If you give oral sex to a woman that is menstruating, I would expect some of the menstrual fluid to be ingested. --Tango (talk) 21:04, 7 June 2009 (UTC)[reply]
The amount would be so small though, would it really matter? —Preceding unsigned comment added by 82.43.88.87 (talk) 22:26, 7 June 2009 (UTC)[reply]
That's exactly what I'm asking. Vimescarrot (talk) 22:34, 7 June 2009 (UTC)[reply]
According to a (purportedly non-fiction) book I read some years ago, committing cunnilingus on a menstruating woman (with or without her consent) was an act for which US Hell's Angels could earn a Red Wings badge, or Black Wings if she were black. I mention this purely in the spirit of encyclopaedic completeness, not because I endorse the custom or attitudes involved in any way. 87.81.230.195 (talk) 23:55, 7 June 2009 (UTC)[reply]
This is a reference desk. Is it too much to ask for a few references, especially when dealing with matters of health? According to Aidsmap, a project of NAM (National AIDS Manual, UK registered charity, founded 1987):

The levels of HIV in vaginal fluid vary. They are likely to be highest around the time of menstruation (having your period), when HIV-bearing cells shed from the cervix are most likely to be found in vaginal fluid, along with blood. Oral sex will therefore be more risky around the time of menstruation. Factsheet here

According to The Body, a service of Body Health Resources Corporation:

It is important that you do not perform unprotected oral sex on a woman when she is menstruating (blood has a much higher concentration of HIV), and if you are experiencing any problems with your oral health (sores, abrasions, inflammation). If you would like to decrease the risk of transmission even further, you can use a latex barrier--like a dental dam or a condom cut into a square--between your partner's vagina and your mouth. Explanation here

Sources please! BrainyBabe (talk) 12:06, 8 June 2009 (UTC)[reply]
vampirism? —Preceding unsigned comment added by 82.43.88.87 (talk) 19:59, 7 June 2009 (UTC)[reply]
Not exactly human blood but animal blood (eg. pig blood) is a delicacy in Southern China; very smooth and tender, with a jello-like texture and a bit of iron after-taste. --antilivedT | C | G 23:06, 7 June 2009 (UTC)[reply]
I believe there is less risk from animal blood than human blood, since the diseases present in animal blood are less likely to be infectious to humans. --Tango (talk) 00:45, 8 June 2009 (UTC)[reply]
I seem to remember reading of a nomad culture where cattle are regularly bled for drinking; but maybe that was fictional. —Tamfang (talk) 04:43, 8 June 2009 (UTC)[reply]
The Maasai of East Africa certainly do that - they mix cow urine into a mixture of cow blood and milk to make it clot. According to our article, these days they only do this on ceremonial occasions or when someone is sick. When we lived in Kenya in the early 1970's, my parents bought a rather beautiful beaded gourd from a Maasai tribesman - sadly, they assumed it was a tourist trinket when in fact it was all-too authentic and still contained the guy's lunch! Despite a lot of cleaning, and soaking the insides with bleach, the...erm..."distinctive" odor of the thing never quite went away! SteveBaker (talk) 10:53, 8 June 2009 (UTC)[reply]
Well, since we are digressing, see Blood as food (alas, not a great article). Note that the consumption of raw duck and chicken blood in dishes that are delicacies in Vietnam is one of the few ways in which humans can catch so-called "bird flu" (H5N1) from birds, see BBC article. BrainyBabe (talk) 08:11, 8 June 2009 (UTC)[reply]
I believe there is less risk from animal blood than human blood, since the diseases present in animal blood are less likely to be infectious to humans. That is a rather risky belief given the number of counterexamples of diseases transferred from animals. BTW. Off hand I could probably come up with a shorter list that can not be transferred than ones that can. Just for tip of the iceberg examples see Rabies, Trichinella, Apicomplexa, vCJD, Cryptosporidiosis. Some of these get most commonly transferred by other means than consumption of blood, but that doesn't mean it can't be transmitted that way. Even intestinal parasites are often present in an infected individual's blood. Some allergologists think that humans need a certain exposure to parasites and infectious agents to prevent allergies. If only 1 in X gets infected it that figure won't make you happy if you turn out to be the one. 71.236.26.74 (talk) 15:59, 8 June 2009 (UTC)[reply]
See selection bias. Most diseases cannot be transferred from one species to another. The only animal diseases you have probably heard of (unless you are a vet) are ones that can be transferred to humans since those are the ones that make the news (or ones your pets have caught, I suppose). There is certainly some risk, but it is far lower than the risk of drinking human blood. Pretty much any human disease can be transferred by drinking human blood (the risk is rather low for some since stomach acid, etc., are quite good at killing ingested contagions, but there is still a significant risk). --Tango (talk) 16:54, 8 June 2009 (UTC)[reply]
We have an article, Zoonosis, but it doesn't seem to make comparisons to human-to-human infections. --Tango (talk) 16:58, 8 June 2009 (UTC)[reply]

Interesting discussion is interesting! I love digression - it's a great way to learn. Vimescarrot (talk) 17:14, 8 June 2009 (UTC)[reply]

You might e right about selection bias (although I do know plenty of diseased that affect only non-humans - and I'm still not a vet). There is however the factor of immunity that makes diseases that "jump species" worse than those within the same. Trusting expert opinion reported in the media in this matter our immune system is just having a lot harder time dealing with pathogens introduced via animal vectors than human to human. So, you could catch the flu by drinking human blood, but would your immune system be more likely to deal with that than a bird flue virus you ingested with chicken blood? 71.236.26.74 (talk) 19:48, 8 June 2009 (UTC)[reply]
Yes, I think that is true. Most human-to-human diseases are fairly minor (like the common cold), whereas animal-to-human diseases, while very rare, are often quite a lot worse. That isn't just due to immunity, it is due to evolution of the disease as well. Killing your host is not generally a good way to reproduce, so serious diseases in humans tend to die out in time, but if they were less serious in a given animal they could live on in that animal and only be serious when they get transmitted to humans (usually it would stay in the original human, diseases of animal origin that can be transmitted from human to human are very rare indeed - that's the big fear about bird flu, that it could mutate into a form that can be transmitted between humans, at the moment it can't so just kills a handful of people that have close contact with birds). --Tango (talk) 17:05, 9 June 2009 (UTC)[reply]

Surgical memento

If surgeons are removing something from you, are you allowed to keep it? Vimescarrot (talk) 14:03, 7 June 2009 (UTC)[reply]

Let me guess. No? Bus stop (talk) 14:06, 7 June 2009 (UTC)[reply]
Why not? And if that's a guess, why would you think that? Vimescarrot (talk) 14:08, 7 June 2009 (UTC)[reply]
I don't know. I'm just joking. I really don't know. But since intuitively I would assume one should have the right to take possession of a part of one's body that has been surgically removed from one's body, it probably is the case that it is the opposite. There is probably some reason why removed body parts become the property of the medical facility in which the surgery took place. But I really don't know. Bus stop (talk) 14:51, 7 June 2009 (UTC)[reply]
Human tissue is generally classified as biohazardous material, and has to be properly disposed of in a high temperature medical incinerator. So the hospital isn't allowed just to give you a decaying necrotic kidney home in a plastic bag. They'll give you an inanimate calculus like a gallstone, because there's no risk of disease. In theory they could preserve stuff in formaldehyde, but there are considerable additional compliance costs to do with human anatomy regulations and biomedical waste; it's not the hospital's core business, so they're unlikely to want to do all that stuff. Hopper Mine (talk) 15:45, 7 June 2009 (UTC)[reply]
Is bone an "inanimate calculus"? Vimescarrot (talk) 15:57, 7 June 2009 (UTC)[reply]
Very much not; it's a complex matrix full of living cells (it couldn't very well get cancer if it was inanimate). It is possible for a lab to sterilise bone (this is done to cadaverous bone allowing it to be safe for transplant), leaving only the inorganic hydroxylapatite matrix, but this is an involved and expensive process - it's not something they'd do for souvenirs. Hopper Mine (talk) 17:57, 7 June 2009 (UTC)[reply]

I guess it depends on what they are removing. My friend kept his gall bladder stones, but some things might be toxic?91.111.74.247 (talk) 14:55, 7 June 2009 (UTC)[reply]

A bone tumour. Vimescarrot (talk) 15:01, 7 June 2009 (UTC)[reply]
note that this is by the OP. So, OP, why would you want to keep a bone tumor??? 94.27.225.206 (talk) 15:47, 7 June 2009 (UTC)[reply]
Well, why do people keep teeth? Vimescarrot (talk) 15:55, 7 June 2009 (UTC)[reply]
To get money from the tooth fairies, obviously. Now, everyone knows that the bone cancer fairies are just a myth. Frank Bruno's Laugh (talk) 18:51, 7 June 2009 (UTC)[reply]
Good thing I don't have bone cancer, then...Vimescarrot (talk) 19:13, 7 June 2009 (UTC)[reply]
This may be of some help to you. Frank Bruno's Laugh (talk) 15:10, 7 June 2009 (UTC)[reply]
Maybe the OP was talking about surgeons removing a foreign object. If it's not dangerous, I see no reason why you couldn't have it. Of course, if it was a bullet, knife, or some other weapon, they might be obliged by law to hand it over to the police for forensic investigation. Astronaut (talk) 17:35, 7 June 2009 (UTC)[reply]
-just re-read that ... a bone tumour? Are you serious? Astronaut (talk) 17:36, 7 June 2009 (UTC)[reply]
Why would I not be serious? Vimescarrot (talk) 17:47, 7 June 2009 (UTC)[reply]
A cancer is no more hazardous to others than regular tissue of the same type (barring use in transplants, obviously, and maybe cancers associated with viruses). It's just a minor programming error, not some primal nuncio of feculence; it's not contagious. That said, all living human tissue is thick with a wide range of pathogens, and it's all considered hazardous for good reason. Hopper Mine (talk) 18:05, 7 June 2009 (UTC)[reply]
If you're just thinking "ew gross", it's surprisingly common for people to feel sentimentally attached to what is, after all, part of themselves. This is true even for tumours; some people seem to think "that's the little bastard that caused me so much trouble", and want to keep it as a trophy. Hopper Mine (talk) 18:14, 7 June 2009 (UTC)[reply]

It sounds like Hopper and Frank answered my question, although if anyone disagree they're welcome to say so. Thanks. Vimescarrot (talk) 19:13, 7 June 2009 (UTC)[reply]

Famous persons could sell their excised body parts for considerable sums. These would pass from hand to hand as collector's items or investments. Soft tissue would be in a sealed glass tube. There would be a registry of ownership to ensure against fraud. If authenticity is still questioned, DNA tests could be made. – GlowWorm. —Preceding unsigned comment added by 98.21.108.254 (talk) 03:39, 8 June 2009 (UTC)[reply]


In some cases where something is removed the surgeon may actually be required by the insurance company to keep what was removed or even send it (preserved) to the insurance company as proof the surgery was actually conducted. I don't know how common that requirement is. Someguy1221 (talk) 03:47, 8 June 2009 (UTC)[reply]

[citation needed] — all due respect, but I think you must have just made that up. I imagine an insurance adjuster in his cubicle taking 50 FedEx shipments per day that are all dripping pus and blood. Tempshill (talk) 04:27, 8 June 2009 (UTC)[reply]
Or rather, I said that because a close relative was unable to keep a bone that was removed for precisely that reason. I do think your image is pretty amusing, and is why I mentioned I don't know how common that is. The incident I mention was the first time I'd ever heard of it being a practice. Someguy1221 (talk) 02:33, 9 June 2009 (UTC)[reply]

Just last week, my wife came home from the hospital with two screws that were removed from her leg. (They had, of course, been installed in a previous surgery.) The screws were clean and shiny, and had been autoclaved after removal. -- Coneslayer (talk) 16:23, 8 June 2009 (UTC)[reply]

Inertia on the bus(non-technical explanation preferred)

When riding the bus, passengers experience inertia when it accelerates(they move backwards relative to the bus) and when it decelerates(they keep moving forwards). However, why does there seem to be less inertia(and consequently less risk of getting tossed around and crashing into things/people) when you're seated than when you're standing? I don't have much background in physics, so I'd like a non-technical answer. 69.224.113.202 (talk) 18:15, 7 June 2009 (UTC)[reply]

You have exactly the same inertia, whether you are seated or standing, because inertia only depends on your mass. While standing, though, your center of mass is higher, and you are generally in a less stable position, so you might feel the sways and even stumble around a bit as the bus accelerates. This is strictly due to how stable your stance is, not how much inertia you have. Nimur (talk) 18:18, 7 June 2009 (UTC)[reply]
To expand a little on what Nimur said: when you're sitting down and typically leaning back against the seat, that's a very stable position. Not only are you braced against the seat, your butt and the backs of your thighs against the seat afford you plenty of friction that tends to hold you on the seat, especially if the seat is made of something relatively coarse, like fabric, instead of a more slippery material. If I tried to yank you away from the seat, I'd probably have to use quite a bit of force. But if I wanted to do the same to you when you're standing, it'd be a lot easier, because not only would your center of mass be higher so it'd be much easier to tip you over, there wouldn't be a lot of friction to hold you in place. Unless you were prepared for me and had strongly braced yourself, I could probably yank you off your feet without any real trouble. Another way to look at this might be to imagine a rolled-up carpet in a bus. If you stand it up, it tips over very easily, but if you lay it down, it'll most likely pretty much stay put. If the roll starts to tip over, you're going to have to use quite a bit of strength to get it back upright, because at that point you'll be working against its own mass. That's what happens when you stand up and have to work to stay on your feet: once your balance starts to go, it's your own mass that's making you fall down, more than anything else. You need to work to re-balance the load that consists of your body. -- Captain Disdain (talk) 23:33, 7 June 2009 (UTC)[reply]
Similarly, notice that when you're standing, you get thrown around most if you don't use a handhold, less if you hold on with one hand, still less if you hold on with two. The force on you is the same, but the more you're braced, the more easily you can resist it. Sitting down braces you even more, and sitting down with a hand against the seat in front (if possible) does still better. --Anonymous, 3:55 UTC, June 8, 2009.
When a person is standing in a bus, the body's center of mass is at the end of a longer leverage arm than when the person is seated. Specifically, the axis of turning is at the feet rather than the buttocks. Thus when seated, the center of mass is only slightly above the center of turning.
Also, when a bus is decelerating, the abdominal muscles of a seated person resist the inertial effect rather than the weaker ankle muscles of a standing person. Specifically, when a person is seated, holding the abdominal muscles and knees rigid and bracing the feet on the floor opposes the turning inertia of the upper part of the body. When the bus is accelerating, the person is simply pressed back in his seat. Furthermore, the mass of the legs of a seated person is below the axis of turning, reducing the body mass above that point.
All this assumes there is no slippage on the seat or floor. Inertia thus results in a turning effect. - GloWorm. —Preceding unsigned comment added by 98.21.108.254 (talk) 03:59, 8 June 2009 (UTC)[reply]

Twins conceived on successive months

Further to the excellent sourced answer above, about the time between the delivery of twins (or other multiple births), I remembered a friend-of-a-friend story. It seemed improbable, but then, so did the idea of delivering one baby and then keeping the other in utero for more than a few minutes. Here is the idea -- twins are born: one is substantially bigger than the other, and hits all the developmental milestones a full month or more before the tinier sibling. What is the probability of the bigger being conceived one month, and the smaller the next, because ovulation for some strange reason was not suppressed? In other words, not "can you get pregnant if you do it standing up?" but "can you get pregnant if you are already pregnant?". BrainyBabe (talk) 20:19, 7 June 2009 (UTC)[reply]

I don't know the answer, but I think it is quite unlikely to happen - you don't just need ovulation to take place, but the embryo to successfully implant in the uterus lining. I know some contraceptive pills don't reliably prevent ovulation, but do prevent implantation, so I would imagine pregnancy could be similar (although ovulation is even more unlikely when pregnant than when on the mini-pill). --Tango (talk) 21:02, 7 June 2009 (UTC)[reply]
This being an encyclopedia, we have an article on this rare phenomenon: see superfetation. --Cookatoo.ergo.ZooM (talk) 21:03, 7 June 2009 (UTC)[reply]
Well, I knew it was quite unlikely to happen; I wanted to know if it was impossible. I am grateful for the pointer to the article, which I never would have found on my own! BrainyBabe (talk) 08:17, 8 June 2009 (UTC)[reply]

Submersed in Water

Is there a length of time after which it would become dangerous for a person to remain in water? Assuming access to a toilet and the water is freshly running would a person eventually get sick? I'm thinking there is some sort of limit as your fingers prune after too long. The Aquatic ape hypothesis, which albeit mostly unsupported, provides some possible insight, but I can't seem to find conclusive proof of a danger from extended exposure. TheFutureAwaits (talk) 21:15, 7 June 2009 (UTC)[reply]

Incidentally, why do fingers prune? And why don't toes prune? Does pruning take place in other land-dwelling animals after prolonged water exposure? —Preceding unsigned comment added by 98.21.108.254 (talk) 02:42, 8 June 2009 (UTC)[reply]
Toes do prune. Try it and see. As for the causes there is a section dedicated to this. Sifaka talk 03:13, 8 June 2009 (UTC)[reply]
OK. Toes prune too. It's been a long time since I took a bag lunch and spent the whole day at the swimming pool with my pals. (What fun we had !) - GlowWorm.
You might like to look at David Blaine - see Drowned Alive. DJ Clayworth (talk) 16:58, 9 June 2009 (UTC)[reply]
Or you might realise that David Blaine is a magician and is therefore irrelevant to this discussion. SteveBaker (talk) 03:27, 10 June 2009 (UTC)[reply]
Interesting, I'm sure I've ever heard anyone allege that the "Drowned Alive" stunt was an illusion. What would be the point? It seems like it would be entirely possible to do it straight, and rather difficult to fake? APL (talk) 04:07, 10 June 2009 (UTC)[reply]
Oh, or did you simply mean that the accounts of his injuries may be exaggerated (or censored) for dramatic publicity reasons, and therefore unreliable? APL (talk) 04:09, 10 June 2009 (UTC)[reply]

The need to urinate

Why is it that if I already need to urinate, the closer i get to a toilet the worse it gets? Like, for instance, if I'm driving home I may notice that I need to urinate, but have no trouble getting it, but once I get to the door of my house I start dancing around, and then it rapidly accelerates into a mad dash to the bathroom! Is it psychological? positional? --96.231.171.95 (talk) 22:46, 7 June 2009 (UTC)[reply]

If it's any comfort, I experience the very same thing. I've always assumed that it's an example of Pavlovian conditioning, and it's interesting that this still works even when one is consciously aware of it. 87.81.230.195 (talk) 23:42, 7 June 2009 (UTC)[reply]
Yep, it happens frequently enough to wonder just how Pablovian could it be if we are so concious of it happening and still can't control it? hydnjo (talk) 00:47, 8 June 2009 (UTC)[reply]
If this were an instance of conditioning, then we would expect the urge to pee to arise whenever a suitably coniditioned individual was near a bathroom, but this is clearly not the case. --Shaggorama (talk) 04:06, 8 June 2009 (UTC)[reply]
I disagree. I've gone into a bathroom to wash my hands or do something else that is only going to the bathroom in the literal sense, and felt the need to do it in the figurative sense. 67.182.169.172 (talk) —Preceding undated comment added 23:06, 9 June 2009 (UTC).[reply]
Something autonomic, I bet. Getting out of the car means you relax, relaxation makes you liable to pee. Once you've actually released the flood gates it's difficult to stop mid-flow, so I guess there's some kind of preparatory stage, a pre-pee process if you will, which is likewise difficult to stop. 213.122.32.39 (talk) 05:08, 8 June 2009 (UTC)[reply]

I suspect it's just that you have mentally 'released', you've made it to your goal (home) and then like 213.122 says once you've released it becomes much harder to stop so makes it more rushed to get there. I know that's how I explain it to myself. 194.221.133.226 (talk) 07:44, 8 June 2009 (UTC)[reply]

I think it's definitely psychological. If you suddenly get distracted by something that's important enough, the sensation will cease -- or at least that's how it goes for me. Sure, it takes a moment, but I've had a phone call knock me out of that "I gotta go right now" mode, for example. -- Captain Disdain (talk) 08:39, 8 June 2009 (UTC)[reply]
Pareto principle definitely. Jay (talk) 18:01, 9 June 2009 (UTC)[reply]

electrical generator

How does a generator work?

In particularly, I'm wondering how a wind turbine generates electricity? —Preceding unsigned comment added by 69.144.23.204 (talk) 23:23, 7 June 2009 (UTC)[reply]

See Electrical generator or more specifically, Electromagnetic induction. In short, voltage is produced across a conductor situated in a changing magnetic field or a conductor moving through a stationary magnetic field. -- Tcncv (talk) 00:00, 8 June 2009 (UTC)[reply]
As for wind turbine specifics, you may find the Wind turbine design article worth reading as well and many of the other articles linked from the "Wind Power" navigation box at the botton of that page. The conversion of wind energy to rotating mechanical energy and the conversion of rotating mechanical energy to electrical energy are best though of as two separate processes. -- Tcncv (talk) 00:15, 8 June 2009 (UTC)[reply]
In brief - when a magnetic field is moved past a wire, it generates a tiny amount of electricity. Spinning a magnet inside coils of wire (or spinning coils of wire inside some magnets) makes the magnetic field move past LOTS of wires - which makes a useful amount of electricity. The wind spins a shaft which spins the magnet (or the coil). It's basically an electric motor working 'backwards' - instead of putting electricity in and getting mechanical rotation out - you put mechanical rotation in and get electricity out. You can prove this yourself: If you take a small battery-operated electric motor (eg from a kid's toy) and connect a flashlight bulb across its terminals - you can spin the motor with your fingers and light the bulb. If you connect some fan blades to your motor and let the wind make it spin - then you've made your own personal wind turbine! SteveBaker (talk) 01:08, 8 June 2009 (UTC)[reply]
Are you sure that's true - I though they were wired in a different way. If it is true, then it would be easy to make a home wind turbine using an old washing-machine motor, although the electricity out of it would fluctuate greatly in its voltage, current, and Hz. 78.149.238.54 (talk) 19:38, 8 June 2009 (UTC)[reply]
It really has to be a DC motor. Washing machine motors run on alternating current - so probably this trick won't work (it depends on the design). But a direct current (DC) motor - pretty much any kids' toy or battery operated electric drill motor - will work wonderfully. And yes - of course you can make your own home wind turbine from bits and pieces you have lying around someplace...you can make nearly anything if you put your mind to it! A few years ago I made a computer controlled milling machine from an old router and some rollerskate bearings (and my current project is to re-make it with more power and precision). However - whether your home wind turbine would be efficient enough to be "useful" - is one of those devils that will be hiding in the details. SteveBaker (talk) 12:33, 9 June 2009 (UTC)[reply]
Ah ha! Now you are saying it won't work. "this trick won't work (it depends on the design)" - in other words the wiring is different as I wrote earlier. 78.147.57.78 (talk) 20:41, 9 June 2009 (UTC)[reply]
No - I'm saying that you have to read what I actually wrote:
"If you take a small battery-operated electric motor (eg from a kid's toy)..."
At no point did I say "It would be easy to make a home wind turbine using an old washing-machine motor"...those were your words. DC motors are NOT wired differently from DC generators - do the experiment it works just fine. AC motors may or may not be - it depends on the design...there are synchronous motors, squirrel-cage motors, induction motors, DC motors with AC rectification...some designs could work as generators - but others do not. It's the design of the motor that matters - not that "they were wired in a different way" as you said. AC motors are totally different beasts from DC motors. So I'm not changing my story - and I'm certainly not agreeing that you're right - I'm telling you to read what I wrote - which was correct. SteveBaker (talk) 03:25, 10 June 2009 (UTC)[reply]
It's absolutely true. A fun thing to do is to wire two small electrical motors to each other and turn the shaft on motor A and watch the shaft of motor B move. (Or maybe I'm just easily amused.) APL (talk) 00:41, 9 June 2009 (UTC)[reply]
Would that work? If you had two toy electric motors for example, I presume the output of one would be AC, yet the required input of the other would be DC. I'm still sceptical that electric motors would work in reverse as a dynamo without changing the wiring. Are there any YouTube videos of this? 89.242.125.32 (talk) 11:04, 9 June 2009 (UTC)[reply]
Yes - it works - providing you have two DC motors of similar size/voltage/etc. You can do it easily - just connect the wires from one motor onto the back of the other - turn one motor and the other one will turn as well. I've done this a gazillion times to demonstrate the effect - mostly using Lego Technic's motors (because you can connect them back-to-back very easily using the standard Lego connectors). Obviously there are losses in the system - so the second motor turns a bit more slowly than the first. But "Proof by YouTube" is a really REALLY bad idea because a solid 90% of the "science" videos on YouTube are faked. SteveBaker (talk) 12:33, 9 June 2009 (UTC)[reply]
Like SteveBaker said, A YouTube proof is worse than no proof at all. But this is a trick you can try yourself really easily. Get yourself two of these and wire them together. You don't even need to solder them, get some wires with alligator clips on them. I've done this many times, (Like I said, I'm easily amused), I promise that it works. APL (talk) 04:23, 10 June 2009 (UTC)[reply]

June 8

Plant without soil

I have this plant in my garden which grows on a rock. It does not touch any soil which I can see and I can move the whole rock (about 1 metre wide and long) with the plant on it anywhere. The plant grows on one face of the rock, about 1 metre long and has big leaves about 30 cm long. What kind of plant is this and how does it live without soil?

Thanks in advance, 220.244.76.121 (talk) 02:40, 8 June 2009 (UTC)[reply]

Some species of air plant maybe? SpinningSpark 03:12, 8 June 2009 (UTC)[reply]
Plants living on a rock or rocks are called lithophytes. Lithophytic plants are way too numerous to list. Can you post a picture of the plant somewhere? That would be the best. Telling us what part of the world you are living in would also help. --Dr Dima (talk) 03:38, 8 June 2009 (UTC)[reply]
Having said what I said, it may actually be a good idea if we at least start a list of lithophytic plants. The problem is, there are plants like Capparis sp. or Hyoscyamus sp. that thrive in cracks of a rock or in a crevice of a stone wall, but are far more often found in regular soil; and there are plants like some species of Laelia that are almost obligate lithophytes. Should we come up with a criterion, or just bundle them and let the reader read? --Dr Dima (talk) 03:48, 8 June 2009 (UTC)[reply]
Even trees can grow without soil. Here is one I photographed in Scotland. Plants only need air, water, and a tiny amount of nutrient. See hydroponics.--Shantavira|feed me

Alternative earphones with remote and microphone for iPod touch

moved to WP:RD/C#Alternative earphones with remote and microphone for iPod touch Nil Einne (talk) 17:01, 8 June 2009 (UTC)[reply]

Which vegetable oil is most healthy for vegetarians?

I'm looking for answers based on scientific evidence rather than something resulting from opinions formed from advertising or habitual use (eg olive oil). These two tables http://www.vaughns-1-pagers.com/food/vegetable-oils.htm http://www.annecollins.com/dietary-fat/omega-3-efa-6-chart.htm give lists of consituents of the oil, since I could not find a similar table on Wikipedia.

Canola oil is another name for rapeseed oil. I see that the amount of Omega3 in Canola oil differs considerably between the two tables - the small print in one of them says the amount listed is reduced during processing. I'm also wondering how much Omega3 there is in supermarket-bought 100% walnut oil, probably formed from hot-pressing, since neither of the labels on bottles from two different supermarkets mention it - yet I would have thought this would be a selling-point.

Fat summary: saturated fats - bad, should be avoided as far as possible. Transfats - very bad but not found in *liquid" vegetable oils. Monounsaturated - good, reduce bad cholesterol. Polyunsaturates - not so good as they reduce good cholesterol. But Omega3, which is a kind of polyunstaturated fat, is on the other hand good for you and said to be insufficient in peoples diets.

So given the above, which vegetable oil is best? Expense is not an issue. It may be that a blend of oils, determined by linear programming might be optimal. 78.149.143.187 (talk) 11:54, 8 June 2009 (UTC)[reply]

The question is clearly expressed, but I am not sure if the title is supposed to add an element to the mix. Is there a reason to believe that the healthiest oil for vegetarians would not also be the healthiest for omnivores? Also, the healthiest oil on strictly nutritional grounds may not be commercially available. Are you looking for something you can purchase and consume now, or a project to research? For example, argan oil and babassu oil are cultivated, and acorn oil to a lesser extent. You may care to investigate Sclerocarya birrea (S. caffra), aka the marula. BrainyBabe (talk) 12:21, 8 June 2009 (UTC)[reply]
We even have an article: Marula oil. BrainyBabe (talk) 12:28, 8 June 2009 (UTC)[reply]

I am looking for something available from supermarkets or health food shops. I do not know if the best oil for vegetarians would be the same as that for omnivores. Marula oil, at least, seems to have a lot of saturated fat in it. 89.242.81.255 (talk) 14:41, 8 June 2009 (UTC)[reply]

I don't think you can come up with a clear answer to this, since as with so many things when it comes to diet, there is still a fair amount of dispute. How can you come up with a 'best' when you aren't sure what's better? For example there is some evidence olive oil is good for you olive oil#nutrition but how good and why? As the phenolic compounds (and vitamins), which may be partially the cause of olive oil's alleged healthy properties, break down significantly in 12 months [15] the oil may be good in 2-3 months, not so good or at least no different if it's 12 months old. How significant the omega 3 to omega 6 ratio is another thing that remains unclear [16]. Another issue particularly since you talked about unusual oils. If these are organic or other speciality products, they may lack added antioxidants etc which despite the beliefs of those who think everything should be natural could be beneficial. Also, virgin olive oil and other oils with a low smoke point may not actually be a good oil for high temperature cooking (particularly anything over 200 degrees C) such as deep frying or perhaps traditional Asian style stir frying and high temperature roasting/baking due to the low smoke point [17]. Indeed this very high temperature cooking particularly deep frying is one area (IMHO) where the science is particularly unclear due to the complexity of what is formed, what breaks down, oxidises, etc [18] [19] [20]. Some of the beneficial compounds in olive oil may also decrease relatively rapidly for example [21]. And oils high in polyunsaturated fatty acids are prone to oxidation [22] [23] so it's possible an oil high in saturated fats may in fact be better then an oil high in PUFA for high temperature deep frying perhaps even if you only use the oil once. Remember that even if if the quantities formed are small, depending on how bad these are it could still be worse. This may be more of an issue in a restaurant etc but it does depend how often you reuse the oil (if at all), how long you deep fry for and temperatures. Home cooking is more likely to lack a temperature controlled deep fryer, filtering to remove crumbs and the deep fryer may be uncovered all of which are likely to be bad. And interesting enough, repeated heating and cooling of the oil may make things worse since the lower temperatures are apparently quite bad when it comes to oxidation [24]. Another factor, if you are referring to deep frying or any other case where you reuse the oil because of the oxidation, formation of potential carcinogens and other issues associated with repeated use, while you've said expense is not an issue, if using a more expensive oil means you will re-use it (more?), then it's potentially better for you to use the cheaper oil but refresh your oil more often. Of course since deep frying is not particularly healthy, it may just not do it, and things may not be so complicated. But then again, a perhaps interesting issue is the answer may even depend on the person. If you have a family background with a lot cases of cancer then you potentially should be more worried about carcinogens then if you have a family background with a lot of cases of coronary heart diease. One final thing that's worth remembering is where the money is coming from for research etc. Ge
You may wish to tryhempseed oil, which according to our article is extremely high in EFA's. However it is not suitable for high temperature frying. --TammyMoet (talk) 18:09, 8 June 2009 (UTC)[reply]

Sorry I should have added that I never fry food, except very ocasionaly vegetable stir-fry. The great majority of the oil would be eaten by drizzeling over pasta, salads, etc. And it was "BrainyBabe" who mentioned the exotic oils, not me. The choices I have in practice are between things like canola, olive, flax, or walnut oils, or combinations thereof. 78.149.238.54 (talk) 19:15, 8 June 2009 (UTC)[reply]

In that case, hempseed oil is the best bet. It tastes good too! --TammyMoet (talk) 08:15, 9 June 2009 (UTC)[reply]

Does cooking linseed make its oil accessable?

The Omega3 oil found in linseed, also called flax seeds, is believed to be beneficial. While flax oil is available it is expensive and tastes bitter in my experience. Flaxseed is much cheaper and tastes OK. But I understand that if you eat it raw it just goes straight through you without any of the oils inside the seed being digested, due to the seed-coat protecting them. To be digested it should be ground before eating - which is inconvenient to do.

My question is, does boiling linseed for 15 or 20 minutes (by adding to pasta or rice etc) rupture the seed coat and make the oils inside available to be digested? 78.149.143.187 (talk) 12:08, 8 June 2009 (UTC)[reply]

Yes it ruptures the seed coat. Whether it affects the quality of the oil is another question, as something boiled is not coldpressed. One simple way of ingesting raw linseed oil is to take a spoonful of linseed and chew it for 30 seconds, then swallow with a little water. I do not find the taste bitter. No need for coffee or spice grinders, etc. BrainyBabe (talk) 12:24, 8 June 2009 (UTC)[reply]
How do you know it ruptures the seed-coat please? 89.242.81.255 (talk) 14:51, 8 June 2009 (UTC)[reply]
Because if you chew with normal vigour for 30 seconds, not bite and gulp in two seconds but masticate with your molars for half a minute, the linseed becomes a paste. Try it yourself. Spit it out and look. Poke it with your finger. Can you see any intact seeds? Some might call that original research, frowned on here, but I would call it the empirical method. "Suck it and see", as they say, or in this case, spit it and see. This method is simple, free, requires no equipment, and does not depend on another level of processing (to gelcap form) with its own issues of stability, etc. Also, he enzymes in saliva help the process of digestion begin correctly; see that article. BrainyBabe (talk) 18:00, 8 June 2009 (UTC)[reply]
Sorry, what I meant was, how do you know it ruptures the seed-coat after being boiled for 15-20 minutes? 78.149.238.54 (talk) 19:23, 8 June 2009 (UTC)[reply]
Ah, the problem of the unclear referent! Again, perform the experiment. Boil a handful of linseed in a pot of water. (Leave out the additional ingredients such as rice, so that you are observing a single variable.) After 20 minutes' boil, turn off the heat. Look at it. Let it cool down. Poke it with your finger. Do you see any intact seed hulls? You can perform the same experiment with a grinder (electric or manual) or a mortar and pestle, for comparative purposes. BrainyBabe (talk) 06:33, 9 June 2009 (UTC)[reply]
FWIW, you can also get cold-pressed lin/flaxseed oil in gelcap form. No taste at all there. --Kurt Shaped Box (talk) 12:31, 8 June 2009 (UTC)[reply]
What you want out of your linseed oil (most likely) are polyunsaturated fatty acids. Heating oil will turn those into ordinary fatty acids at temperatures ard. 195 F- 265 F. In addition flaxseed oil also has it's Smoke point at the upper end of that range. Labels like "cold extracted" or "cold-pressed" make a lot of sense for your salad oil. For cooking oil it's less useful and can even be bad. For frying you have to keep and eye on the smoke point and Trans fat. (P.S.: For more info see Nil's detailed post above.) 71.236.26.74 (talk) 16:52, 8 June 2009 (UTC)[reply]
"Heating oil will turn those into ordinary fatty acids at temperatures ard. 195 F- 265 F." That's very interesting - do you think that is the reason why the supermarket walnut oil does not mention Omega3 on the label, despite giving the % composition of other fat types, because its likely to have been obtained by hot-pressing and thus the Omega3 has been turned into a more ordinary fat?? And, by the way, I almost never fry food. 78.149.238.54 (talk) 19:29, 8 June 2009 (UTC)[reply]
More likely because it isn't a mandatory part of Nutrition facts labels. The second reason is that "high in omega 3" is just as efficient for marketing and they can't be sued if some curious consumer group does some testing and finds there's not as much as they thought. Then there is the fact that omega3s are mot all that stable and do deteriorate after a while. With walnut oil you also don't get mega market manufacturers who can shell out money for testing that isn't requited by law. With all types of oil there's things they don't test for like Benzopyrene, Benzyl butyl phthalate or Bis(2-ethylhexyl) phthalate. Cyanide is another ref-desk favorite. Then there's Tannin which is good or bad for you depending on how much (total) you consume and whom you ask. Buying organic or at health food stores won't make a difference there, because their products are just as likely to contain the above as the stuff off the supermarket shelf. In short not everything is all that healthy as the media would make it seem once you start looking under the hood. And there is such a thing as way too much information. Use what you think tastes good and try to eat a varied and mixed diet. (If you have to be a vegetarian remember to take B12 supplement. You won't get that from any plants and your body will run out of it at some point. 71.236.26.74 (talk) 22:34, 8 June 2009 (UTC)[reply]
You are confusing vegetarians with vegans. 89.240.49.146 (talk) 22:48, 8 June 2009 (UTC)[reply]
By heating these polyunsaturated fats, you can polymerize them and make a skin. This is how oil paint works, although that may use oxygen as well. The branched chain fatty acid is very hard for humans to metabolise. (we need an article on these) Graeme Bartlett (talk) 22:04, 8 June 2009 (UTC)[reply]

Death by hypodermic to the heart?

My roommate and I noticed that "The Rock" was showing again yesterday. This inspired the conversation that led to me posting the following. Towards the end of the movie Nicolas Cage's character injects an antidote he desperately needs directly into his heart. One of us believes that this scene is realistic, at least in principle, believing that if the antidote had the proper effect quickly enough and if he had a high enough pain threshold, then the character should be ambulatory rather soon and functioning at nearly his normal level. The other of us believes that the process of injecting anything directly into the heart would lead to death rather quickly, given the required injury to the heart itself. I tried Googling this issue without much luck. Any thoughts, anyone? —Preceding unsigned comment added by 68.37.188.33 (talk) 15:01, 8 June 2009 (UTC)[reply]

Hopefully someone with more medical knowledge will reply in detail. but intracardiac injections are real. I think they're only used in extreme emergencies because of the obvious danger to the heart itself and the coronary artery. APL (talk) 15:10, 8 June 2009 (UTC)[reply]
Am personally skeptical that it would work like that, but its a great movie anyway. 65.121.141.34 (talk) 16:37, 8 June 2009 (UTC)[reply]
As APL said, intracardiac injection is real, but it is only done by professionals. Nicholas cage was not paying much attention to where he jabbed that needle, and wasn't working to hard to keep it still while it was in. It seems to me that he would have microscopically shredded the tissues in his heart, and would have died. Plus, with that quick jab, he could be injecting the antidote into the muscle instead of he blood stream, which would kill him.Drew Smith What I've done 21:23, 8 June 2009 (UTC)[reply]
In old murder mysteries, people were killed by a hatpin jab to the heart. There are also medical reports of a hatpin thrust into the heart causing death. Seems much like a hypodermic jab. Edison (talk) 17:06, 9 June 2009 (UTC)[reply]
FYI nerve gas antidotes in autoinjectors are not injected in the heart. AFAIK they're always injected in a muscle (e.g. on the outside of the thigh). It's doesn't make for such a dramatic scene, though.Sjö (talk) 19:24, 9 June 2009 (UTC)[reply]

heat into electricity

is there something maybe like a solar panel which can convert relatively large amounts of heat say a 10000 joules possessed by its surrounding air at a 1000 kelvin within a few seconds or even a few milliseconds? --harish (talk) 15:04, 8 June 2009 (UTC)[reply]

In principle this is just a heat engine - you pump a working fluid through a heat exchanger so it absorbs heat and maybe undergoes a phase change as well. Then you pass the hot fluid through a turbine which extracts work from it. While driving the turbine, the fluid expands and cools, so you pump it round again. In practice, 1000K is a high working temperature - for comparison, the superheated steam in a fossil fuel power plant has a maximum temperature of about 800K. So you probably need to borrow technology from the nuclear industry which is designed for higher working temperatures - maybe advanced gas-cooled reactor or very high temperature reactor technology. If you want to increase the power of your heat engine then you increase the surface area of your heat exchanger and pump fluid through at a faster rate. Gandalf61 (talk) 15:56, 8 June 2009 (UTC)[reply]
There is also the thermoelectric effect, which has some similarities to the photoelectric effect, but it is even less efficient in practice. Nimur (talk) 17:00, 8 June 2009 (UTC)[reply]
You really need a temperature DIFFERENCE in order to extract some energy. So if all of the machine's surrounding air is at 1000K, you've got to find a way of connecting the thing to something at much lower temperature. If you can do that then using some substance that is a gas at 1000K but a liquid at the lower temperature (water might be a reasonable choice) then water is flashed into steam by your 1000K heat source - is used to drive a turbine - the outlet from which is passed through cooling coils in the cold part of the system to condense back into water and continue around to the hot side again. A steam engine would work - so would a stirling engine...there are many possibilities...although it's hard to imagine anything that could do that within seconds. But if the entire machine is immersed in 1000K temperatures - there is nothing you can do to extract energy because that would violate the laws of thermodynamics. SteveBaker (talk) 18:40, 8 June 2009 (UTC)[reply]
It depends on exactly how you interpret the question - if it is one batch of hot stuff containing 10kJ being converted at once, then a few seconds is probably unrealistic, if it is a continuous process converting 10kJ per second, that is perfectly achievable. A typical fossil fuel power plant generates far more than 10kW. --Tango (talk) 18:56, 8 June 2009 (UTC)[reply]
The archetypal device that does this conversion is a Stirling engine. The max possible efficiency is determined by the Carnot cycle. 207.241.239.70 (talk) 19:25, 8 June 2009 (UTC)[reply]

How far do iron ore Lake Freighters travel from Minnesota or Northern Michigan, through the Soo Locks & past

the Mackinac Bridge then south on Lake Michigan to the steel mills in Gary, Indiana? —Preceding unsigned comment added by 72.204.36.33 (talk) 15:52, 8 June 2009 (UTC)[reply]

You should be able to figure this out yourself, I would think, by looking at a map and using its scale. You might use our articles about Duluth, Minnesota, Soo Locks, and Gary, Indiana to get an idea of where these places are. —Bkell (talk) 00:48, 9 June 2009 (UTC)[reply]
Using Google Earth, I get between 850 and 900 miles from Duluth to Gary. I tried to keep a short path but actual lake freighters have to stay in separated upbound and downbound shipping lanes so the the actual mileage may vary a little depending on the direction of travel. Rmhermen (talk) 02:58, 9 June 2009 (UTC)[reply]

porcupines

Are porcupine quills coated with venom or bacteria to cause infection? The article did not seem ot mention this one way or another. 65.121.141.34 (talk) 18:57, 8 June 2009 (UTC)[reply]

No, they are not. They are modified hairs - nothing more. The keratin makes them stiff. You may be thinking of hedgehogs. Many breeds of hedgehogs perform a ritual commonly called "anointing". They spread various chemicals (which may contain poisons) on their quills. Some people think that they do it increase the odds of infection in a predator. Others think it is just for camouflage. -- kainaw 19:03, 8 June 2009 (UTC)[reply]
I read in The Book of General Ignorance that the hedgehogs specifically chew poisonous frogs and suchlike and spread the resultant juice over their spines. Vimescarrot (talk) 21:37, 8 June 2009 (UTC)[reply]
Page 93. It also mentions the smells of many things can cause this behaviour, including shoes, cigar butts, furniture polish, creosote, coffee, boiled fish, face cream and distilled water. The animal world is a funny thing. Vimescarrot (talk) 21:39, 8 June 2009 (UTC)[reply]
Distilled water has a smell? (I mean, to any creature other than Shai-Hulud?) Tempshill (talk) 23:59, 8 June 2009 (UTC)[reply]
There are creatures far smaller than the mighty Sandworm that can nevertheless smell pure water quite well. Try "insect water receptors" in Google Scholar, and you will not lack the stuff to read for the evening :) --Dr Dima (talk) 01:13, 9 June 2009 (UTC)[reply]
I'd say The Book of General Ignorance is well titled. Richard Avery (talk) 06:56, 9 June 2009 (UTC)[reply]

While it true that porkies don't purposefully coat themselves in biological agents, they are not particularly "clean" animals. Part of thier mating ritual involves the male urinating on his partner. Yes a "natural" Golden Shower. They also "Den" for the winter and live with thier own faeces. Thier meat is parasite free and you can eat them raw (tastes awful though) but I wouldn't suggest licking one. 67.193.179.241 (talk) 10:35, 9 June 2009 (UTC) Rana sylvatica[reply]

Urine is not "dirty", and faeces (from a healthy animal) are not infectious (how would the porcupine survive the winter while "incubating" with it, if it were "dangerous"?). You may find this behaviour disgusting, but this does not make the animal unclean in any objective way. --TheMaster17 (talk) 12:40, 9 June 2009 (UTC)[reply]
Feces from healthy animal may still contain pathogens dangerous to other animals. APL (talk) 12:52, 9 June 2009 (UTC)[reply]
I read in The Book of General Ignorance that the hedgehogs specifically chew poisonous frogs.. Since Hedgehogs are not found in South America which is AFAIK the only place that has poison frogs: How the blazes can they chew them? (Help remedy my general ignorance:-)71.236.26.74 (talk) 18:13, 9 June 2009 (UTC)[reply]
They probably get them in pet stores.
Seriously, though. Poison Dart Frogs are not the only frogs that contain poisons. A bunch of them contain skin irritants that might be sufficient for the hedgehogs' purpose.
Or the book could have been completely wrong. Very possible. APL (talk) 18:34, 9 June 2009 (UTC)[reply]
If hedgehogs were to buy poison dart frogs in a pet store, they'd be in for a very bad surprise: captivity-bred poison dart frogs do not contain batrachotoxin! The poison dart frogs don't produce it, they get it from the beetles they eat; see the ref in batrachotoxin#Source. On the other hand, toad posion bufotenin is probably produced by the toad itself. --Dr Dima (talk) 20:50, 9 June 2009 (UTC)[reply]
Thought they might have mistaken a toad for a frog. Mistaking a frog for a toad isn't that unusual, but the other way round? Maybe the "General Ignorance" people should wait for the next edition of their own book ;-)71.236.26.74 (talk) 04:08, 10 June 2009 (UTC)[reply]

Name of those polystyrene packing forms?

Hi - I need a generic name for those shaped polystyrene blocks you find in boxes of new equipment. Is there one, can anyone help?

Thanks,

Adambrowne666 (talk) 20:20, 8 June 2009 (UTC)[reply]

Do you mean Foam peanuts? Or do you mean Expanded polystyrene shaped blocks like the ones used to protect electronic equipment in its shipping box? --Dr Dima (talk) 20:33, 8 June 2009 (UTC)[reply]
The latter, please, if there is a simple name for them. Adambrowne666 (talk) 20:43, 8 June 2009 (UTC)[reply]
They are called "molded expanded polystyrene" or "foam shipping blocks". --Dr Dima (talk) 20:48, 8 June 2009 (UTC)[reply]
Foam shipping blocks - perfect! - thanks for the prompt replies, Dr Dima Adambrowne666 (talk) 20:51, 8 June 2009 (UTC)[reply]
"Rigid plastic foam blocks" is another term used. —Preceding unsigned comment added by 98.17.43.186 (talk) 11:33, 9 June 2009 (UTC)[reply]
Styrofoam Cuddlyable3 (talk) 19:57, 9 June 2009 (UTC)[reply]
No, not Styrofoam. That is a common mistake. Styrofoam is a patented brand of polystyrene foam, and can come in any shape (e.g. sheets or disposable coffee cups), and not necessarily as shipping blocks. In other words, not all styrofoam is molded into shipping blocks, and not all shipping blocks are made of styrofoam. --Dr Dima (talk) 20:30, 9 June 2009 (UTC)[reply]

All matter Black hole

If all of the matter in the Universe fell into a Black hole how big would the Black hole be? -- Taxa (talk) 20:45, 8 June 2009 (UTC)[reply]

As big as the whole universe. --Jayron32.talk.contribs 21:02, 8 June 2009 (UTC)[reply]
That depends on the size and density of the universe. The entire universe is probably infinite, so that would result in an infinitely big black whole. If you mean just the observable universe, then it still depends on the density. I believe if the density were exactly equal to the critical density, then the Schwarzschild radius of the black hole would be equal to the current radius of the observable universe (the event horizon would essentially be the cosmological horizon). This isn't a coincidence and, in that situation, you could sort of think of the universe as being inside a black hole (emphasis on "sort of"). I believe current estimates put the density quite a bit lower than that, though, so the black hole would be smaller than the current observable universe. --Tango (talk) 21:09, 8 June 2009 (UTC)[reply]
BenRg seems to think that the statement above is not quite correct as you can see here. Dauto (talk) 02:49, 9 June 2009 (UTC)[reply]
I've done the maths, BenRG is correct (well, I get 390 billion light years, rather than 250 billion, but close enough). I've done some further calculations and it isn't the volume of the observable universe that corresponds to a black hole of critical density, it's the Hubble volume (which is often mistaken for the volume of the observable universe, which probably explains my mistake - either I, or whoever wrote wherever I read it, got them mixed up). (Actually, my calculations came out with it being about 10% off the Hubble volume, I'm not sure where the discrepancy came from - different sources for the various constants, probably.) --Tango (talk) 03:15, 9 June 2009 (UTC)[reply]
Could what we consider the universe be inside a black hole, without us being aware of it? 89.242.125.32 (talk) 10:32, 9 June 2009 (UTC)[reply]
If we are not aware, how should we answer your question? ;-) Scientific answer: As no theory at the moment is able to describe the central singularity of a black hole (where the mass is located), there is no way we could answer this. But you are free to speculate that these "coincidences" have a deeper meaning, it is just not proveable at the moment. --TheMaster17 (talk) 12:45, 9 June 2009 (UTC)[reply]
A black hole doesn't necessarily have all the mass at the centre. It takes time for the mass entering the black hole to get to the centre. General relativity tells us it will always get there in finite (proper) time, but it will spend some time not being there. If the universe was going to undergo a big crunch, that might be consistent with it being in a black hole (that isn't expected to happen, though, the expansion is measured to be accelerating, not slowing). There is a multiverse theory that involves universes in black holes, see Lee_Smolin#Fecund_universes. --Tango (talk) 16:58, 9 June 2009 (UTC)[reply]

June 9

Strange photometer problems

I was doing an experiment on Malus' law using a photometer, which was basically a light detector hooked up to a power supply unit and an ammeter. I turned up the sensitivity using a knob on the power supply, and starting measuring light intensity for different angles between the polarizers. For every angle that's not 85-95 degrees, I measured a current of 65.0-65.2 mA. The light falling on the photometer was obviously changing in brightness, but the ammeter reading was not! So I had to repeat the whole experiment using the lowest possible sensitivity, which gave better results.

Then, out of curiosity, I turned off my optical bench's light source. There was still a reading on the ammeter. I covered the photometer with my thumb, and the reading did not change AT ALL! I would attribute that to electronic noise, except I walked over to somebody else's photometer, which was EXACTLY THE SAME, and the ammeter read 0.0 when I covered it.

What's the explanation for my strange observations? Why didn't the ammeter readings change when I turned up the sensitivity? Why the reading when I covered the photometer? --99.237.234.104 (talk) 00:28, 9 June 2009 (UTC)[reply]

Your photometer is broken. SpinningSpark 15:56, 9 June 2009 (UTC)[reply]
Good--I wanted to make sure I wasn't missing something obvious about the operation of photometers. --99.237.234.104 (talk) 18:57, 9 June 2009 (UTC)[reply]

Plant species identification??

Could anyone help in identifying the plants in these photos. Here is the first entire plant [25] and here is a closeup of the flowers [26], the second entire plant is here [27] with a closeup of the flowers [28] The plants are growing in New Hampshire. Thanks --Captain-tucker (talk) 00:51, 9 June 2009 (UTC)[reply]

Some species of Lilac maybe? Not sure. We had lilacs in our yard growing up in New Hampshire, so I know they grow there. --Jayron32.talk.contribs 03:53, 9 June 2009 (UTC)[reply]
I'd say they are both members os the genus Weigela. The first possibly being weigela florida , the second I cannot determine. Originally from Asia but now common garden shrubs widely grown in temperate regions. Richard Avery (talk) 06:49, 9 June 2009 (UTC)[reply]

I concur but it is most likely a cultivar, bred for colour, flower size, duration of bloom and shrub size.67.193.179.241 (talk) 10:44, 9 June 2009 (UTC) Rana sylvatica[reply]

dynamics

plane kinetics of rigid body —Preceding unsigned comment added by Msohaibg (talkcontribs) 01:19, 9 June 2009 (UTC)[reply]

Do you have a question? The reference desk is not a search-engine, it is staffed by human volunteers, and we can best help you if you phrase your question in full sentences rather than keyword queries. Nimur (talk) 01:49, 9 June 2009 (UTC)[reply]

Surprising BBC statement about type 2 diabetes

This article http://news.bbc.co.uk/1/hi/health/8085762.stm says (in the info box on the right) that type 2 diabetes is a "Long-term condition caused by too much glucose in the blood". The first impression this gave me was that they are saying that eating sugary foods will give you diabetes, but that's a myth, isn't it? My understanding is that 1) being obese may aggravate a genetic predisposition to get diabetes, and 2) we don't really know what causes it.

Perhaps they mean something else, such as "characterized by" or "aggravated by" rather than "caused by"?

I am thin, with a sweet tooth, a state of affairs I enjoy, hence my interest. 213.122.54.27 (talk) 01:51, 9 June 2009 (UTC)[reply]

As I understand it, diabetes causes too much glucose in the blood, not the other way around. That seems like a mistake by the BBC - their science reporting is better than many news sources, but it still leaves a lot to be desired. --Tango (talk) 03:17, 9 June 2009 (UTC)[reply]
Actually, Diabetes mellitus type 2 IS caused by extended periods of elevated blood glucose. It is called "insulin resistant diabetes" also; and is a very different disease than Diabetes mellitus type 1. Type 1 Diabetes is basically a disease where your pancreas shut down and stops making insulin at all. No insulin means that your body stops processing glucose correctly. Type 2 Diabetes means that your body makes insulin just fine, but that your metabolism is out-of-whack and your body "resists" the insulin, and your glucose remains elevated at unsafe levels. Basically, with Type 2 Diabetes, if you constantly maintain a high blood-glucose level, your body gets used to that, and stops trying to lower it using insulin. It is caused mainly by eating too much high-glycemic index foods. Unlike Type 1 diabetes, much Type 2 Diabetes is mostly behavioral disease (except in the elderly and in a few other cases) and in the early stages is reversable with lifestyle changes. The BBC report is 100% accurate in the statement about what causes Type 2 Diabetes. --Jayron32.talk.contribs 03:49, 9 June 2009 (UTC)[reply]
I'd disagree slightly about this description of Type 2 Diabetes... While it is generally true that the pancreas "makes insulin just fine" the problem is that the pancreas eventually can't make enough insulin to overcome the peripheral insulin resistance. The problem develops slowly and insidiously, probably as a result of BOTH environmental/behavioral (poor diet, lack of exercise) and genetic components leading to impaired glucose (blood sugar) homeostasis. I agree with Tango that the high blood sugar is really an end result of a system that's out of whack. In the very early stages of the disease (what some would call "pre-diabetes" but which is really just part of the ongoing process), you might not have abnormally elevated blood sugar but rather abnormally elevated insulin levels, as the body struggles to keep the blood sugar in the normal range. Eventually, the peripheral resistance to the action of insulin becomes so great that even with maximal insulin production the body doesn't respond enough to keep the blood sugar level normal. Elevated blood sugar is what the diagnosis of "diabetes" is based on. This is why, as Jayron says, the early stages of the disease are reversible, since if you can fix the diet and increase exercise you can reduce the resistance of the peripheral tissues (mostly adipose tissue) so that they respond to insulin normally. What the BBC might have meant is that type 2 diabetes is characterized by high blood sugar, which then has a whole host of secondary effects (peripheral neuropathy, renal failure, coronary artery disease) that collectively make up the major morbidity of the condition. --- Medical geneticist (talk) 13:44, 9 June 2009 (UTC)[reply]
Sorry 213 but BBC is right, as Jayron pointed out. You should watch your sugar intake even if you are thin as a broom. Dauto (talk) 04:18, 9 June 2009 (UTC)[reply]
You need to be more specific. What exactly do you mean by "watch your sugar intake"? How much is too much? It would be a bit reactionary to tell someone that they can't enjoy a bite of chocolate or a cookie every so often. That is most certainly NOT going to "cause" diabetes. --- Medical geneticist (talk) 13:57, 9 June 2009 (UTC)[reply]
I think watch you sugar intake, while not that specific, is good enough in simple terms. If the OP wants to know more, they can ask or research themselves. I don't think it's misleading as it's not as if Dauto said you should never intake sugar or should cut your sugar intake down to zero. You can 'watch' you sugar intake fine even if you enjoy a bite of chocolate or cookie every so often. Similarly, it is fine general advice to watch you sodium intake or watch you intake of fats and oils. It doesn't mean you need to cut them completely out of your diet or that you can never eat a Pavlova or bag of chips Nil Einne (talk) 19:11, 9 June 2009 (UTC)[reply]
It IS misleading. Everyone has a different basal metabolic rate and different levels of physical activity. Some will do better with a low-fat diet, others should follow a low-carb diet. There isn't really a one-size-fits-all recommendation other than to maintain a healthy body weight and try to exercise regularly. The OP may be perfectly fine with whatever "sugar intake" s/he maintains. It isn't our place to give advice or opinions, just references. If the OP wants advice s/he should go to a nutritionist. (I'm not trying to invoke the medical advice restriction here, just pointing out that advice is complicated and "watch your sugar intake" is too vague and may not apply to everyone.) --- Medical geneticist (talk)
Calm down there. My statement was indeed prety vague, but not by accident. I gave a vague statement because I didn't know all the necessary information in order to give a less vague statement but I still wanted to convey the idea that just because someone is thin doesn't necessarily mean they can eat whatever amount of sugar they fill like without any health consequences. I think my statement was the best possible under the circunstances. I stand by what I said enougth to say it again. People should watch their sugar intake even if they are thin as a broom. Dauto (talk) 03:51, 10 June 2009 (UTC)[reply]
The high blood sugar in Type 2 diabetes is a symptom or result rather than a cause. The BBC is misleading. If a non-diabetic, who does not have impaired glucose tolerance eats some big sugar laden treat, it does not result in prolonged high blood sugar, thanks to the adequate release of insulin and the normal tissue response to it. See Glucose tolerance test. In a normal patient, even 75 g of sugar does not produce high blood sugar(over 7.8mmol or 140 mg/dl) 2 hours later. Edison (talk) 17:02, 9 June 2009 (UTC)[reply]
However, people who maintain high blood sugar levels over long periods of time, for example, have demonstrably higher incidences of Type 2 Diabetes. There is a correlation between the onset of Type 2 Diabetes and lifestyle... --Jayron32.talk.contribs 02:50, 10 June 2009 (UTC)[reply]

Reduction Potential in Pourbaix Diagrams

Hello. I am wondering how to calculate the reduction potential in Porubaix Diagrams. For example, the reduction potential between Fe3+ and FeO4 -2 in thisdiagram? Please! I have looked in my textbook, lecture notes, and online and I find nothing, and my exam is tomorrow! Thanks =) P.S The answer is NOT 0.4. P.PS Link to the diagram http://upload.wikimedia.org/wikipedia/commons/thumb/8/85/Pourbaix_Diagram_of_Iron.svg/605px-Pourbaix_Diagram_of_Iron.svg.png Cuban Cigar (talk) 10:10, 9 June 2009 (UTC)[reply]

It's not that hard. You don't give enough information to find the answer; but you also need to know the pH of the solution. At a given pH, draw a vertical line on the graph. The cell potential needed to make the transition between two states is where this vertical line crosses a region border on the graph. For example, at pH=2, the cell potential of the Fe2+ -> Fe reduction is E0 = ~-0.6 while the Fe3+ -> Fe2+ has E0 = ~0.79 and the FeO42- -> Fe3+ has E0 = ~1.75. The other forms in the chart are not stable at pH = 2 and could not exist. At other pH levels, there would be different values. --Jayron32.talk.contribs 12:13, 9 June 2009 (UTC)[reply]

Oh yes I forgot to include: the ph is 0. I'm still a bit confused though...is there a diagram that would explain it? I think a diagram would make it much more clear =)114.77.68.9 (talk) 12:51, 9 June 2009 (UTC)[reply]

OK, so if the pH=0 then use the vertical line at pH equals 0, and the reduction potential of each of the transitions is listed on the vertical axis. --Jayron32.talk.contribs 02:47, 10 June 2009 (UTC)[reply]

This is maybe rather a question concerning language than science: When Conway introduced this sequence in his 1986 article, he directly explained the rule and stated: "I note that more usually one is given a sequence such as [example] and asked to guess the generating rule or next term." Can we conclude from that whether he invented the system himself or not? If not - is there a way to ask him (there is no public contact address)? --KnightMove (talk) 11:33, 9 June 2009 (UTC)[reply]

Are you asking whether or not Conway invented the "Look and say sequence" (which is a specific sequence of numbers that obeys certain rules) OR are you asking if Conway invented the "Here's a series of numbers, identify the next number in the series" type of question? Because the former may be true, but the latter is definately not; it has been part of IQ tests such as WAIS since well before Conway was a working as a mathematician and likely before he was born. See Integer sequence for some examples. --Jayron32.talk.contribs 12:04, 9 June 2009 (UTC)[reply]
Also, you are likely to find better answers at the Mathematics Ref Desk. --Jayron32.talk.contribs 12:06, 9 June 2009 (UTC)[reply]
Thx for the hint. I was asking for the former, but you may start with any other sequence; Conway himself gave the example 55555 ; 55 ; 25 ; 1215 ; 11121115. --KnightMove (talk) 12:23, 9 June 2009 (UTC)[reply]
In that case, if you have access to the Journal he published the original 1986 paper in you should read that paper. I would guess that if he were basing his research on earlier work, he would clearly have listed his references which contained the more seminal work. The other option is that the concept of a "Look and say" sequence is old enough to have been lost in the mists of time; and as such does not have an original discoverer. But either way your best chance is to find his original paper and look there. --Jayron32.talk.contribs 12:32, 9 June 2009 (UTC)[reply]
These sorts of tests always infuriated me. As anyone who has ever studied polynomial interpolation can attest to, providing a subsequence of n numbers defines a countably infinite number of valid polynomials - so asking "what is the next number in this sequence" is making a huge number of unstated assumptions about the behavior of the overall sequence. Most commonly, a requirement is that all members of the sequence must be integers; but even then, with sufficient mathematical maneuvering, a countably infinite number of valid polynomials will still always fit. And if you're willing to use any other method of sequence generation (besides integer-sampled polynomials), you have an uncountably infinite set of sequences. Bluntly put, the people who ask these sorts of questions profess little to no knowledge of higher mathematics - and they intend to test somebody else's "IQ" ? Nimur (talk) 17:05, 9 June 2009 (UTC)[reply]
I've heard this sentiment before, but it seems tantamount to saying that science is impossible. The sequence is raw data and the answer you give is a theory, and some theories are better than others. I think almost anyone would agree that the sequence 55555, 55, 25, 1215, 11121115 is better explained by the look-and-say rule than by 5/6 (66666 − 3473104x + 6210289x2 − 3362372x3 + 558587x4). The latter is clearly not an explanation at all since it's longer than the data. The look-and-say rule I think is shorter than this data in whatever encoding system people use when judging the value of scientific theories. -- BenRG (talk) 18:55, 9 June 2009 (UTC)[reply]
There is another factor in science, though - the mechanism. A simple law is of limited use if you don't have a sensible theory regarding the mechanism behind that law. You can use the law to make predictions about precisely what the law is about, but nothing else, to come up with predictions about new things requires a mechanism. With these kinds of number sequences there is never any kind of mechanism, it is all completely arbitrary. --Tango (talk) 18:59, 9 June 2009 (UTC)[reply]
What do you mean by mechanism? Is that just another word for a model? Is that extra step always necessary or even useful? What is the mechanism behind quantum mechanics? Does that last question even make sense? Dauto (talk) 21:24, 9 June 2009 (UTC)[reply]
The mechanism behind something is what makes it happen. A scientific law is just a formula, for example Ohm's law is V=IR, the mechanism has to do with electrons bouncing around. Kepler's laws of planetary motion are just simple rules with no explanation, the mechanism behind them is the inverse square law of (Newtonian) gravity (Kepler didn't know that, he derived the laws empirically, and they were still useful before Newton explained them, but once you understand the mechanism behind them you can improve upon them - for example, you can introduce new factors to get more precise results). These examples aren't very good... The concept is easier to understand in something like biology. We may have results that say a particular drug increases someone's chances of surviving a disease by 10%, which is all well and good, but if we want to improve on that it is useful to know the mechanism by which the drug works - for example, it might inhibit a particular enzyme. If somebody makes a scientific claim without giving a plausible mechanism, you would be more sceptical of their results. So, if the sequence above arose in nature and someone wanted to predict what would come next, they would probably go with the polynomial extrapolation, rather than the look-and-say rule, since it is easier to think of a plausible mechanism for it. I can't think of any way natural could follow the look-and-say rule, but polynomials come up in all sorts of ways. --Tango (talk) 23:15, 9 June 2009 (UTC)[reply]

Dead arm while sleeping

I'm not sure where to find info on this, and in fact, I'm not even sure how to really look for it (is there no different term for limb numbness [i.e. that which isn't caused by some sort of condition] than parasthesia?), but I've always been rather prone to sleeping on my arm and then waking up finding it numb; as far as I can remember it would happen every few weeks or so, definitely nothing of concern. Lately it's been interrupting my sleep more though, yesterday I woke up three times! I sleep on my front (I can't sleep on my back -- so much so that I often turn on my back in the morning to prevent myself from falling back asleep!) and I'm pretty tall and skinny, which I think might make it easier for me to get caught up in my arms while I'm asleep, but I was wondering if anybody had any suggestions for how I stop my arm from getting squashed so much? I alternate between sides (front-ish orientation) and sleeping on my front, but my arm always seems to be able to find a position to get caught in! Thanks! 210.254.117.186 (talk) 12:17, 9 June 2009 (UTC)[reply]

Not that I have any recommendations, but the medical term for "limbs falling asleep", or rather going numb due to laying on them is called Paresthesia and is usually caused by compressing a nerve in the limb. The compressed nerve stops sending signals to the Central Nervous System, so your brain basically changes the way it interprets the signals from that limb. When the compression stops, the nerve begins to transmit signals to the CNS normally, but the CNS takes some time to "catch up" and re-adjust its processing to correctly interpret the signal. This "readjustment" period is what causes the numbness and "Pins and needles" feeling as the limb comes "back on line". Its exactly like walking from a dark room to a light room; your body does not adjust instantly so there is temporary (but harmless) discomfort during the adjustment phase. --Jayron32.talk.contribs 12:28, 9 June 2009 (UTC)[reply]

Anything that is an ongoing problem with sleeping is worth talking to a doctor about. I don't mean to sound alarmist, but there is always a chance that what's changed is something you aren't thinking of. Of course the Reference Desk does not allow medical advice, let alone speculation. --Anonymous, 12:47 UTC, June 9, 2009.

Assuming it is just a quirk of your nature and nothing medical a body pillow or memory foam mattress topper might be worth considering. But do check with your doc. 71.236.26.74 (talk) 14:33, 9 June 2009 (UTC)[reply]

Logic Gate Amplifiers.

File:DigitalInverterVTC.png
I built that on silicon in 2005, measured it, and it's one of my first contributions to Wikipedia! Nimur (talk) 17:08, 9 June 2009 (UTC)[reply]
  1. Where can I find articles and analysis on using NOT gates in their linear region as amplifiers?(TTL, CMOS, LS TTL, high speed CMOS)
  2. Also, up to what frequency can they be used?
  3. I want to see if I can use them to amplify signals in 10-50MHz range. —Preceding unsigned comment added by 59.93.15.239 (talk) 15:03, 9 June 2009 (UTC)[reply]
That's a blast from the past, haven't seen that trick used in a long while. Not sure where to find articles, don't think I've seen one since the internet started. Basically you need lots of negative feedback to keep it in the linear region. Probably won't work with most modern gates because they are heavily buffered to keep the device out of the linear region. It worked with older unbuffered CMOS because basically, you only had one transistor stage in there and it was an amplifier, just not being used as one. The frequency bandwidth it will work on depends on the speed of the device, rough rule of thumb, halve the maximum bit rate and call it Hz. SpinningSpark 15:23, 9 June 2009 (UTC)[reply]
You can check the Inverter (logic gate) article - and take a look at that fine image of a voltage transfer curve for a NOT gate operating in its linear region! (Un)fortunately, most commercial inverters designed for CMOS or TTL logic have much much higher gain, so you'll have to carefully select your operating point. Why wouldn't you just use a high-gain amplifier, which is designed for tunable bias points? Take a look at the datasheet of any amplifier you are interested in using. 50MHz is not at all unreasonable, but you're really going to have to watch your operating point, because unless you are right on the dead-center of the linear region, you'll likely trip the circuit to high- or low- output only, (after all, they are intended for digital circuits). Nimur (talk) 17:08, 9 June 2009 (UTC)[reply]
Also, read through the Application Notes for the 7404 inverter at Texas Instruments, especially Designing with Logic and Input and Output Characteristics of Digital Integrated Circuits (data). You may find the App Notes discussions a bit more high-level than the data-sheet (which is, of course, just the specifications). Nimur (talk) 17:48, 9 June 2009 (UTC)[reply]
A resistor connected from output to input of the NOT gate (inverter) suffices to bias it in its linear region. If the resistance is low enough for the voltage drop due to gate input current to be insignificant, the working point can be found simply by drawing a line from origin (0,0) to (2,2) on the transfer curve shown. The working point is where the line crosses the curve. Connect the input signal to the gate input via a capacitor (ac-only amplifier) or a resistor (dc and ac amplifier). The inverter shown has only about 2x voltage gain which limits its usefulness as an amplifier. Actual high gain CMOS and TTL inverters are easy to use this self-biased way and can provide the bandwidth you want. However you cannot use Schmitt trigger inverters that have no linear region. A useful application is to amplify a weak ac signal to a full logic swing. A clock oscillator can be constructed from one or two inverters. Cuddlyable3 (talk) 19:46, 9 June 2009 (UTC)[reply]

Help Me Preparing My Work

There is going to be held a competition in our medical school in which students have been asked to present their research works about any scientific topic. I am going to present my work about the relation that exists between structure of an object and the function realized by it, especially focused on human body going from atomic level to organism as a whole. I need answers of the questions like , if the protons are inside the nucleus then what importance it may have on the functioning of an atom and its role in biological systems, if glycogen has a cyclic structure then how it is important in its role in human body, if renal cortex has a vertical arterial system then why it is so, the functional importance of the cells participating in the structure of brain. Can someone guide me knowing answers of such questions and getting interesting information , Something precise PLZ. I searched on Pub Med but there is cluster of scattered things. Some web addresses, some opinions, some guidelines. Thanks —Preceding unsigned comment added by 200.55.135.211 (talk) 16:35, 9 June 2009 (UTC)[reply]

The topic "structure and function" sounds far too general for a presentation of less than encyclopedic length. For a med school presentation, your idea on the structure of glycogen, or the value of the arrangement of arterial structure of the kidneys are possibilities, but the function of the proton in the atom or the "function of cells in the brain" sound far too general. Maybe start with an interesting new development in one of these areas and make that the conclusion of your presentation. Edison (talk) 16:55, 9 June 2009 (UTC)[reply]
Are you looking to do a "report" on a topic (i.e. summarize a body of work) or are you really planning to prepare an independent "research project" where you identify a question and study it in a laboratory setting? It sounds like what you want to do is summarize what others have discovered about "structure and function" which, as pointed out by Edison, is far too broad-reaching a question to be summarized in a precise fashion. Perhaps you should pick an area to focus on. It doesn't matter what RefDesk contributors think is interesting, you should pick something that is deeply interesting to YOU so that you'll be motivated to do the best job. There are so many possible examples that it doesn't even make sense for us to suggest articles or books for you to start from. I'm sure you'll get better answers here (not to mention a better final result) if you focus your question first. By the way, glycogen is a branched polymer (of glucose molecules), not a cyclic structure. Glucose is cyclic. --- Medical geneticist (talk) 17:29, 9 June 2009 (UTC)[reply]
Careful with micro- vs macromolecular description: flycogen's structure discussion says it's a branched structure of glucose units, each of which is cyclic. DMacks (talk) 17:43, 9 June 2009 (UTC)[reply]
Isn't that what I said? Clarification added. BTW, I'd like to see what "flycogen looks like!  ;-) --- Medical geneticist (talk) 17:51, 9 June 2009 (UTC)[reply]
d'oh. google/googlescholar find a bunch of that typo in the literature too:) DMacks (talk) 18:04, 9 June 2009 (UTC)[reply]

Internet over radio (not internet radio)

Is it possible to connect everything through radio? Would it be much slower than fiber optic? Is it in use somewhere in the world? --Mr.K. (talk) 16:52, 9 June 2009 (UTC)[reply]

Many people access the internet via a wireless (radio) connection. At what point do you contemplate having it work by radio? There might be bandwidth problems if every home communicated with every other home at will by radio connected internet. Irt could bedone via cellphone-like technology, but the charges might be extremely high. Edison (talk) 16:58, 9 June 2009 (UTC)[reply]
Take a look at packet radio, which was a precursor to the modern 802.11 family of wireless protocalls. It is indeed possible to connect things by radio, but open air is a shared-channel by "unknown number" of users (unlike a wire, which is shared by N users, a maximum number pre-determined by the engineering specifications of the protocol). As per my earlier discussion about air-to-ground radio, available bandwidth is proportional to operating frequency (by physical limitation); and unfortunately, on earth (with atmosphere), higher frequencies have limited range. This means you need a hierarchical, hub-based "repeater" system - with all of the associated troubles of an untrusted network and a peer-to-peer network routing protocol (at the network switching level, not at the software-level as a modern file-sharing program uses "p2p"). In short, if you could control every radio in the world, and force it to comply with your protocol, then "wireless-only" internet would probably be a very efficient, low-infrastructure-cost alternative, and it would certainly solve the "last mile problem" that plagues wire-based network infrastructures. But, radio is a shared channel, and despite government regulations, you can't assure that your signals won't be interfered with by other users - so it's best to keep wireless transmissions short-range, high-bandwidth, and use lots of error-checking along the way. Nimur (talk) 17:23, 9 June 2009 (UTC)[reply]

endangered animals

OK, I have a question on how the endangered labels work. An example using data from our articles shows that the black rhino is critically endangered (the highest level of scarcity) with 3,600 animals left. The White Rhino is only near threatened (1 level lower then common) and it has 17,500 animals left. That does not seem like a very big difference. Can someone explain how this works? 65.121.141.34 (talk) 19:11, 9 June 2009 (UTC)[reply]

It may be worth reading IUCN_Red_List#Categories and look into the rules. I think this is the standard framework used for grouping species in terms of their endangered. ny156uk (talk) 20:57, 9 June 2009 (UTC)[reply]

Speed of light...

I was under the impression that it is impossible for anything with mass to move at the speed of light. Yet earlier when I asked about an object falling for infinity long (imagine it falling in a vacuum or something close to a vacuum such that its terminal velocity is greater then c), someone told me the relativistic mass would increase. Shouldn't the relativistic mass go to zero as v goes to c? —Preceding unsigned comment added by 24.171.145.63 (talk) 21:44, 9 June 2009 (UTC)[reply]

The rest mass of something traveling at c must be zero. The relativistic mass of something traveling at c need not be zero. However, to examine the condition you ask about, the mass (rest or relativistic) of something approaching c can be anything. — Lomn 22:02, 9 June 2009 (UTC)[reply]
(edit conflict) Interesting question. As I understand it, no object with mass can go faster than the speed of light, even if it is falling (and therefore accelerating) an infinitely long ways. Your friend is right that the object's mass would increase—as the object's velocity approaches c, some of the energy that is added to it with the intent of speeding it up is instead converted into mass through E=mc2. Interestingly enough, this would only increase the gravitational pull in your scenario, compounding the effect...I'm no physicist, but I imagine this scenario would violate a law of thermodynamics because energy is being constantly created. Also, gravitational pull decreases exponentially as you move farther out, so there would theoretically be no gravitational force making the object fall if the larger object is infinitely far away. Just tossing ideas out there—hopefully I'm correct.  :) —Pie4all88 T C 22:06, 9 June 2009 (UTC)[reply]
Definitely not. As an object speeds up, its relativistic mass increases, making it harder for it to go faster and impossible for it to reach the speed of light. You may be thinking along the lines that as matter approaches c, it becomes more like light, but that's not the case. Clarityfiend (talk) 22:26, 9 June 2009 (UTC)[reply]
It's an odd thing. The relativistic mass increase is a factor that gets larger and larger as you approach the speed of light - and AT the speed of light, it's infinite. So an object that has any mass at all at 'normal' speeds would have infinite mass at the speed of light. That's why things with mass can't go that fast - to give it infinite mass requires infinite energy. But for photons - which conveniently have a zero rest-mass - their mass at the speed of light is (in a sense) zero times infinity...which could be any number you could imagine. Hence they can have non-zero relativistic mass - but if they were to slow down by even the tiniest amount - their mass would drop to zero. SteveBaker (talk) 03:10, 10 June 2009 (UTC)[reply]

Limit to Acceleration?

Is there a limit to how fast something can accelerate? I imagine there would be, based on the inertia of an electron. —Pie4all88 T C 22:16, 9 June 2009 (UTC)[reply]

Do you mean a theoretical limit like the fact that speed is limited to the speed of light? I don't believe so. If you apply an arbitrarily large force, you'll get an arbitrarily large acceleration (it isn't proportional once you take relativity into account, of course). --Tango (talk) 22:31, 9 June 2009 (UTC)[reply]
The limits are essentially the amount of energy you can apply (which in theory is something like the mass of the visible universe times the speed of light squared!) and the structural limits of whatever it is that you're accelerating. Of course this insane acceleration could not be sustained for very long because relativistic effects will increase the effective mass of the thing you're accelerating...and thereby reduce the acceleration that your energy budget allows you to apply. But I can't think of any 'hard' limits. SteveBaker (talk) 03:01, 10 June 2009 (UTC)[reply]

How to safely hang a suspended sign like this one

Hello, thanks for taking a look at my question. I have been doing a lot of searching and I can't find a suitable answer to this.
I have seen a few instances of a sign being suspended over a roadway, often as sort of an "entryway" into a city or neighborhood. An example is this one hanging over Highway US 199 in Grants Pass, Oregon.
My questions are:

  1. Assuming the sign is held up by cable(s) hung between 2 vertical poles (one on either side of the road) with no other stays or trusses or guy wires, how would one calculate the needed materials for structural integrity? In other words, if the sign has a mass of X, and the distance between the poles is Y, then how would I find out how strong/thick the cables and poles must be? (We can assume the poles and cable are steel)
  2. Within the United States, is there any national building code or regulation that would cover a sign such as this, or is it up to local building codes in each jurisdiction? (We can assume the sign is hanging over a local street, and not over a state or national highway)

Thanks for your help! John —Preceding unsigned comment added by JohnMGarrison (talkcontribs) 22:22, 9 June 2009 (UTC)[reply]

See this e.g.[29]. Remember you can't just rigidly hang something with cables at full tension. You have to allow for wind, heat expansion/contraction of the material and the like. One of those engineering headaches where you get something like the Tacoma Narrows Bridge if you overlook something or underestimate some effect. 71.236.26.74 (talk) 23:21, 9 June 2009 (UTC)[reply]
Thanks for the link to the Interstate sign guidelines. For my question, I am just wondering about suspending over a local city street - would that be a matter of local codes? As for the structure, I am mainly interested in just supporting the weight. We can assume for now that the cables will allow for heat expansion, etc. JohnMGarrison (talk) 23:31, 9 June 2009 (UTC)[reply]
Check what DOT is in charge for your area. They also have lower level county/city ones. I don't know who does what, but I know you can't just hang as sign by yourself. You have to get them to do it for you (For the above reasons). Unless we are talking private road. They'd probably be able to help you with that, too. 71.236.26.74 (talk) 23:56, 9 June 2009 (UTC) P.S. in my experience when calculating suspending things like signs, weight is the least worry when calculating cable gauge and tension. I've long since forgotten most of what I knew. I just work with the guys who do stuff like that somewhere down the line. Even they now just type their numbers into the software. :-) 71.236.26.74 (talk) 00:12, 10 June 2009 (UTC)[reply]

June 10

Long term stability

From this site they said about :"long term stability time, what's the collisison of mercury, Earth, and Mars even mean, and what is the eccentricity of inner planets, and what the section mean about Pluto's eccentricity?--69.226.38.106 (talk) 01:01, 10 June 2009 (UTC)[reply]

The orbits of the planets (see orbital eccentricity) could change enough after billions of years for some of them to smack into each other (i.e. collide). Clarityfiend (talk) 02:29, 10 June 2009 (UTC)[reply]
In physics, there is a concept known as the n-body problem which basically states that for any value of n>2, the reliability of predicting the long-term motion of interacting bodies quickly drops to zero. In other words, where you have any group of objects which are all interacting with each other, if there are more than 2 objects in the group, their behavior is chaotic and their motion cannot be reliably predicted in the long term. In the short term, planets in our solar system behave like a 2-body problem (i.e. the planet itself and the Sun) since the gravitational effects of each planet on other planets is small, so on the order of, say, a few decades or a few hundred years, we can fairly accurately predict the motion of each planet by ignoring interplanetary gravitational effects. However, over time the system becomes less and less predictable because, over time, the planets gravitational effects on each other will sort of "add up" resulting in a system which diverges greatly from our predictions after several millions of years. What this all means for that article is that, while it looks like now the planets are in stable orbits, in 10-20 million years there is no way to predict how they will behave. We can be pretty sure the planets are not going to smash into each other simply by the fact that they have SO much space to move around in that even if their behavior becomes entirely unpredictable, it still only results in an infinitessimally small chance of collision between two of the major planets. However, what the chaotic behavior of the solar system means is that while we can be fairly certain the planets won't smash into each other, we can make no reliable predictions about exactly WHERE they will be in say another 50 million years. --Jayron32.talk.contribs 02:41, 10 June 2009 (UTC)[reply]
What I think they are saying is that while the average distance of these planets from the Sun may not change by much - the orbits may become more oval...less circular. If that's enough to make the orbit of one planet cross the orbit of another - then KAPOWW!!! - a very big mess! However, the timescale for these events is in the billions of years - more than you, I or humanity in general need worry about. Many of the events described are not due to happen for 5 billion years - and that's about as long as the Sun will last. So it's really kinda unimportant in the grand scheme of things. SteveBaker (talk) 02:54, 10 June 2009 (UTC)[reply]

Quantum physics paradox

Where's the mistake in this formula?

70.26.154.226 (talk) 04:48, 10 June 2009 (UTC)[reply]